Kolmogorov Complicity And The Parable Of Lightning

A good scientist, in other words, does not merely ignore conventional wisdom, but makes a special effort to break it. Scientists go looking for trouble.

Paul Graham, What You Can’t Say

I.

Staying on the subject of Dark Age myths: what about all those scientists burned at the stake for their discoveries?

Historical consensus declares this a myth invented by New Atheists. The Church was a great patron of science, no one believed in a flat earth, Galileo had it coming, et cetera. Unam Sanctam Catholicam presents some of these stories and explains why they’re less of a science-vs-religion slam dunk than generally supposed. Among my favorites:

Roger Bacon was a thirteenth century friar who made discoveries in mathematics, optics, and astronomy, and who was the first Westerner to research gunpowder. It seems (though records are unclear) that he was accused of heresy and died under house arrest. But this may have been because of his interest in weird prophecies, not because of his scientific researches.

Michael Servetus was a sixteenth-century anatomist who made some early discoveries about the circulatory and nervous system. He was arrested by Catholic authorities in France and fled to Geneva, where he was arrested by Protestant authorities, and burnt at the stake “atop a pyre of his own books”. But this was because of his heretical opinions on the Trinity, and not for any of his anatomical discoveries.

Lucilio Vanini was a philosopher/scientist/hermeticist/early heliocentrism proponent who was most notable as the first person recorded to have claimed that humans evolved from apes – though his theories and arguments were kind of confused and he probably got it right mostly by chance. City authorities arrested him for blasphemy, cut out his tongue, strangled him, and burned his body at the stake. But nobody cared about his views on evolution at the time; the exact charges are unclear but he was known to make claims like “all religious things are false”.

Pietro d’Abano was a fourteenth century philosopher and doctor who helped introduce Arabic medicine to the West. He was arrested by the Inquisition and accused of consorting with the Devil. He died before a verdict was reached, but the Inquisition finished the trial, found him guilty, and ordered his corpse burnt at the stake. But he wasn’t accused of consorting with the Devil because he was researching Arabic medicine. He was accused of consorting with the Devil because he was kind of consorting with the Devil – pretty much everyone including modern historians agree that he was super into occultism and wrote a bunch of grimoires and magical texts.

Giordano Bruno was a contemporary of Galileo’s. He also believed in heliocentrism, and promoted (originated?) the idea that the stars were other suns that might have other planets and other life-forms. He was arrested, tortured, and burned at the stake. But although his “innumerable worlds” thing was probably a strike against him, the church’s main gripe was his denial of Christ’s divinity.

I’m not a historian and I don’t want to debate any of these accounts. Let’s say they’re all true, let’s accept every excuse we’re given and accept the Church never burned anybody just for researching science. Scientists got in trouble for controversial views on non-scientific subjects like prophecies or the Trinity, or for political missteps.

Scott Aaronson writes about the the Kolmogorov option (suggested alternate title: “Kolmogorov complicity”). Mathematician Andrey Kolmogorov lived in the Soviet Union at a time when true freedom of thought was impossible. He reacted by saying whatever the Soviets wanted him to say about politics, while honorably pursuing truth in everything else. As a result, he not only made great discoveries, but gained enough status to protect other scientists, and to make occasional very careful forays into defending people who needed defending. He used his power to build an academic bubble where science could be done right and where minorities persecuted by the communist authorities (like Jews) could do their work in peace.

It’s tempting to imagine a world where Servetus, Bacon, and Bruno followed Aaronson’s advice. They pursued their work in optics, astronomy, anatomy, or whatever other subject, but were smart enough never to go near questions of religion. Maybe they would give beautiful speeches on how they had seen the grandeur of the heavens, but the true grandeur belonged to God and His faithful servant the Pope who was incidentally right about everything and extremely handsome. Maybe they would have ended up running great universities, funding other thinkers, and dying at a ripe old age.

Armed with this picture, one might tell Servetus and Bruno to lay off the challenges. Catholicism doesn’t seem quite true, but it’s not doing too much harm, really, and it helps keep the peace, and lots of people like it. Just ignore this one good prosocial falsehood that’s not bothering anybody, and then you can do whatever it is you want.

II.

But Kolmogorov represents an extreme: the politically savvy, emotionally mature scientist able to strategically manipulate tough situations. For the opposite extreme, consider Leonid Kantorovich.

Kantorovich was another Russian mathematician. He was studying linear optmization problems when he realized one of his results had important implications for running planned economies. He wrote the government a nice letter telling them that they were doing the economy all wrong and he could show them how to do it better. The government at this point happened to be Stalin during his “kill anybody who disagrees with me in any way” phase. Historians are completely flabbergasted that Kantorovich survived, and conjecture that maybe some mid-level bureaucrat felt sorry for him and erased all evidence the letter had ever existed. He was only in his 20s at the time, and it seems like later on he got more sophisticated and was able to weather Soviet politics about as well as anybody.

How could such a smart guy make such a stupid mistake? My guess: the Soviet government didn’t officially say “We will kill anyone who criticizes us”. They officially said “Comrade Stalin loves freedom and welcomes criticism from his fellow citizens”, and you had to have some basic level of cynicism and social competence to figure out that wasn’t true.

Even if the Soviet government had been more honest and admitted they were paranoid psychopaths, the exact implications aren’t clear. Kantorovich was a professor, he was writing about a very abstract level of economics close to his area of expertise, and he expressed his concerns privately to the government. Was that really the same as some random hooligan shouting “I hate Stalin!” on a street corner? Surely there were some highly-placed professors of unquestionable loyalty who had discussed economics with government officials before. Even a savvier version of Kantorovich would have to consider complicated questions of social status, connections, privileges, et cetera. The real version of Kantorovich showed no signs of knowing any of those issues even existed.

If you think it’s impossible to be that oblivious, you’re wrong. Every couple of weeks, I have friends ask me “Hey, do you know if I could get in trouble for saying [THING THAT THEY WILL DEFINITELY GET IN TROUBLE FOR SAYING]?” When I stare at them open-mouthed, they follow with “Well, what if I start by specifying that I’m not a bad person and I just honestly think it might be true?” I am half-tempted to hire babysitters for these people to make sure they’re not sending disapproving letters to Stalin in their spare time.

The average person who grows up in a censored society may not even realize for a while that the censorship exists, let alone know its exact limits, let alone understand that the censors are not their friends and aren’t interested in proofs that the orthodoxy is wrong. Given enough time, such a person can become a savvy Kolmogorov who sees the censorship clearly, knows its limits, and understands how to skirt them. If they’re really lucky, they may even get something-like-common-knowledge that there are other Kolmogorovs out there who know this stuff, and that it’s not their job to be a lone voice crying in the wilderness. But they’re going to have a really cringeworthy edgelord period until they reach that level.

All of this would be fine except that, as Graham says in the quote above, scientists go looking for trouble. The first virtue is curiosity. I don’t know how the internal experience of curiosity works for other people, but to me it’s a sort of itch I get when the pieces don’t fit together and I need to pick at them until they do. I’ve talked to some actual scientists who have this way stronger than I do. An intellectually curious person is a heat-seeking missile programmed to seek out failures in existing epistemic paradigms. God help them if they find one before they get enough political sophistication to determine which targets are safe.

Did Giordano Bruno die for his astronomical discoveries or his atheism? False dichotomy: you can’t have a mind that questions the stars but never thinks to question the Bible. The best you can do is have a Bruno who questions both, but is savvy enough to know which questions he can get away with saying out loud. And the real Bruno wasn’t that savvy.

III.

So imagine the most irrelevant orthodoxy you can think of. Let’s say tomorrow, the government chooses “lightning comes after thunder” as their hill to die on. They come up with some BS justification like how atmospheric moisture in a thunderstorm slows the speed of light. If you think you see lightning before thunder, you’re confused – there’s lots of lightning and thunder during storms, maybe you grouped them together wrong. Word comes down from the UN, the White House, the Kremlin, Zhongnanhai, the Vatican, etc – everyone must believe this. Senior professors and funding agencies are all on board. From a scientific-truth point of view it’s kind of a disaster. But who cares? Nothing at all depends on this. Even the meteorologists don’t really care. What’s the worst-case scenario?

The problem is, nobody can say “Lightning comes before thunder, but our social norm is to pretend otherwise”. They have to say “We love objective truth-seeking, and we’ve discovered that lightning does not come before thunder”. And so the Kantoroviches of the world will believe that’s what they really think, and try to write polite letters correcting them.

The more curiosity someone has about the world, and the more they feel deep in their gut that Nature ought to fit together – the more likely the lightning thing will bother them. Somebody’s going to check how light works and realize that rain can’t possibly slow it down that much. Someone else will see claims about lightning preceding thunder in old books, and realize how strange it was for the ancients to get something so simple so wrong so consistently. Someone else will just be an obsessive observer of the natural world, and be very sure they weren’t counting thunderclaps and lightning bolts in the wrong order. And the more perceptive and truth-seeking these people are, the more likely they’ll speak, say “Hey, I think we’ve got the lightning thing wrong” and not shut up about it, and society will have to destroy them.

And the better a school or professor is, the better they train their students to question everything and really try to understand the natural world, the more likely their students will speak up about the lightning issue. The government will make demands – close down the offending schools, fire the offending academics. Good teachers will be systematically removed from the teaching profession; bad teachers will be systematically promoted. Any educational method that successfully instills curiosity and the scientific spirit will become too dangerous to touch; any that encourage rote repetition of approved truths will get the stamp of approval.

Some other beliefs will be found to correlate heavily with lightning-heresy. Maybe atheists are more often lightning-heretics; maybe believers in global warming are too. The enemies of these groups will have a new cudgel to beat them with, “If you believers in global warming are so smart and scientific, how come so many of you believe in lightning, huh?” Even the savvy Kolmogorovs within the global warming community will be forced to admit that their theory just seems to attract uniquely crappy people. It won’t be very convincing. Any position correlated with being truth-seeking and intelligent will be always on the retreat, having to forever apologize that so many members of their movement screw up the lightning question so badly.

Some people in the know will try to warn their friends and students – “Look, just between you and me, lightning obviously comes before thunder, but for the love of God don’t say that in public“. Just as long as they’re sure that student will never want to blackmail them later. And won’t be able to gain anything by ratting them out. And that nobody will hack their private email ten years later, then get them fired or imprisoned or burned at the stake or whatever the appropriate punishment for lightning-heresy is. It will become well-known that certain academic fields like physics and mathematics are full of crypto-lightning-heretics. Everyone will agree that the intelligentsia are useless eggheads who are probably good at some specific problems, but so blind to the context of important real-world issues that they can’t be trusted on anything less abstruse than e equalling mc squared. Dishonest careerists willing to go in front of the camera and say “I can reassure everyone, as a physicist that physics proves sound can travel faster than light, and any scientists saying otherwise are just liars and traitors” will get all the department chairs and positions of power.

But the biggest threat is to epistemology. The idea that everything in the world fits together, that all knowledge is worth having and should be pursued to the bitter end, that if you tell one lie the truth is forever after your enemy – all of this is incompatible with even as stupid a mistruth as switching around thunder and lightning. People trying to make sense of the world will smash their head against the glaring inconsistency where the speed of light must be calculated one way in thunderstorms and another way everywhere else. Try to start a truth-seeking community, and some well-meaning idiot will ask “Hey, if we’re about pursuing truth, maybe one fun place to pursue truth would be this whole lightning thing that has everyone all worked up, what does everybody think about this?” They will do this in perfect innocence, because they don’t know that everyone else has already thought about it and agreed to pretend it’s true. And you can’t just tell them that, because then you’re admitting you don’t really think it’s true. And why should they even believe you if you tell them? Would you present your evidence? Would you dare?

The Kolmogorov option is only costless when it’s common knowledge that the orthodoxies are lies, that everyone knows the orthodoxies are lies, that everyone knows everyone knows the orthodoxies are lies, etc. But this is never common knowledge – that’s what it means to say the orthodoxies are still orthodox. Kolmogorov’s curse is to watch slowly from his bubble as everyone less savvy than he is gets destroyed. The smartest and most honest will be destroyed first. Then any institution that reliably produces intellect or honesty. Then any philosophy that allows such institutions. It will all be totally pointless, done for the sake of something as stupid as lightning preceding thunder. But it will happen anyway. Then he and all the other savvy people can try to pick up the pieces as best they can, mourn their comrades, and watch the same thing happen all over again in the next generation.

The Church didn’t lift a finger against science. It just accidentally created a honeytrap that attracted and destroyed scientifically curious people. And any insistence on a false idea, no matter how harmless and well-intentioned, risks doing the same.

IV.

I’m not against the Kolmogorov Option. It’s nothing more than a band-aid on the problems that even a harmless orthodoxy will cause – but if there’s no way to get rid of the orthodoxy, the band-aid is better than nothing. But politically-savvy Kolmogorov types can’t just build a bubble. They have to build a whisper network.

They have to build a system that reliably communicates the state of society. “Stalin claims that he welcomes advice from everyone, but actually he will kill you if you try to give it.” Or “God probably doesn’t exist, but lots of us know this, and we all just go to Mass and mouth the right words anyway.”

This is harder than it sounds. A medieval monk being told God doesn’t exist probably has a lot of questions. He’s likely to go kind of crazy for a while, crave the worldview-shards that he needs to rebuild his fractured philosophy. “What about Heaven? Does that exist? Where do we go when we die?” (“Psssst, Epicurus has some good arguments for why the soul doesn’t survive death, you can get a copy of his books from the monastery library.”) You might even have to prevent overcorrection: “Is there even such a place as Jerusalem?” (“Yes, now you’re just being silly, Brother Michael went there on Crusade and says it’s very nice.”)

(when a heretical belief turns out to really be completely wrong – maybe occultism would be a good example here – a whisper network might be the only place where you could get high-enough-quality debate to be sure.)

They have to serve as psychological support. People who disagree with an orthodoxy can start hating themselves – the classic example is the atheist raised religious who worries they’re an evil person or bound for Hell – and the faster they can be connected with other people, the more likely they are to get through.

They have to help people get through their edgelord phase as quickly as possible. “No, you’re not allowed to say this. Yes, it could be true. No, you’re not allowed to say this one either. Yes, that one also could be true as best we can tell. This thing here you actually are allowed to say still, and it’s pretty useful, so do try to push back on that and maybe we can defend some of the space we’ve still got left.”

They have to find at-risk thinkers who had started to identify holes in the orthodoxy, communicate that they might be right but that it could be dangerous to go public, fill in whatever gaps are necessary to make their worldview consistent again, prevent overcorrection, and communicate some intuitions about exactly which areas to avoid. For this purpose, they might occasionally let themselves be seen associating with slightly heretical positions, so that they stand out to proto-heretics as a good source of information. They might very occasionally make calculated strikes against orthodox overreach in order to relieve some of their own burdens. The rest of the time, they would just stay quiet and do good work in their own fields.

Such a whisper network would be in the best interests of the orthodox authorities. Instead of having to waste their good scientists, they could let the good scientists could join the whisper network, learn which topics to avoid, and do good science without stepping on orthodox toes. But the authorities couldn’t just say this. Maybe they wouldn’t even think of it, and nobody (except maybe Kantorovich) would be dumb enough to try to tell them. Individual secret policemen are always going to see the written law – “arrest heretics” – and consider the whisper network a legitimate target. Kolmogorov is doing the Lord’s work, but that won’t give him a pass from the Inquisition.

His reward will be that people with a drive to make the world make sense – to have everything fit together seamlessly and beautifully – will be able to quietly collect all the orthodox and all the heretical pieces, satisfy themselves, and then move on to doing good work in math or physics or whatever harmless field doesn’t affect Christianity or Marxism or lightning or whatever. Academies other than the worst and most curiosity-crushing have a little better chance to endure; academic bureaucrats other than the most slavish have a little more chance to remain in their position.

But also: maybe this is how common knowledge spreads. Maybe some atheists survive, go into science, become vaguely aware of each other’s existence, feel like they have safety in numbers, get a little bolder, and maybe the Church decides it’s not worth killing all of them. Maybe everyone stays quiet until Mao dies, and then Deng and Zhao look at each other and say “So, just between you and me, all of that was totally insane, right?” I don’t know how often this happens. But the chances seem better than for open defiance followed by certain retribution.

This entry was posted in Uncategorized. Bookmark the permalink.

616 Responses to Kolmogorov Complicity And The Parable Of Lightning

  1. jdwill07 says:

    “Let’s say tomorrow, the government chooses “lightning comes after thunder” as their hill to die on. ”

    What if you took a more subtle one, like CO2 increase precedes warming?

    You could have some real fun with that.

  2. loldrup says:

    The blanket dismissal of *all* corruption theories, prior to their actual investigation, seems to me to be a good example of this.
    I mean, which philosopher, metaphysicist, scientist or similar epistemic authority wrote the paper that concluded that, yes indeed, we can make that broad a priori judgement about all corruption theories? And where is this guys paper on that? I want to read it.

    PS. I Hope I spelled ‘corruption theory’ right..? I might have confused the wording a bit.

    PPS. Here’s an article about how how news media are handling *conspiracy theories*:
    https://truthandshadows.wordpress.com/2017/10/29/media-ignore-911-scoop-of-the-century/

    PPPS. I, of course, strongly condemn all conspiracy theories! They are all just silly and false, obviously!

    • Nornagest says:

      We put a low prior on conspiracy theories because there are a number of brain glitches that make them intuitively appealing way out of proportion to their actual feasibility: we’re more comfortable thinking about hostile agents than anonymous structural problems, our intuitions about social dynamics don’t scale up to the numbers of people necessary to carry out any respectable conspiracy, etc.

      That doesn’t mean conspiracy theories never turn out to be true — a handful have. It just means that’s not the way to bet.

      • loldrup says:

        So what is the decision theory canon on when to investigate hypotheses that have low priors?
        Should one *never* investigate them?

        If so, how is this default modulated by such things as:
        * level of uncertainty of the prior in question ( some priors are better grounded in reality than others )
        * The potential importance of said hypotheses. It might be worth investigating something that has a low prior IF the potential consequence of that hypothesis being true is that much high than your average hypothesis.
        I would love to read some good argumentation for the rational approach to conspiracy theories. It usually stops at “… well, the prior is low, so, there you go”.

      • loldrup says:

        I could imagine that several topics in the scientific discipline ‘psychology’ might have similar honey trap properties: they invite certain conclusion to be drawn because these conclusions seem attractive to our psyche.
        Does that mean we could never ever approach these topics in psychology?
        Or that we should never ever do so?

        There also seems a problem in how to ever reach any sort of update on our priors from some posterior piece of evidence. If one of us goes and investigate a given conspiracy theory, and in that process derives a posterior that deviates from our consensus prior, then obviously our response to this person will be “well, it seems you fell into the honey trap”.
        Which has the implication that, under the current rationalistic approach to conspiracy theories, we can never ever get to update our priors. Which is, ehm, an interesting situation! And a situation which seems to short circuit the Bayesian chain of updated probability estimates that we’re supposed to condone.

      • albatross11 says:

        I think “conspiracy theory” needs a bit more definition.

        There is a particular broken mode of thinking often applied to conspiracies, in which all possible evidence is somehow interpreted as being consistent with/re-enforcing the conspiracy theory. That tends to make the theory’s enthusiasts evidence-proof. When we talk about conspiracy theories, we might be talking about this mode of thinking.

        There’s also a set of socially unacceptable (in most of US society, anyway) theories involving conspiracies, which tend to be pretty elaborate and fantastic. Often most of the adherents of those theories are running on the conspiracy theory mode of thinking.

        Those are quite different. Some conspiracy theories (mostly supported by the broken mode of thinking) are commonplace in US culture, and in fact are pretty mainstream. A *lot* of discussions of racism, sexism, the Patriarchy, the Rich, etc., happen in the conspiracy-theory mode of thinking.

        Also, a lot of *actual* conspiracies that we have good documentation for now were pretty sketchy, socially-unacceptable fringe theories that only weirdos held. (The US network of secret prisons/torture chambers run by the CIA, the Catholic Church’s sex scandal, the LIBOR scandal, US domestic surveillance programs revealed by Snowden et al., Madoff’s massive Ponzi scheme, etc.) And some people claiming those were going on before they came out in public were probably running on the broken conspiracy theory mode of thought, but the conspiracies were real, and were ignored by respectable news sources/society until they became socially validated.

        • loldrup says:

          I agree with all you said, but your reply doen’t address this problem (quoted from my earlier comment):

          “There also seems a problem in how to ever reach any sort of update on our priors from some posterior piece of evidence. If one of us goes and investigate a given conspiracy theory, and in that process derives a posterior that deviates from our consensus prior, then obviously our response to this person will be “well, it seems you fell into the honey trap”.
          Which has the implication that, under the current rationalistic approach to conspiracy theories, we can never ever get to update our priors. Which is, ehm, an interesting situation! And a situation which seems to short circuit the Bayesian chain of updated probability estimates that we’re supposed to condone.”

          By now we’ve all spent at least 10 minutes on this question. We could also have used this time to read some piece of alleged evidence for some conspiracy theory. But we shouldn’t do such a thing because… we are rational…?

          • loldrup says:

            Jon wrote:
            “There also seems a problem in how to ever reach any sort of update on our priors from some posterior piece of evidence. If one of us goes and investigate a given conspiracy theory, and in that process derives a posterior that deviates from our consensus prior, then obviously our response to this person will be “well, it seems you fell into the honey trap”.”

            This attitude can be rephrased as: “the more you study a conspiratory topic, the less you know about it (because you will get honey trapped)”.
            Or as: “when it comes to conspiracy theories, your prior probability estimates will always have higher reliability than any posterior probability estimates you might derive”.

            This is a very interesting stance. But how does it look in the perspective of theory of science, Bayesian theory or ‘epistemic psychology’ (if that’s a thing)? Can it be rephrased/integrated meaningfully in any existing framework in these fields?

            In other words: does this interesting stance have *any* sort of scientific or theoretical backing?

            Or is it just a fancy rationalisation for us to avoid having to touch icky topics?

          • albatross11 says:

            Imagine making a chart. On the X axis is how socially acceptable a description of the world is, from left (maximally unacceptable–say this stuff and everyone dismisses you as a nut) to right (maximally acceptable–the stuff all right-thinking people belief). On the Y axis is how good a description of reality it is–how well it fits the available evidence.

            I think there are commonly-known models in all four quadrants. For example:

            a. Widely accepted and good fit for reality:
            (Law of supply and demand)

            b. Socially unacceptable and good fit for reality:
            (IQ tests as a good proxy for mental ability)

            c. Widely accepted and bad fit for reality:
            (Sexism as main cause of gender wage gap)

            d. Socially unacceptable and bad fit for reality
            (Vaccines cause autism)

            I suspect if you plotted a lot of serious attempts to describe the world on a chart like this, you’d see a broad trendline that was going up to the right–the better fit for reality the model is, the more likely it is to be socially acceptable. But you’d also find a lot of good descriptions of the world, with real insights to be had, in that top left quadrant (socially unacceptable but a good model), and a lot of really popular but wrong/dumb descriptions of the world in that bottom right quadrant.

            The top left and bottom right quadrants are where the return for being at least somewhat skeptical of conventional wisdom live. That’s where you’re trying to harvest insights that most people don’t have, or discard “insights” that lead you off a cliff because they’re wrong.

            The best marker I know for places where the CW is likely wrong and you should look elsewhere is that there’s an “all right thinking people know X” type model, and it describes reality really badly.

            Another way I know to look for those is to look for markers for why some kinds of discussions don’t work very well. People getting fired or fined or sent to jail for expressing the wrong opinions in some area is a useful marker. That doesn’t tell you the conventional wisdom is wrong, though–it just tells you that the normal mechanisms that would correct wrong ideas aren’t working so well in that area. Holocaust denial will get you fired/shunned/jailed, but it’s also entirely wrong and nuts.

            Topics where most people get really mad or scared when they’re debated are a marker for places where the CW might be wrong, for similar reasons. Similarly, topics where a lot of powerful people have a visible interest in supporting a particular view of the world and suppressing others, and especially where you see any indication that this power is being used, are fertile places to look for stuff that’s living in the bottom right (acceptable but wrong) or top left (unacceptable but right) quadrant of my chart.

            Finding places where people seem to hold beliefs based on popularity or identity instead of having thought things through is a somewhat useful marker, except that this describes most widespread social/political views.

            There are some topics or theories that seem to attract crazy people, or at least people caught in some crazy mode of thinking. That includes a lot of classic conspiracy theories, some cults, and unfortunately, a whole lot of political/social theories and theories about international relations. Those are hard to deal with–there could be some actual insights there, but most of the people you can read/talk to about those ideas aren’t thinking straight, so it’s hard to learn much.

          • loldrup says:

            Your thoughts are, once again, well considered. But they still maneuver in the space of ‘a priori’ assessment. It still doesn’t tell how we can find a path forward towards posterior updates for our probability estimates of conspiracy theories. It doesn’t even commit to the ideal of seeking such posterior updates at all (perhaps the policy is that we should never seek posterior updates on any hypotheses to which we have ascribed low priors? If so, then at least this policy should be given some thought, and we should figure out which implications it will have for how to conduct science in general. Ultimately this could result in a set of recommendations for universities and other science institutions, guiding them in how to conduct science in a rational way).

            I, personally, am a ‘fallen rationalist’ – I used to consider myself a rationalist, but now I’m more akin to a crook out crank. It wasn’t fun to realise that that’s exactly what I am now.

            If anybody else would like to subjugate the following uncomfortable effects:

            1. A pervasive, uncomfortable, feeling of disillusionment.
            2. A general loss of faith in humanity.
            3. A significant loss of social status (or, the option of jumping into a closet).
            4. Loss of trust in own intuitions.

            Then I can recommend you to read any of these books:

            Classified Woman-The Sibel Edmonds Story: A Memoir
            National Security and Double Government, by Michael J. Glennon
            Confessions of an Economic Hit Man: The shocking story of how America really took over the world by John Perkins
            Manufactured Crisis: The Untold Story of the Iran Nuclear Scare
            NATO’s Secret Armies: Operation GLADIO and Terrorism in Western Europe
            Legacy of Ashes: The History of the CIA
            The CIA as Organized Crime: How Illegal Operations Corrupt America and the World
            Behind the War on Terror: Western Secret Strategy and the Struggle for Iraq, by Nafeez Mosaddeq Ahmed
            Me & Lee: How I Came to Know, Love & Lose Lee Harvey Oswald (this book says nothing about how Kenedy died, but it gives a lot of context)
            Washington’s Long War on Syria, by Stephen Gowans
            Secret Affairs: Britain’s Collusion with Radical Islam, by Mark Curtis
            American War Machine: Deep Politics, the CIA Global Drug Connection, and the Road to Afghanistan, by Peter Dale Scott
            American Deep State Wall Street, by Peter Dale Scott
            The Israel Lobby and U.S. Foreign Policy
            Manufacturing Consent: The Political Economy of the Mass Media

            From the point of view of someone who is ‘in the fray’, it seems odd to look back and see my former fellow rationalists doing loads of mental gymnastics to justify *not* updating their prior assessments.
            I see independent researchers who are subjugated to intense physical harassment because they do what they do. I have made thousands of posterior updates on hundreds of conspiracy theories (sone increase in probability, others decrease), and I acquire new updates on a weekly basis. As I am doing all these updates, from loads of research, it just seems odd when people have such a hard time committing to even doing just a single update. That attitude seems akin to preferring to discuss how many of the proverbial angels can stand on the pin of a needle. Why don’t you just go out there and *do it* (as the Nike slogan suggests), instead? Buy a book by one of the former CIA personnel who have since come out to tell their story. It’s not that hard to do, nor is it a preposterous suggestion.

            My suggestion is: Don’t read to believe. Don’t read to disbelieve. Read to get an overview of that wast landscape you currently refuse to even look at. Simply knowing what specific claims or specific pieces of alleged evidence lies out there, will widen your horizon.

          • loldrup says:

            Your thoughts are, once again, well considered. But they still maneuver in the space of ‘a priori’ assessment. It still doesn’t tell how we can find a path forward towards posterior updates for our probability estimates of conspiracy theories. It doesn’t even commit to the ideal of seeking such posterior updates at all (perhaps the policy is that we should never seek posterior updates on any hypotheses to which we have ascribed low priors? If so, then at least this policy should be given some thought, and we should figure out which implications it will have for how to conduct science in general. Ultimately this could result in a set of recommendations for universities and other science institutions, guiding them in how to conduct science in a rational way).

            I, personally, am a ‘fallen rationalist’ – I used to consider myself a rationalist, but now I’m more akin to a crook out crank. It wasn’t fun to realise that that’s exactly what I am now.

            If anybody else would like to subjugate the following uncomfortable effects:

            1. A pervasive, uncomfortable, feeling of disillusionment.
            2. A general loss of faith in humanity.
            3. A significant loss of social status (or, the option of jumping into a closet).
            4. Loss of trust in own intuitions.

            Then I can recommend you to read any of these books:

            Confessions of an Economic Hit Man, by John Perkins
            Classified Woman-The Sibel Edmonds Story: A Memoir
            National Security and Double Government, by Michael J. Glennon
            Manufactured Crisis: The Untold Story of the Iran Nuclear Scare
            NATO’s Secret Armies: Operation GLADIO and Terrorism in Western Europe
            Legacy of Ashes: The History of the CIA
            The CIA as Organized Crime: How Illegal Operations Corrupt America and the World
            Behind the War on Terror: Western Secret Strategy and the Struggle for Iraq, by Nafeez Mosaddeq Ahmed
            Me & Lee: How I Came to Know, Love & Lose Lee Harvey Oswald (this book says nothing about how Kenedy died, but it gives a lot of context)
            Washington’s Long War on Syria, by Stephen Gowans
            Secret Affairs: Britain’s Collusion with Radical Islam, by Mark Curtis
            American War Machine: Deep Politics, the CIA Global Drug Connection, and the Road to Afghanistan, by Peter Dale Scott
            American Deep State Wall Street, by Peter Dale Scott
            The Israel Lobby and U.S. Foreign Policy
            Manufacturing Consent: The Political Economy of the Mass Media

            From the point of view of someone who is ‘in the fray’, it seems odd to look back and see my former fellow rationalists doing loads of mental gymnastics to justify *not* updating their prior assessments.
            I see independent researchers who are subjugated to intense physical harassment because they do what they do. I have made thousands of posterior updates on hundreds of conspiracy theories (sone increase in probability, others decrease), and I acquire new updates on a weekly basis. As I am doing all these updates, from loads of research, it just seems odd when people have such a hard time committing to even doing just a single update. That attitude seems akin to preferring to discuss how many of the proverbial angels can stand on the pin of a needle. Why don’t you just go out there and *do it* (as the Nike slogan suggests), instead? Buy a book by one of the former CIA personnel who have since come out to tell their story. It’s not that hard to do, nor is it a preposterous suggestion.

            My suggestion is: Don’t read to believe. Don’t read to disbelieve. Read to get an overview of that wast landscape you currently refuse to even look at. Simply knowing what specific claims or specific pieces of alleged evidence lies out there, will widen your horizon.

  3. Worley says:

    One aspect of this question is that you are presuming that the central question is what is or is not true in an objective sense. My strong suspicion is that the vast majority of people, almost all of the time, make statements whose only real content is social. That is, *every* statement that appears to be about objective reality is a stalking-horse for a statement about what other people should or should not do. As a consequence, people will make superficially factual statements that are blatantly false as code for a social-political stance — and their listeners will argue for or against the apparent facts based on which politics they advocate.

    This is fairly clear with the more political sorts of issues. E.g., (at least in the US), you can’t say “Well, policy X leads to bad consequences Y, but X is sufficiently important that we must endure Y anyway.” Instead, people say “X doesn’t cause Y”.

    Sometimes this can get extreme enough that both sides of an issue will argue about the truth/falsity of something that is either obviously true or obviously false and the debate starts to look demented. But nobody (at least, nobody normal) will suddenly wake up and say the debate is ridiculous, because everybody involved knows that the supposed facts aren’t what is being argued about.

    Where this breaks down is if one is of the minority that’s kept around to discern actual factual reality and devise better technologies. In a world with technological advancement, people like that are needed to prevent a society being technologically overtaken, dominated, and destroyed by other societies. But people like that have to talk (and probably think) in fairly abnormal ways to do their job, and they often cause trouble when they wander off the reservation.

    • albatross11 says:

      For most people, it costs nothing to have a factually-wrong view of many big-picture questions, but a lot to have a socially-wrong view. You can be a perfectly good electrician or HR manager or programmer or elementary school teacher even if you believe in young earth creationism, or think Obama is secretly a Muslim non-citizen, or are convinced that the Bildebergers or Bavarian Illuminati are running everything behind the scenes. There’s little cost to being wrong about those things, because your day-to-day life’s decisions don’t rely on them. If you live in a subculture that believes them, you probably benefit socially from going along. Similarly, if you think mercury preservatives in vaccines cause autism or global warming is a sham, it’s not really a particular issue for you personally (other than maybe your kids getting sick more often because you didn’t get them vaccinated).

      Similarly, all kinds of political theories can become widespread, and it doesn’t really make you a much worse plumber or nurse or nuclear engineer to believe that the proletariat seizing the means of production and imposing true socialism is the best path to human well-being, or that we should abolish democracy and set up a king and an established church. You don’t have the power to do any of that stuff, so who cares what you think?

      But then, we sum up our beliefs via polls, elections, markets, Twitter, etc. And it turns out to matter that lots of people have dumb counterfactual beliefs, because when summed up, we *do* act on them. Millions of people believing dumb things about politics leads to dumb things being done by politicians to chase votes; millions of people believing dumb things about vaccines leads to measles outbreaks. And so on.

  4. B_Rat says:

    Obligatory “I find this blog really interesting on the whole”.

    I swear that Scott is on a dare to piss off every history enthusiast might run into his blog. :O

    The problem with trying to learn human behavior from history is that one has to, well, get history right.
    In the present case, the post argues that the Church accidentally created honey traps for scientists, as demonstrated by these unlucky chaps. The problem is that if one actually looks at their cases, even adding Galileo and another from Unam Sanctam Catholicam, he can round up to 4 guys that could be pertinent, less than the digits in a hand. Which does not really make much of a honey trap in a pair of thousands of years of history.

    Here’s a brief review:
    – Roger Bacon: Renowed scholars like Lindberg seriously doubt that he was imprisoned (the story appeared only a century later and we know it can’t be right on some details), but even if this happened he was able to write and publish up to little before his death, so no big interference with the scientific endeavor here.
    – Lucilio Vanini: Not only he was by no means a proto-scientist, but authorities didn’t even realize whom they were condemning since by that time he had adopted a wandering life.
    – Pietro d’Abano: Apparently the details of his process and (post-mortem?) punishment are uncertain, but since I wasn’t able to find much let’s count him.
    – Giordano Bruno: Since these days whoever attempts to pass Bruno for a proto-scientist gets usually mauled by history buffs, his inclusion in this list is a mystery. In his “Ash Wednesday Supper” not only he scolds Copernicus for daring to use maths to study his beloved divine bodies (the planets), arguing that by no means they could follow regular motions, but he also presents clearly nonsensical geometric arguments. As many emphasized, he was pretty much a New-Age crackpot.

    So, as proto-scientists somehow condemned for heresy by Christianity in it’s whole history so far we have identified Servetus, d’Abano(?), Galilei and Cecco d’Ascoli. In total, 4 guys, and not for lack of searching. (All are also characterized by a plethora of personal enemies that worked hard to have them targeted, and Galileo was good until his great idea “Let’s call my great supporter the Pope a moron!”.)

    Moreover, as many other commenters observed, unlike today’s social censure the Church’s specific objective was to make crystal clear what exact opinions were not to be defended, and pretty much none of those had any practical consequence other than “you must acknowledge the Church is THE authority over religious matters”. Anyone with a bit of acumen knew how to stay out of these and study whatever he he saw fit to, so if anything this selected negatively for intelligence.

    On the other hand, it puzzles me that the example on the Russian part is the rare one who got away with his criticism (I’ve seen the suggestion that Kantorovich had to thank his importance in the nuclear program, which would be consistent which the relative freedom of opinion enjoyed by physicists), while Lysenkoism and the very crackdown on “non-Pavlovians” in psychiatry would have been far straighter examples. (As has been sensibly noted, while the Church was explicit in condemning heresy and how to recant, Stalin himself formally approved dissent)
    These also highlight the qualitative abyss between the Church’s so-called “honey traps”, totaling an insignificant amount of scientists in thousands of years with well-defined and rather stable definitions of religious heresy, and the URRS’s crackdown on entire fields of study, which in a bunch of years punished thousands of researchers over their actual scientific positions, suddenly and arbitrarily labeled as reactionaries and obliterated from the academia in favor of pseudosciences.

    Treating the two phenomenons like they’re analogous totally baffles me.

  5. ec429 says:

    Maybe if there were more heroes, their lives wouldn’t be so lonely, or so short.
    Telling people “yeah, you can’t get rid of the oppressive orthodoxy, but if you set up a whisper network you can still do some good” sounds like a great way of getting more and longer-lived oppressive orthodoxies.
    I’d rather to tell them “if the orthodoxy is really wrong, then even if it outlasts you, your speaking out now will help to destroy it in the end”.
    I find it highly improbable that the long-run supply of Soviet oppression was perfectly inelastic in Solzhenitsyns.

  6. Watchman says:

    A historical note here (segueing into something more). The Catholic church only started to allow secular authorities to punish heresy from the eleventh century, although mob violence against heretics is recorded earlier. Note the church never did the punishment, and was therefore impotent when secular authorities chose not persecute heresey – so the open development of catharism in eleventh-century southern France was because the church was unable to bring the local rulers (the king of France being unable to exercise practical jurisdiction in the region) on board with attacking the religion. So, ironically, the dark ages as understood by historians (the early medieval period, up to c. 1000) was actually a period when heresy would not get you burnt at the stake…

    There is an implication here though. The system under which you live might appear the same, but to preach heresy in ninth-century France was relatively safe and might not even effect your status (one notable heretic Gottschalk got exiled to monasteries a lot, but as he was a monk that was hardly a huge punishment; the heretical Bishop Felix of Urgell was able to hold on to his see despite the fact it was (then) clearly in the Frankish empire). Under the same church with basically the same teachings – the emergence of canon law might be a relevant difference but theologically the intervening period witnessed limited innovation – being a heretic in late-eleventh-century France was potentially life-threatening. Yet there are no major signals from the church or the secular authorities (if anything, the Carolingians in the ninth century were much more vocal about enforcing orthodoxy than their Capetian successors) that would indicate the expression of heretical views were more dangerous. And this has an implication for selecting a strategy – to select the Kolmogorov strategy in the ninth century would be to limit your influence: letters by Gottschalk and works of Felix of Urgell survive despite their heretical status because they were circulated in debate. Kantrovich’s approach was appropriate for the time – the worst outcome was to be told that the view was heretical and to stop expressing it, which most people seem to have done if this occured. In the eleventh century, Kantrovich’s approach was riskier, although as noted elsewhere in the thread there was the option of recanting, whilst Kolmogorov would have been sensible – for an actual practical application of this, see the history of the Lollards in fourteenth-century England for a movement (?) whose teachings were never quite heretical and seem to have known where the boundaries were.

    And the question this historical diversion entails is simple. How do we know which strategy to utilise? Even Soviet Communism had periods of allowing criticism as well as periods of unwavering orthodoxy, and that was an ideology that doomed its own hegemony through insistence of compliance, with the inevitable consequences of repeated compliances with the wrong choice stacking up. For something as tolerant as non-fundamentalist religion, such as medieval Christianity, then knowing the boundaries of acceptable discource are that much harder. Or, to take a modern example, how acceptable is anti-feminist thinking, which might not get you burned, but clearly can get you ostracised – how do you know whether you should express concerns with the (liberal) feminist orthodoxies or not? It is all very well to have the option of following Kolmogorov, but if you do it at the wrong time your practical utility is vastly reduced, perhaps to the point of irrelevance in a crowded field such as modern opinion forming.

    In effect, you need more information about whether stating unacceptable views is acceptable before deciding on a strategy – the existence of an orthodoxy does not necessarily imply the existence of severe penalties from varying from that orthodoxy. And that information is not always available, or easily read (so it is easier for a right-wing politician to question feminist orthodoxy than a left-wing one – but the boundaries here are not clear cut). The virtue of a strategy to deal with a constraining orthodoxy only applies in certain circumstances – and especially in the modern world, it is hard to know if those circumstances are in force at any given moment.

  7. Plucky says:

    Not to be an obnoxious jerk, but dealing with this sort problem is at least half of Leo Strauss’s thought, and the entirety of his thought on the topic of esoteric writing. He probably deserves at least a name-drop here. If this is something that bugs you and you haven’t read him, you should

  8. JohnBuridan says:

    It is worth noting that Medieval Universities were divided into two faculties, Arts Faculties and Theology Faculties and there were written constitutional rules at the University of Paris, for example, declaring that philosophers, grammarians, natural philosophers were not allowed to make pronouncements on theology, and theologians were not authorities on logical method. They had the “separate magisteria” thing written down and enshrined in custom by the year 1300.

    This doesn’t repudiate Scott’s larger point. It is to the RCC’s shame that the church hierarchy threatened imprisonment and torture upon dissenters to the “Kingdom of God” just as kings threatened punishment on their own traitors and revolutionaries. I don’t think Catholic culture on its own can get you to intellectual liberalism, and that is what a lot Catholic apologists seem to sit around wishing.

    ASAIK, the Church was more civil than Twitter and Tumblr about dissent. A late Medieval dissenter could retract his statements and delete them from the public forum and be free to go. In the modern world, what you say stays out there, causing rage, and no amount of apology will save you from the mob. At least the censors of Medieval Europe had a hierarchy with a procedure.

  9. Christopher Hazell says:

    You touch on this, but my completely historically uninformed idea of what Kantorovich was thinking is this:

    He was thinking “Communism sure is awesome.”

    Perhaps he was thinking “Massive trials, firing squads and gulags are for the enemies of communism, the capitalists and fascists who want to undermine our society and break it apart. I think communism is totally rad, everything I’ve ever said and done bears out my devotion to communism, and I just found out a way to make sure we’re doing our very best and being our most hopeful to the workers of the world.”

    Why would he imagine Stalin would kill or imprison someone like that?

    One thing that gives the Soviet Union its pervasive sense of unreality is that survival required, well, doublethink. The “savy” that allows you to understand when and how to mount small challenges to the regime while still appearing to be acceptably orthodox can only come from a lack of orthodox feeling.

    Somebody who was truly concerned, above all things, about the plight of the proletariat, and believed his bosses were also motivated that way would have no earthly reason to hide it if he thought his bosses’ policies were hurting the proletariat.

    The only reason to hide a belief like that would be if you thought your bosses were venal tyrants. Now, it might seem amazing to not notice that the communist leadership under Stalin were, in fact, monstrous tyrants, but ideology does strange things to people.

    So I think “edgelord” is not the right terminology here, because it sort of conflates two kinds of people: the folks who break social taboos consciously, and the folks who break social taboos without realizing that’s what they’re doing.

    There’s a difference between someone who challenges the Communist leadership out of a fundamental distaste for communism, and someone who challenges the Communist leadership out of loyalty to communism.

    And one thing that makes those totalitarian orthodoxies so destructive is that they don’t want to admit that difference.

  10. srconstantin says:

    Scott, you’re a major influencer. People read your blog. I think it’s harmful to spread the message that “Kolmogorov complicity” is the right way to go.

    I don’t think it was right in Soviet Russia — Kolmogorov betrayed his teacher to the Great Purge — and it’s certainly not true in the democratic West, where speaking out can make you lose your job but not your life.

    We do not need a whisper network. We need more people to eat a little social cost. We need to egg each other on to courage. We need to say “What are you, chicken?”

    When the other Scott A. wrote about the Kolmogorov Option, I wrote to him privately, saying, essentially, “What are you, chicken?”

    Then he invited me to write a guest blog post, and I did. I got mostly positive responses and no internet harassment. I think this is repeatable, and I am happy to keep doing it, in any cases where I am confident that the prevailing conventional wisdom is wrong and harmful.

    • The Nybbler says:

      We do not need a whisper network. We need more people to eat a little social cost. We need to egg each other on to courage. We need to say “What are you, chicken?”

      What good does it do? Granted, I’m not sure the Kolmogrov Option is any good EITHER; personally I don’t have the temperament for it. If I thought I could keep quiet long enough to get into a position of power and really do some good, maybe, but the fact is I will never be in a position of power; my options when faced with some of this stuff are speaking out and getting struck down, or keeping silent and hating myself. I’ve taken both, what they have in common is they do no good.

      • srconstantin says:

        You don’t want to be a strict act-consequentialist! That’ll lose, predictably, in iterated games. Don’t respond to threats because you don’t want you and those like yourself to be the kind of people who respond to threats.

        • The Nybbler says:

          You didn’t answer my question: What good will it do? Suppose I act and egg the others on; perhaps I start writing an openly and blatantly heretical journal, and when nonsensical orthodox beliefs are championed I bring up inconvenient evidence which undermines them. And in the fullness of time I’m fired for doing so, and anyone following my example is also, and victory for the orthodoxy. The only difference with the Kolmogrov option is that I and those that would have followed me remain employed and unpersecuted; obviously it’s still victory for the orthodoxy.

          In fact, I’ve had it argued to me that it’s inherently selfish and egotistical to challenged the orthodoxy that way, that a moral person would show proper humility and bow his head as required and thus spare the system the pain of punishing him, his friends and family the pain of seeing him punished (and the material consequences of his doing so), and his followers the pain of being punished for following him into error.

  11. Deiseach says:

    Given that I have been stamping my foot fairly hard in the comments here, I do want to convey my appreciation and gratitude to Scott for letting us debate this out, even to the point of disagreement with him. It is in very marked contrast to responses elsewhere when someone’s dicta on a subject are challenged.

  12. albatross11 says:

    The discussion here made me think of this short essay.

  13. Claw21 says:

    Once upon a time, there was a very vain and foolish emperor who thought he was enlightened and wise would only wear the most elegent and refined clothing.

    One day, two con artists set their sights on a big prize, the Emperor. They went to the Emperor and showed him their finest cloths, satins, silks, and gold embroidered fabrics. But the emperor was not impressed by such things. Had he not an entire wardrobe like that already? “Have you nothing else?” asked the Emperor.

    The two con artists smiled. “Oh, Glorious Emperor, we do have one final cloth that is more exquisite than any other. We feared to show it to Your Majesty because it has a most wonderous and peculiar feature. This cloth is so fine and delicate that only the wise and enlightened can see or feel it.”

    “Do you think me a fool?” asked the Emperor. For a moment the two con artists thought they had been caught. “Of course I will be able to see it. Bring me this cloth.” The con artists smiled in relief.

    So the two con artists opened their final case, which contained nothing at all, and began to pantomime showing off their beautiful wares. “See the fine embroidery,” said one. “Come feel the smooth texture,” said the other.

    “Oh, how splendid!” said the Emperor, running his hand over the place where the cloth was supposed to be. Of course, could not see or feel the cloth, but in embarrasment, he pretended that he could. And actually, wasn’t that a gleam of light he saw from the corner of his eye? If he looked ever so carefully, he could convince himself that he was seeing the most fabulous silk cloth that he had ever seen.

    “I want new robes made of this cloth immediately,” said the Emperor.

    So the two con artists and the Emperor agreed upon a price and they immediately got to work. They lived in extravagent rooms in the imperial palace and whenever anyone came to check on them, they pretended to be hard at work at their sewing. Any servant who dared to say that the two con artists were working on nothing at all became the laughing stock of the palace and was punished for his foolishness.

    Finally, the day arrived that the Emperor demanded his new set of robes and the two con artists “displayed” all their works. The Emperor clapped in delight. He had convinced himself that he could see the robes in the swirling dust in the air.

    “Come, dress me in these robes,” said the Emperor. “I will wear them to court this day.”

    So, the two con artists “dressed” him in his new, fabulous robes and the Emperor mounted his favorite horse, and he paraded through the city to the building where he held court.

    As was usual, a crowd gathered to applaud and cheer their emperor. They had heard that he was to wear some new garments that were so exquisite that only the enlightened could see them, so more people than usual gathered to see for them selves this new cloth.

    Each individual in the crowd was shocked to see the Emperor riding naked to court, but in embarassment to discover their own foolishness, they clapped and cheered even louder than usual.

    The Emperor was a bit cold in his new robes, but was very pleased that he was ruler of such an enlightened people. He waved happily back to the crowd. Then he heard a lone, small voice. It was a child in uproarious laughter. The Emperor, embarrassed by this laughter, sent his guards to discover its source.

    “Why are you laughing?” asked the guard when he found the boy. “Don’t you know to show respect for your Emperor?”

    “The Emperor has no clothes!” shouted the boy, laughing still.

    Hearing this, the Emperor realized he truly had been a fool ordered the boy imprisoned for his insolence and then he proceeded to court.

    At court, all of his advisors and staff complimented him profusely on his clothing and all proceded as normal.

    When he returned to his palace, he ordered the two con artists to make garments for his servants and advisors from the same fabulous cloth. And of course, the two con artists agreed once a wealth of gold in payment was agreed upon.

    In time, clothing made from this “fabric” became very fashionable. Everyone who wore it complimented each other on their good taste and mocked the “fools” who refused to wear it. Soon, all the surgeons and the teachers all had to wear uniforms of this “cloth” or they would not be able to find work.

    Children were made to wear such uniforms as well. If they refused, they were thrown out of school because it was impossible to educate such an intractable fool.

    This clothing spread from the capital city throughout the entire empire and the two con artists became very rich. There were others who began to learn the art of weaving this “cloth” and soon, there was no one left alive who knew that it had all begun as a con.

    Soon, there was no one left who dared to say, “The Emperor has no clothes,” for fear of being locked out of every profession and priviledge. And the people who loved the truth, and valued being warm outside, suffered. All because of the foolishness of the Emperor whom everyone knows is wearing no clothes, but no one is allowed to say.

    • MostlyCredibleHulk says:

      I wonder on which stage the con artists realized that they’ll have either leave (which would be inevitably questioned and there would be a risk that all con blows up and they’d be beheaded) or spend the whole rest of their lives prancing around naked and tolerating the horrible sights of esteemed (and very very elderly) court nobles parading around naked and praising them with new and inventive, never repeating, praise every day – and they can do absolutely nothing about it, that’s their life now and forever, until the day they die? And when they asked themselves whether it was worth it – what was their answer?

      • Conrad Honcho says:

        And when they asked themselves whether it was worth it – what was their answer?

        “Woohoo, we’re rich!!!!” They were con artists. They were never interested in anything other than easy money.

  14. Nate the Albatross says:

    I feel like both approaches are necessary for science.

    As the oldest of four brothers, there was obviously a whisper network built around Santa Claus. As older brothers we made sure our younger brothers didn’t openly question the orthodoxy around the presents, and especially that they wouldn’t poison the well with younger siblings. But once the youngest was old enough to question it, we let him openly test the orthodox position. The whisper network never inducted him into our ranks and let his curious mind say what we were all thinking.

    Kolmogorov’s whisper network is great in order to keep hope alive and keep scientists alive. Mentorship, private conversations between young people and people whose loyalty can’t easily be questioned, and making sure not recruit any Kantoroviches who might through lack of savvy or idealism get everyone killed. But the world needs test cases too. There will inevitably be people who either aren’t savvy enough to join the whisper network, or are super smart and just don’t want too. Galileo was a martyr for his cause, and while every child in school learns about Gailieo, nobody studies the writings of any of his opponents. Having once been 20 years old myself, I sympathize with Kantorovich. Ten millions of people experienced enormous hardship because idiots didn’t see things Kantorovich’s way. Perhaps he was a young idealistic fool who thought Stalin wanted advice. Or maybe he was smart enough to see the graves of millions of people and would have rather died than do nothing.

    If Russia took over Europe right now, as a man with children I wouldn’t write an economic paper suggesting the invasion of Ukraine was the worst economic blunder this century because Putin would most assuredly torture and kill my kids. I’d create a whisper network. But if some young idealistic people with more courage than brains published it, I’d be sure my whisper network quietly cataloged it, and we’d be sure to build a memorial to them later. But there are limits. Sometimes it is better to speak up and die than to live with complicity.

    • MostlyCredibleHulk says:

      Galileo was a martyr for his cause, and while every child in school learns about Gailieo, nobody studies the writings of any of his opponents

      That was not because Galileo was a martyr – which was arguably not “for his cause”, if by “his cause” we understand not playing contemporary politics and personal squabbles, which we couldn’t care less about, but anything we do care about – there were people that suffered much worse and we have no idea who they are. If Galileo’s opponents did work as important for as he did, we would be right to study their works too, and we probably would. We have a number of examples of disagreements between famous people where we know both sides.

      Or maybe he was smart enough to see the graves of millions of people and would have rather died than do nothing.

      Or, maybe, he thought some traitors and saboteurs, or possibly foreign spies – who then were found on each corner every day – deceived Stalin into wrong actions, and saw himself as heroic young man that discovers the enemy network and causes its demise by informing the security forces. This is a very popular scenario in these times in USSR, see Pavlik Morozov and such. Of course, Morozov died, but at the hands of the enemy and not the security forces. Kantorovich had reasons to believe that won’t happen to him.

      • B_Rat says:

        If Galileo’s opponents did work as important for as he did, we would be right to study their works too, and we probably would.

        Except that this is exactly the wall that historians furiously bang their heads against: there were plenty of Galileo opponents doing important if not better works and he is only famous for his ability as a writer and the infamous “Ops, I insulted my great benefactor the Pope” process.

  15. Jameson Quinn says:

    Let me suggest a modified parable of the lightning. Suppose that the orthodoxy were “thunder actually doesn’t come before lightning, but there are some very nice and smart people who believe that it does; on the other hand, anybody who thinks you can calculate the distance to a thunderstorm by counting seconds and dividing by the speed of sound is a horrible person, because in actuality that calculation isn’t right without correcting for the speed of light.” In other words, the orthodox position isn’t actually empirically wrong, it’s just massively asymmetric about how it deals with empirical errors in various directions.

    And suppose further that there is actually some justification for that asymmetry. It’s pretty obvious that Damore is one of the kind of heretics whom we’re actually talking about here, so let’s take that case. The anti-Damore people would say that his memo was actually harmful to women at Google and thus to Google itself. I think that’s actually true to some extent. Certainly, if it’s true, it provides some justification for being asymmetrical about looking for errors in what he says.

    Note that probably most of what the memo says is neither objectively true nor objectively false, because though it’s not unrelated to directly-testable empirical claims, it’s far enough away from them to provide wiggle room in either direction. In that context, asymmetry about empirical deviation is going to end up resembling a false orthodoxy.

    So one side is going to believe “Damore said things which are false” which, when pressed, actually means “Damore said things which are not quite specific enough to be false, but which would be false if a good-faith effort were made to translate them into specific claims in a way that was unbiased by any pre-existing assumption that Damore is not an idiot.” The other side is going to believe “Damore was persecuted for saying things which are true” which, when pressed, actually means “Damore was persecuted for saying things which are not wrong, including the fact that the orthodox beliefs are false” where “false” actually means the same thing it did in the previous sentence. Both sides are “true” if interpreted with enough benefit of the doubt, and “false” if interpreted uncharitably enough.

    Nobody here is actually being burned at any stakes or sent to work to death in Siberia. Damore lost a nice job and faced some vicious vitriol, and those are real suffering, but not really comparable to the suffering of Spanish heretics or Soviet dissidents. Given that, I think that there’s actually a healthy debate to be had about how asymmetrical society should be in punishing deviations from the truth. Scott’s position (that is, with only a modicum of reading between the lines, “Damore shouldn’t have been fired”) is certainly a reasonable side in that debate; but the other side, that the firing was justified, is also reasonable.

    • zkbxbava says:

      Even if every single word Damore wrote was 100% false, he should not have been fired. Firing people because they say something false is completely fucked.

      • cuke says:

        I like the simplicity of this and I agree. Protecting people from being fired for saying something false would require employment contracts or other legal protections. I favor those as well.

        • Evan Þ says:

          We can still say he ethically should not have been fired without favoring legal protections.

          • albatross11 says:

            I agree. Whatever legal protections exist, we would benefit from a strong social norm that says that, at the very least, your off-work-hours political, social, religious, sexual, etc., behavior is none of your employer’s business. That firing people for marching, campaigning, voting, supporting, etc., the wrong side in a political or social debate *shouldn’t* get you fired, even if the boss finds your opinions objectionable.

            We’re quickly moving into a world where it’s *hard* to keep our outside-work and inside-work lives so separate that it’s possible to, say, spend your weekends going to impeach Trump rallies or protesting abortion clinics, without this coming to the attention of the boss. So either we develop some norm like this, we get some kind of legal protection that has the same effect, or we accept that everyone who’s not independently wealthy loses a huge amount of practical freedom of expression and ability to be involved in political and social issues.

            It seems to me that most of the very visible cases right now involve a liberal/SJW orthodoxy and conservative employees getting hammered, but that’s probably not how things will work out long-term. For every potential Damore who’s convinced to keep his political and social views secret to keep his job, there are going to be a hundred Wal-Mart employees who discover they don’t dare go to the pro-choice march, for fear of losing their jobs. The difference is, those people aren’t usually very articulate or influential, and so nobody ever hears about them.

    • albatross11 says:

      Jameson:

      The principle you’re proposing, if I understand correctly, is something like this:

      Suppose Alice makes a statement which may have an error in one of two directions. If we believe that people erring one of those two directions would have bad social effects, and the other would not, then we should be willing to punish errors in the bad-social-effects direction harshly, while pardoning errors in the other direction.

      A common variant of this situation is that Bob makes a statement which is plausible, but for which there is not enough evidence or knowledge to say with certainty it is true or false. I infer that your principle here (applicable to the Dalmore case) would be that if Bob’s making plausible but unproven statements which would cause bad social effects if believed, we should be willing to punish him harshly, whereas plausible but unproven statements in the other direction should be pardoned.

      Am I understanding your principle correctly?

      It strikes me that this would have made an excellent argument for suppressing the theory of evolution. Darwin’s original construction of it had some problems (he needed someone to tell him about genes, and the age of the world needed was longer than what a lot of scientists at the time believed), so it was at least unclear whether he was right or not. But the social consequences of undermining the religious faith of the public with these arguments, and offending millions upon millions of people who would find the theory an affront to their deepest beliefs–those were huge and obvious.

      If your principle leads us to jail Darwin for blasphemy and undermining public order, I think we maybe need to think it through a little harder before accepting it.

      • vV_Vv says:

        It strikes me that this would have made an excellent argument for suppressing the theory of evolution.

        And heliocentrism too. As noted in other comments, heliocentrism predicts stellar parallax and the Coriolis effect, which are difficult to measure (the Coriolis effect had not yet been predicted at the time of Galileo trials, possibly stellar parallax had). Galileo also made various weak, or outright wrong arguments.

        Given that Galileo’s contemporaries could reasonably doubt that he was factually correct, and since his theories had the potential of undermining public order and offending many people, @Jameson Quinn’s argument could be used to justify the trial outcome.

    • The Nybbler says:

      …therefore it’s OK to persecute heretics, provided we don’t actually torture them or send the to Siberia, because they’re rarely 100% right.

      Your apologia here is doing exactly what your parable does; you’ve got two positions, one of which is mostly wrong and the other which is mostly right, and claiming they’re somehow equivalent because neither is absolutely correct or absolutely wrong. It’s the Fallacy of the Grey, to believe that because #010101 and #FEFEFE aren’t black and white that there isn’t still a clear distinction between the two.

    • albatross11 says:

      I don’t mean to pick on Jameson Quinn, but he wrote an interesting comment, and I wanted to unpack a bit of it. The discussion here is mainly about the Damore memo:

      Given that, I think that there’s actually a healthy debate to be had about how asymmetrical society should be in punishing deviations from the truth.

      One striking thing about this is that the “deviations from the truth” we’re talking about aren’t something like young-Earth creationism or vaccines-cause-autism, where there’s really strong evidence in one direction. The orthodoxy against which Damore was arguing isn’t some kind of rock-solid settled science, it’s a mix of moral and empirical claims, where many of the empirical claims are at least questionable. In general, the social sciences rarely give us rock solid explanations for observed phenomena, they give us approximate explanations that maybe have some experimental or observational data behind them. That makes it a little hard to see something like firing Damore as punishing deviations from truth, and a lot easier to see it as punishing deviations from the party line or from some accepted-by-the-powerful dogma. Now, maybe that dogma has some positive social effects, but that seems like an empirical question, and one that it wouldn’t be safe for a Google employee to ask in public.

      And that leads to a second issue: How do we determine the truth on contentious issues? Obviously, the internet will be full of stupid and crazy people arguing for whatever their personal bit of nutjobbery is on any issue. And yet, the best way I can see to get to the truth is to let people discuss questions without threat of being punished if they say something too offensive to current sensibilities. Why are there so few women at the top of software development? I don’t know, and I doubt Damore does, either. But one way to make sure we *don’t* make much progress on finding out an answer is to make it unwise for most people to participate in the discussion at all, other than by mouthing whatever the most powerful people in the room are saying.

      This strikes me as the biggest problem with “punishing error.” There are probably a lot of people working in software right now who have valuable insights on the gender imbalance in technology, and a large fraction of them know that it’s as much as their job is worth to share their insights or describe their experiences. This is an environment in which we won’t be hearing from many of them.

      Now, if the people with power in this situation are entirely right and wouldn’t benefit from those insights, then there’s no problem. But if they’re wrong–if they’re pursuing policies that won’t actually work, or that will impose a lot of collateral damage they don’t understand, they won’t be able to find that out for a long time, because they’ve made it unsafe to complain about those policies, or to share those insights.

      Our mechanisms for figuring out the truth work badly in the best of times. Punishing error throws sand in the gears of that machinery.

    • Steve Sailer says:

      One irony in the Damore tale is that the winning voice in the struggle within Google to make an object lesson out of Damore’s fate was Youtube CEO Susan Wojcicki, who got to where she was by being Larry and Sergey’s landlady in the garage days and then by introducing Larry to his future wife (and ex-wife) Anne Wojcicki. Anne is famous as a founder of 23andMe, which, ironically, is in the genetic testing business about what your chromosomes determine about your fate:

      http://takimag.com/article/a_tale_of_two_sisters_steve_sailer/print#axzz4wlrM5yr9

  16. BBA says:

    For me, the most compelling stories of scientists killed for political reasons are Archimedes and his circles (apocryphal) and Lavoisier being guillotined (true, though “The Republic has no need for chemists” is apocryphal). No amount of political astuteness would have saved them. So, I assume, is the case today – if “they” are out to get you, there’s nothing you can do to stop “them.” (I don’t actually know who “they” are, but work with me here.)

  17. cvxxcvcxbxvcbx says:

    If you read this and you didn’t think of something in our society that the lightning problem is a euphemism for, you’re too unobservant or conformist for Kolmogorov or Kantorovich to be an Option.

    Scott, thank you for another great article.

  18. quaelegit says:

    Has Scott or anyone else tried to check if “scientists” were actually in MORE danger of being burned at the stake/declared heretics than other people (this seems like it would be very hard to do rigrously and control for, but would checking an arbitrary group of biographies help with calibration?).

    Because a LOT of people got targeted for ostensibly religious reasons during the reformation/countereformation. There’s all these religious wars (Hussite Wars, French Wars of Religion, 30 years war, just off the top of my head), and witch burning had resurgences in the 14th, 15th, and 16th century (hey, that covers time periods of most of Scott’s examples).

    Basically, Early Modern Europe was a shitty and dangerous time to be alive (probably worse than the High Middle Ages, depending on when and where you were). Yes, when society is facing major upheavals, that affects thinkers too! No need to think “science” is being targeted specifically.

    [To be fair, maybe “scientist” were targeted more than, say, non-“scientist” courtiers, nobles, and businessmen — I haven’t tried my suggested test yet, but will attempt it after reading the comments. I just want to point out that Scott seems to be assuming they were more targeted, but I don’t see where he checks that.]

    I’m sorry if someone has already made/or addressed this point in the comments, this annoyed me enough that I wanted to write it up before reading all 350 comments to see if it had.

    • Le Maistre Chat says:

      Basically, Early Modern Europe was a shitty and dangerous time to be alive (probably worse than the High Middle Ages, depending on when and where you were).

      Joseph de Maistre wrote a Defense of the Spanish Inquisition around the thesis that suppressing heresy leads to an orderly and happier society. “Spain had the Inquisition, Germany had the Thirty Years War”, he says.
      He would have agreed that Early Modern Europe was “a shitty and dangerous time to live” compared to the High Middle Ages, and that Whigs deserved to have their politicized historiography refuted in the edgiest possible way. Interesting to note that in other works, the violently orthodox Maistre pretty much accepts Voltaire’s historiography of four golden spots* in a sea of human darkness.

      *Athens 480-323 BC, Rome’s generations leading up to Augustus, “the Age of Leo X” as the Italian High Renaissance was termed, and age of Louis XIV.

    • jhertzlinger says:

      The rest of Eurasia was authoritarian at the same time as Early Modern Europe. It may have been due to bubonic plague, the Little Ice Age, the government monopoly on guns, or the effects of the Mongol Empire.

    • Christopher Hazell says:

      I think you are slightly misinterpreting the main post:

      It’s not so much that scientists were targeted by heretic burners, in the sense of the heretic burners specifically looking to fuck with scientists, but rather that scientists tend to have the mix of curiosity and lack of social acumen that makes a person more likely to come to the attention of those who want to root out heresy.

      And actually, reading the comments above, I would add that perhaps the temperament which makes a person want to be a scientist is also the temperament that gives them a pressing psychological need to express their ideas and have those taken seriously (albeit not necessarily shared) by others.

      Or, to put it another way, that the kind of temperament that says, “This seems obviously true to me, but I’ll get in trouble for saying it out loud, so I’ll just go ahead and lie” is particularly unlikely to be found amongst scientists.

      • The original Mr. X says:

        It’s not so much that scientists were targeted by heretic burners, in the sense of the heretic burners specifically looking to fuck with scientists, but rather that scientists tend to have the mix of curiosity and lack of social acumen that makes a person more likely to come to the attention of those who want to root out heresy.

        As someone else pointed out on this thread, you really needed to try pretty hard to get executed for heresy in the middle ages. The Inquisition didn’t actually want to kill people, they wanted them to repent and to stop spreading heresy, and medieval Catholicism had plenty of creeds and official documents helpfully spelling out exactly what counted as orthodoxy. If somebody doesn’t understand “Sign this document saying that Christ is the Son of God, and you can go free; don’t sign it, and you’ll be executed,” that’s not a sign of poor social skills or difficulty perceiving unwritten rules, it’s a sign of serious mental disability.

        • Christopher Hazell says:

          If somebody doesn’t understand “Sign this document saying that Christ is the Son of God, and you can go free; don’t sign it, and you’ll be executed,” that’s not a sign of poor social skills or difficulty perceiving unwritten rules, it’s a sign of serious mental disability.

          At that point it’s not about a lack of understanding, it’s more about the second thing I said:

          Or, to put it another way, that the kind of temperament that says, “This seems obviously true to me, but I’ll get in trouble for saying it out loud, so I’ll just go ahead and lie” is particularly unlikely to be found amongst scientists.

          Like… even in the ideal case where you have an actual heretic, who is not being given a Joan of Arc style show trial (and Joan’s case shows that at least occasionally you did get show trials whose verdicts were political and which killed basically orthodox people) but is actually being allowed to repent, that’s still a monstrous affront to them. Certain people have a need to do things which they see as morally important (e.g. worship god according to their own conscience, work towards a society that values the truth) even if it causes them material harm, and I am pretty much wholly on the side that says they shouldn’t be lit on fire for doing so.

          It is very worrisome to present any action other than those aimed at survival as a form of insanity.

          Like, you really don’t get why people would find it intolerable to be forced to constantly lie?

          Again, I am going to say that it is at least plausible that a personality that is willing to die rather than be forced to lie over and over again is over-represented among scientists compared with the general population.

        • JohnBuridan says:

          @MR. X

          I like both your posts and was thinking along the same lines.

          If as a public school teacher I go in to class and start teaching about the Truth of Quarkerism, I get in trouble. It doesn’t matter how deeply I am convinced of Quaker religion. Likewise, as far as I can tell, you only get kicked out of the medieval university if you are teaching ideas outside your subject area, or your conclusions directly contradict a defined Catholic position.

          I just don’t see how that is a “trap” for the intellectually curious.

          Admittedly, I know less about the counter-reformation than about the Middle Ages. The Reformation threw the Catholic church into a deeply conservative, Inquisitorial mindset which I guess it (the Church) only started to emerge from in the 19th Century.

          I am not saying the RCC lends itself to creating a culture of total free inquiry. But compared to a lot of other cultures, it does a decent job of promoting education, and defining the limits of inquiry.

        • The original Mr. X says:

          @ Christopher Hazell:

          Like… even in the ideal case where you have an actual heretic, who is not being given a Joan of Arc style show trial (and Joan’s case shows that at least occasionally you did get show trials whose verdicts were political and which killed basically orthodox people)

          Let’s not get carried away here. The vast majority of heresy trials were nothing like Joan’s, and you can find miscarriages of justice in any country and period.

          It is very worrisome to present any action other than those aimed at survival as a form of insanity.
          Like, you really don’t get why people would find it intolerable to be forced to constantly lie?

          I didn’t say anything about “any action[s] other than those aimed at survival”, I was talking about being unable to understand somebody who explicitly tells you “Do X, and you’ll survive; do Y, and you won’t”. Nice straw man, though.

          Again, I am going to say that it is at least plausible that a personality that is willing to die rather than be forced to lie over and over again is over-represented among scientists compared with the general population.

          It wasn’t a case of “lie over and over again”, it was a case of “stop spouting off about things outside your subject area”. Maybe some scientists would be unwilling to do even that, but if the amount of careerism, politicking and brown-nosing to funding bodies that goes on in modern science departments is any guide, the vast majority of scientists aren’t quite as single-mindedly focused on searching for truth as your argument seems to suggest.

          @ John Buridan:

          Admittedly, I know less about the counter-reformation than about the Middle Ages. The Reformation threw the Catholic church into a deeply conservative, Inquisitorial mindset which I guess it (the Church) only started to emerge from in the 19th Century.

          That’s certainly the popular view, although I’m not sure how true it is and how much it’s just the result of Protestant anti-Papism.

          • Christopher Hazell says:

            I didn’t say anything about “any action[s] other than those aimed at survival”, I was talking about being unable to understand somebody who explicitly tells you “Do X, and you’ll survive; do Y, and you won’t”. Nice straw man, though.

            Well, you said:

            If somebody doesn’t understand “Sign this document saying that Christ is the Son of God, and you can go free; don’t sign it, and you’ll be executed,” that’s not a sign of poor social skills or difficulty perceiving unwritten rules, it’s a sign of serious mental disability.

            I believe that there are reasons other than “serious mental disability” why one might refuse to sign such a document; perhaps you do too.

            What I’m kind of losing in your argument is why you think these people were burnt? Like, to a certain extent you seem to be saying that it was their choice, that the inquisition was so averse to killing people that you had to go very far out of your way to get the death penalty.

            (And perhaps that someone who is incapable of avoiding the inquisition is showing such bizarre behavior that they couldn’t make a good scientist? Not sure.)

            Why do you think these particular people went out of their way to receive it?

            And, as a follow up, what would the consequences be if the temperment that leads one to cross the inquisition is more likely to be found in certain professions than in others?

          • The original Mr. X says:

            Why don’t you try looking at the bit of your post I originally quoted:

            It’s not so much that scientists were targeted by heretic burners, in the sense of the heretic burners specifically looking to fuck with scientists, but rather that scientists tend to have the mix of curiosity and lack of social acumen that makes a person more likely to come to the attention of those who want to root out heresy.

            I wasn’t saying that anybody who crossed the Inquisition had to be mentally disabled; I was saying that “Innocent scientist gets burnt at the stake because he didn’t realise you weren’t supposed to say X” would have been a highly implausible scenario. Of course, some people would have knowingly crossed the Inquisition and got in trouble for it, but that’s not a case of poor social skills.

            Why do you think these particular people went out of their way to receive it?

            For the same reason that people nowadays go out of their way to publish wacky screeds about why the Holocaust never happened or why 9/11 was an inside job. (If you think that’s an uncharitable comparison, I suggest you look at the beliefs that got, say, Giordano Bruno condemned.)

            And, as a follow up, what would the consequences be if the temperment that leads one to cross the inquisition is more likely to be found in certain professions than in others?

            That depends on how much more likely that temperament is. Also, do note that scientists have all sorts of limits on their speech nowadays: a scientist who, say, suggested that the historical reality of the Holocaust is by no means certain, or that society would be better with widespread chattel slavery, would almost certainly get fired, and yet scientific research still gets done.

          • Well, you said:

            “If somebody doesn’t understand “Sign this document saying that Christ is the Son of God, and you can go free; don’t sign it, and you’ll be executed,” that’s not a sign of poor social skills or difficulty perceiving unwritten rules, it’s a sign of serious mental disability.”

            I believe that there are reasons other than “serious mental disability” why one might refuse to sign such a document; perhaps you do too.

            As you can see by what you just quoted, he didn’t say “refuse to sign” he said “doesn’t understand.”

            Do you disagree with what he said, or only with what you think he meant?

      • The Nybbler says:

        Or, to put it another way, that the kind of temperament that says, “This seems obviously true to me, but I’ll get in trouble for saying it out loud, so I’ll just go ahead and lie” is particularly unlikely to be found amongst scientists.

        Or perhaps it is, but you never hear of them. Imagine a world where a bunch of 10th century astronomers were using heliocentric calculations but never saying that’s what they were doing… who would know?

      • Steve Sailer says:

        “And actually, reading the comments above, I would add that perhaps the temperament which makes a person want to be a scientist is also the temperament that gives them a pressing psychological need to express their ideas and have those taken seriously (albeit not necessarily shared) by others.”

        A friend of mine is the son of a Soviet scientific giant who did the regime an immense service. As far as I can tell, his father was basically a conservative loyalist by personality who was proud to serve his country to the best of his ability: in other words his dad was like the average person, except he was a genius scientist. (As far as I can tell, he didn’t even get sent to the Gulag once, and he was awarded lots of Stalin and Lenin Prizes, which he was proud to receive.)

        In contrast, dissidents with complex views like my friend’s dad’s colleague Sakharov are rare in the general population, but more common among the best scientists. You could say to Sakharov: Why not just be like my friend’s dad and have complex views about science but simple views about politics?

        But often it doesn’t work that way. People who have complex views about science are more likely to have complex views about politics.

        • albatross11 says:

          Smarter people will have more elaborate theories/ideas about politics, but it sure seems like a lot of very smart people I know have pretty vanilla politics–a smarter version of standard Democratic or Republican ideas. In some fundamental sense, we’re herd animals, and we tend to follow the intellectual herd unless we make some special effort to think through the default assumptions of our society and upbringing.

          • loldrup says:

            And basically, what rationality tells us is: don’t do that individual research into political topics, because the chance that you’ll end up being wrong/bad is pretty high (because the prior of you coming up with a novel but true/good theory is pretty slim)

  19. Whitney g says:

    “My guess: the Soviet government didn’t officially say “We will kill anyone who criticizes us”. They officially said “Comrade Stalin loves freedom and welcomes criticism from his fellow citizens”, and you had to have some basic level of cynicism and social competence to figure out that wasn’t true.”

    You might be interested in reading The Gulag Archipelago by Aleksandr Solzhenitsyn

  20. 6jfvkd8lu7cc says:

    In the interplay of the object level and the parable level, it is interesting that Kolmogorov’s influence on institutions of teaching those eager and capable to learn — in terms of what to teach and how to teach— survived not only Kolmogorov himself, but also survived the Soviet Union.

    (His attempts to write textbooks for all schools didn’t go as well — there were some reasons related to internal politics of Academy of Sciences, but the textbooks were also too hard).

  21. bapgwh says:

    I can think of a better example of a system that honeytraps Scientists than both the Catholic Church and the Soviet Union/Communist Bloc. Islam, especially after say 900, 1000,definitelty 1100. A formerly relatively productive area of the world almost totally ceased to contribute scientific advancements, certainly among Muslims, and that has continued to this day with no Muslims winning a Nobel Prize in one of the three scientific fields in its 100 year plus history. I think it might even be true of any Muslim majority country period, or At least close to it. The phenomenom is especially pronounxed for Sunnis, less so for minority Shia. They even have a name for this phenomenon, closing the gates of Iljtihad. https://en.m.wikipedia.org/wiki/Ijtihad

    • cvxxcvcxbxvcbx says:

      Actually, there have been three, unless I misunderstand you:
      https://en.wikipedia.org/wiki/List_of_Muslim_Nobel_laureates#Sciences

      Still a small number.

      • The Nybbler says:

        Sancar and Zewail both did their work in the US. And Salam’s version of Islam was declared non-Muslim by his government (Pakistan) and he left the country as a result. Which makes the original point quite well.

    • They even have a name for this phenomenon, closing the gates of Iljtihad.

      Whether the gates were ever closed is a highly disputable matter–the majority opinion among modern scholars seems to be that they were not. In any case, Ijtihad is the process of interpreting religious sources in order to deduce religious law, not in order to determine scientific truth.

    • rlms says:

      Regarding your chronology: Averroes died in 1198. There were also a lot of people like this guy doing impressive things centuries after 1100.

  22. yoshi says:

    > Did Giordano Bruno die for his astronomical discoveries or his atheism? False dichotomy: you can’t have a mind that questions the stars but never thinks to question the Bible. The best you can do is have a Bruno who questions both, but is savvy enough to know which questions he can get away with saying out loud. And the real Bruno wasn’t that savvy.

    Thing is, you had to work quite a bit to get burned at the stake and even if Bruno wasn’t savvy enough to realize at first that he was in trouble, a highly dubious assumption since he was a theologian, he would have realized that after a few rounds of torture, or when there is a formal tribunal, or when he sees the pyre atop of which he will burn shortly. In all of these instances he did not recant and instead assumed that his best course of action is enduring torture and execution. I think that shows quite clearly, that he was not motivated by anything resembling a modern understanding of scientific truth, a modern scientific realist does not need to endure torture, he can recan’t without changing the truth of his position.

    The same thing applies to Stalin, Stalin did not deny using force in service of the USSR. His entire claim to legitimacy was that he took up arms for the revolution at a time when facing a firing squad was a much likelier outcome than commanding one. He was at least at one time convinced, that using force is the right thing to do.

    And this is, where the example of thunder before lightning falls flat, a orthodoxy needs something that makes it work in the first place, because the first proponents have to be convinced themselves by something and you will not understand their actions, if you are trying to understand it in a modern liberal context. Bruno made his decisions in a medival context, Stalin worked in the context of a communist revolutionary. And Kantorovich worked in the context of being a highly successful Soviet academic. (Btw, is there a source that historians are surprised that he did not get into trouble? All I found googling were biographies that suggest a very straightforward carer, including winning the Stalin and Lenin medals as well as a Nobel in economics for his work on planning.)

    • Machine Interface says:

      It’s worse than that: not only Bruno was not tortured (instructions were specifically given in that direction, is it was estimated that the case was delicate and sensitive), but he *pretended* to recant, multiple times, before getting back to preaching his heterodoxy as soon as he was out of the inquisitors’ sight. He was absolutely not naive about what could happen to him — he just thought the inquisitors were idiots who would never see past his constant game of switching gears depending on his audience.

    • dansimonicouldbewrong says:

      This is an important point. The vast majority of “truths”–even those held by scientists–have a large dose of pragmatism to them. We don’t actually know any scientific “laws” to be “truth”, after all–and every now and then one of them is proven to be quite incorrect–but we have lots of pragmatic reasons for believing those laws. (They help us design technologies that function well, for example.)

      Now, sometimes there are pragmatic reasons for believing things that aren’t scientific laws, or that aren’t scientific statements at all, or that are contradicted by enough empirical evidence to cast doubt on their scientific “truth”. Sometimes those pragmatic reasons are highly idealistic (“believing this helps me be a kinder, gentler, more generous person”), sometimes they’re merely practical (“if we all believe this, then we seem to get along better”), and sometimes they’re nakedly cynical (“getting people to believe this is my path to wealth and power”). But neglecting the pragmatic motivations behind beliefs is always at best naïve, and at worst catastrophic. And anyone who declares that his or her beliefs are motivated by nothing more than dedication to truth needs his or her motivations examined most skeptically of all.

  23. Working of the Unborn God of Winds says:

    The object-level discussion of whisper networks is interesting, though it seems to me that that mostly isn’t the point.

    Of course, we can’t get away from all the baggage that’s coming to this post, and I’m sure Scott didn’t want to. I won’t talk about the things he’s alluding to directly, since I understand that’s proscribed in most threads. But it’s worth remembering that although Stalin purged plenty of people who, at least on the subject of Stalin, had it right (and it’s not hard to have it more right on Stalin than Stalin himself did), he presumably also purged plenty of actual conspiracy theorists and nazi agents. (Maybe the Church had a similar thing going on, I really have no idea. Presumably they burned some people at the stake who weren’t scientists but were Satanists or occultists)

    Now, I don’t say that to excuse Stalinist purges specifically or censorship in general, but just to point out that being censored by powerful interests doesn’t mean you have it right. In addition to maintaining their own power, powerful people tend also to have an interest in dissuading generally antisocial behaviors and ideas that, even if wrong, are “dangerous” to expose to uninformed people (which isn’t to say these are good reasons to censor–only that they’re reasons that aren’t “they’re actually right”). And in pluralistic society there can be multiple groups with opposing interests that can censure people. You can imagine a scenario where two opposed and powerful groups persecute one another’s underclass base, but don’t target each other for mutually assured destruction or shared culture reasons, even if only one of the groups (or neither) is right.

    Give this essay to Kolmogorov, a nazi cell in the Stalinist USSR, left-wing activists today, or the people who forcefully agree with the things Scott has been alluding to in this post, and they’ll know what you’re talking about. You have to imagine anyone spouting views that are unpopular in some local social space to them will fancy themselves the wise Kolmogorov who’s seen through the lies of the system, or alternatively, the cool heroic Kantorovich. But some of them are nazi agents in 1939, and I feel Scott’s piece, to the extent that it’s intended to be an honest exploration of the object level ideas it presents, should have done a better job of owning up to that.

    • multiheaded says:

      he presumably also purged plenty of actual conspiracy theorists and nazi agents

      Old Bolsheviks were among the people with the most to fear by far, certainly (Nazi collaborators being mostly local officials and PoVs during ww2, while educated Communists already knew to fear them far more; news of Molotov-Ribbentrop pact massively hurt many loyal Stalinists’ morale). It was more on a personal loyalties/etc basis, like the “Trotsky-Zinoviev block” (not really a thing, Trotsky was ambivalent about lending full support to the anti-Stalin ‘left opposition’ in the 20s and later narrated his stance at the time as far more adamant than it was) – but stressing Trotskyist-sounding/left-Bolshevik ideas like permanent revolution, criticism of resurgent Russian chauvinism, etc – or standing up for the old guard in the party apparat and the military, even the NKVD – would have put one in enormous danger.

      (Re: the NKVD part – a great many of its members actually completely deserved the 1937 purge in particular, and got no better than their victims. The sheer random/slanderous character of earlier purges was in large part on them, even if Stalin was ultimately responsible. For all the ideological cohesion and annihilation of dissent that he pursued, and for all his inability to rein in the obsessive paranoia, it was obvious that handing random citizens/officials free rein to anonymously denounce enemies was actually a massive source of instability.)

      • Working of the Unborn God of Winds says:

        I didn’t know that about the NKVD — that would have been a strictly better example than what I was able to come up with with my level of knowledge about the history.

    • Christopher Hazell says:

      “You have to imagine anyone spouting views that are unpopular in some local social space to them will fancy themselves the wise Kolmogorov who’s seen through the lies of the system, or alternatively, the cool heroic Kantorovich. But some of them are nazi agents in 1939, and I feel Scott’s piece, to the extent that it’s intended to be an honest exploration of the object level ideas it presents, should have done a better job of owning up to that.”

      One of the major things that the Nazi agents of 1939 had in common was, well, full consciousness of being Nazis.

      Well, I suppose dupes have always been useful in spy circles; you can convince somebody that they’re working loyally for a communist spy agency while their leaders are actually Nazis.

      But one of the things that makes a totalitarian regime more and more frightening is that it refuses to distinguish between the two. You might think you were a loyal communist pointing out a problem that was making the lives of the proletariat worse, but in the eyes of the regime you were obviously a nazi sympathizer working to destroy the regime.

      I remember that, in the days not long after the attacks on 9/11, Jonah Goldberg coined the phrase “objectively pro-terrorist” to describe left-wingers who questioned the government in ways he didn’t like. Those left-wingers might imagine that they hated the terrorists as much as Goldberg did, but in reality they were obviously aiding terrorism, and the only sensible way to discuss the left anymore is as terrorist sympathizers.

      Or, to put it another way, it was time to stop talking to the left, and simply view them as a problem to be talked about. And furthermore, that in talking about them and discussing what to do about them, their own narratives about why they were doing what they were doing where wholly irrelevant.

      Once you say “Your sense of why you do the things you do, and your sense of what you’re trying to accomplish are irrelevant” you are basically admitting to that person that you have no desire to communicate with them, that they themselves are completely superfluous to any discussion about themselves.

      Who would accept that? Who could possibly do anything but laugh at or chafe at that attitude?

      I think your analogy is completely inapt. Nobody talking about politics in the English speaking world today is a dupe. There’s no equivalent to the man who thinks he’s working for the USSR but actually his handler is a Nazi; nobody on the internet is writing for Stormfront because somebody tricked them into thinking that Stormfront is actually a communist website.

      People’s narratives about why they do what they do are valuable tactical and strategic information, if you want to convince them that you’re right, or undermine their support. Even if you really think the American left are advocating for policies that will make terrorism worse, they’re still doing it for different reasons than Al Qaeda, and that means different tactics will be necessary to convince them, or to convince neutral observers that they are wrong.

      Telling me that 9/11 was monstrous was never going to make me a Republican, because I never wavered in my belief that it was monstrous and unacceptable.

      When you start saying “You may think you believe X, but actually you believe Y, let me explain why Y is completely wrong” you’re basically refusing to engage in dialog (And just as a PS, the ability to refuse to engage in dialog, the privilege to stop talking to someone and instead turn and talk about them to a receptive audience is an indication of real social power, and is going to look awfully strange if you then turn around and talk about how you have no social power).

      • Working of the Unborn God of Winds says:

        That wasn’t the point I was trying to make and I agree that, if I had been trying to make that point, I would have been wrong. That miscommunication is probably my fault because I decided I had already spent enough time trying to write a post in a comments section and just posted what I had. But my point with nazis is not that you might be a nazi and not know it, it’s that you might be wrong (and wrong in a way that’s harmful) and not know it. And I used Nazism as synecdoche for an idea that people sincerely believed and were ultimately punished for, but was nonetheless wrong and hurtful.

        I don’t believe Scott (slash the vast majority of readers) are Nazis or believe anything even close to as wrong as what Nazis believe. I just wanted an example where as many people as possible would agree that the people in the example were wrong. But obviously that was a questionable writing decision.

      • I remember that, in the days not long after the attacks on 9/11, Jonah Goldberg coined the phrase “objectively pro-terrorist” to describe left-wingers who questioned the government in ways he didn’t like.

        That sense of “objectively” was discussed by Orwell much earlier in the context of Stalinists claiming that Trotskyites were objectively pro-Nazi, and similar charges.

        • The original Mr. X says:

          As I recall, Orwell himself also made similar accusations about pacifists during WW2.

    • Steve Sailer says:

      “a nazi cell in the Stalinist USSR”

      There really weren’t many Nazi cells in the USSR, given that the Nazis viciously hated just about every ethnic group within the USSR.

      What’s extraordinary about Stalin in 1937 is how many people he murdered who were largely on his side. Hitler murdered his opponents in vast numbers, but Stalin murdered remarkable numbers of his supporters.

  24. MB says:

    Some people are bad at reading the tea leaves and adapting.

    Galileo chose a bad time for his satire — at the height of the Counterreformation, when religious passions ran high and the figure of the Pope was at their center. Fifty years earlier or later, he might have got away with it, based on his personal friendship with high-placed people. Or a different pope might have been amused, not insulted by his presumption.
    Leaders’ personalities make a big difference. What Sakharov said and wrote would have got him shot in the 1930s, was grudgingly tolerated under Khrushchev (maybe due to the WWII victory and Sputnik launch-related optimism?), and then was repressed again under Brezhnev.
    The important thing was not whether Galileo was right, but whether the Church was firmly in control and had the power to punish and/or reward Galileo as it pleased. Maybe the Pope even agreed with Galileo in private that the church’s stance was outdated and it was time to reform it — save for the unfortunate connotation that Reformation had acquired by then, which made him reluctant to let one more challenge to his authority go unpunished.

    Personalities like Watson or Summers don’t realize that what was daring or controversial science 50 years ago comes off as tone-deaf and beyond the pale nowadays. Opinions that might have been tolerated in the name of “freedom of expression”, when it was mostly invoked to provide an entry point for leftist opinions, have no more room now that leftists have consolidated their control over universities and other public fora and are locked in a struggle for the control of the whole country, against their hated right-wing enemies.
    Maybe once the US are made safe for socialism then this stuff will again be safe to circulate in some sort of samizdat form (if one has a sharp wit to amuse, an impeccable background, and/or high-placed protection). But now is not the right time and absolutely not in a public fashion.

    There was a time when worshippers of Venus or Hermes probably incurred the risk of being stoned to death or torn to bits while alive. A few hundred years later, it was a harmless eccentricity (for the gentry) and Western European aristocrats once again claimed descent from pagan deities and demigods.
    Maybe Deng, Zhou, Zhao, *and* Mao all had their private doubts about Communism at one point or another, but nobody could afford to voice them publicly until even the shadow of an opposition had been eliminated from mainland China. Afterward, they could afford to loosen the belt a bit — though it still helps to be well-connected when starting a business.

    Not every good scientist has the requisite mental flexibility. Sometimes change happens too fast — and then one has to scramble to keep up and always end up on the right side, so that one’s past heretical opinions aren’t held against oneself.
    In other occasions, naive (borderline autistic?) people know that what they are in the wrong, but assume that due to their fame and to their past services their displays of “original” thought will be tolerated. But a regime or party that derives its power from its control of public opinion must never let such controversial public stands go unpunished, especially with victory still hanging in the balance.
    One would think that, after Zeno, Galileo, Bruno, Sakharov, and many others, scientists and/or natural philosophers would have learned their lesson. However, scientists, as a rule, are surprisingly naive and don’t know much about the past or about the present.

    • Machine Interface says:

      I often see US conservative-types projecting that the liberal-leaning censorship, especially on race-related subject, will just keep growing forever and progressively engulf everything. As seen from Europe, this seems ridiculous.

      Ten-twenty years ago, Europe, especially France, was at the height of the antiracist craze. Antiracist organizations seemed powerful and unstoppable, able to destroy someone’s career on a whim, the far right was seen as the antithesis of what every other party stood for, and there were frequently media-pumped affairs because X or Y public personality said this racist thing.

      Nowadays this seems really far away; antiracist organizations are still there, they still occasionally sue people, but it has faded into the background. Parties which were saying “the National Front is the antithesis of everything we stand for” are now saying “yeah ok our economic program is the same but they’re still racist”, many people and personality on the right are now leaning towards alliance with the far right, political analyst can now talk dispassionately about the National Front without feeling the need to remind us every other sentence of how evil they are.

      Discourse critical of islam is now heard on every side of the political spectrum, trials for apology of terrorism and jihadism have replaced those for hate speech in news headlines, people who just 5 years ago were calling former prime minister Manuel Valls a fascist for dismantling illegal Roma camps and deporting their occupants back to their countries are now grudgingly admitting that dismantling the Calais migrant camp was necessary and good for many reasons.

      People get blasé, what is offensive today is not what will be offensive 20 years from now; the orthodoxy doesn’t grow as much as it moves around — for each new domain it conquers, an old one is left behind.

      • MB says:

        The part I agree with:
        It is important to keep one’s head down, while at the same time nodding enthusiastically, until the mob gets bored and the danger passes.

        But this sounds like a feat of physical dexterity that not everyone can manage. Besides, doing it has its own problems: what if the mob never goes away, won’t one get tired after a while?

        Here is my disagreement:

        I never saw any leftist publication admit that the dismantling of the Calais refugee camp was good or even necessary. At most there is an uncomfortable silence or an attempt to change the subject for tactical reasons — the camp was embarrassing and contributed to the Left’s losing the Pas-de-Calais to the FN. Here is a title from L’Humanité: “Jacky Hénin va saisir le Préfet face à la décision de la maire de Calais d’interdire la distribution de repas aux réfugiés”. Here is one from Le Monde: “Un an après l’évacuation de la jungle, les conditions de vie inhumaines à Calais”.

        Leftists are tenacious and never give up a cause they have embraced. Even now it is forbidden to criticize Mao in China. Sacco, Vanzetti, the Rosenbergs, and Fidel Castro are still heroes to the US Left.
        To the extent that people stop caring, it is because a new generation will have arrived on the stage. But embarrassing episodes will have been edited out of history or turned into net positives. 20 years from now, those Calais campers will be hailed as heroes and receive an official pension in compensation for the indignity they had to endure. Valls will still be a fascist in the history books, next to Le Pen and Sarkozy, unless he makes a full conversion soon.

      • Aapje says:

        @Machine Interface

        France is not Europe.

        Anti-racism is stronger than it ever was in The Netherlands.

      • Steve Sailer says:

        “Discourse critical of islam is now heard on every side of the political spectrum”

        All it took in France to make Houellebecq less beyond the pale intellectually were a number of spectacular Islamist terrorist massacres in the heart of Paris in 2015.

    • Steve Sailer says:

      One _very_ important later 20th Century biologist with whom I was distantly acquainted escaped the current version of the Inquisition by:

      – dying somewhat young

      – doing most of his work in equations, and when writing using a complex prose style that was hard to quote in chunks of less than a couple of hundred words.

  25. sustrik says:

    I’ve lived through 70’s and 80’s in Ostblock (Czechoslovakia). Admittedly, that was the era of decay of the system, but what follows may have applied to some extent even earlier.

    First, it was not that people were unaware that communism sucks. Look at the big trove of jokes from the era. These jokes were told by everyone, even small kids, not just by the dissidents. In the terms of the lightning parable, everyone knew that light comes before sound but everyone also knew that only way to communicate that was through a joke.

    Second, it wasn’t like secret police was doing hard investigation work to find people who believed that communism sucks. They’ve understood that everybody does. They also knew who dissidents were and, in fact, interacted with them on regular basis.

    Third, in many cases one haven’t come to be a dissident by writing a naive letter to the authorities. Too often it was just something silly and mundane. Offending a wrong person. Making a silly joke in front of a wrong person. IIRC, Kundera tried to make this point in “The Joke” novel.

    That, I guess, sheds a bit different light on the “whisper network” concept. If there was a network it wasn’t that much about sharing prohibited information but rather about supporting each other in the face of harassment by the system.

  26. Le Maistre Chat says:

    Scott, you don’t make an argument for what the Church was supposed to do differently to be sincerely pro-science.
    Roger Bacon was never censured, and the Pope was so excited about his work that he let him go over his superiors’ heads on some bureaucratic rule about publication.
    Pietro d’Abano was executed for demonology and Western Christendom merrily went on using his medical findings. I know, I know, Godwin’ s Law, but if a doctor from Nazi Germany who’d made a real medical advance was found to also be doing neural net research in hopes of duplicating Hitler’s brain, would banning him from science prove our society is accidentally anti-science?
    Bruno and Vanini had a New Age mindset, not a scientific one. It may have been bad to make their teachings a capital crime, but they don’t support your argument.
    That reduces your list to “16th century Christendom may have retarded itself by killing iconoclast scientist Michael Servetus.”

    You might strengthen your hypothesis by shifting it to “the Reformation was good for science because afterwards scientists had a selection of orthodoxies to live under rather than a monolith.”

    • moscanarius says:

      I think the argument would be that the Church should have been less strict in enforcing the official religious interpretations, just as she was not strict in defining which scientific hypotheses could be advanced. At least, the Church should not have killed the heretics, but instead commuted their sentences to something lighter.

      Which, I’m afraid, is both reasonable for us moderns and impractical for the medieval churchman. As some have pointed in previous comments, the actual number of burned heretics was not that great, and the Inquisition was often less harsh than the secular courts. We tend to forget that the Church, powerful as she was, was not the only power in Europe: there were also the violent and not-so-sofisticated noblemen, and there were the numerous, violent, superstitious peasants who really believed in witches and werewolves. I would not be surprised if the low leniency towards the heretics was actually all the leniency that was possible to have before the people rioted and burned everything down.

  27. nimim.k.m. says:

    In any non-obvious example of object-level scientific belief that has real life relevance, it seldom is clearcut how the things really are. Often it makes sense to actually ask, does lightning become before thunder, or the other way around? And this affects the problem of discussing whether the object-level issue can be discussed in public. Substituting an obvious example skips all the inherent ambiguity, and is a disservice to a discussion about any particular scientific disagreement you might have had in mind.

    How would the situation look like if the establishment truth actually is “lightning comes before the thunder”, and it’s the whisper network that insists otherwise?

    This brings us to one glaringly obvious failure case: “I know that lightning comes after the thunder, like all the other honest scientists in my whisper network! My belief is validated by the whisper network of true and honest curiosity-seeking scientists! (Which, by the way, also makes me feel like the underdog and the romantic revolutionary scientist hero of the story.) I see every day lots of honest scientists who are savvy enough to say the official truth publicly, but I bet that privately they secretly believe as I do! Everyone who agrees with the official message is either naive or dishonest!” How, exactly, you would test your beliefs in Popperian fashion?

    As far as I know, the sincere belief that they truly are the underdogs that Have The Truth, who unfortunately stay as the underdogs because they continously face setbacks orchestrated by the Establishment’s False Science (which is obviously false because they have studied the True Science and judged it Rational and and observed that It Simply Makes Sense), is one of the main motivators of the leftist extremists (False Science being the so-called neoliberal orthodoxy of economics).

    • Tracy W says:

      >How, exactly, you would test your beliefs in Popperian fashion?

      This might not be exact. But, rough idea, video the start of a thunder storm. Or several thunderstorms. Show the video to some people who know nothing of your beliefs and ask them to record when they first hear thunder. Don’t tell them it’s from the start of a storm.

      Show the same video to some other equally ignorant people and ask them to record when they first see lighting. Again don’t tell them it’s from the start of the storm.

      Put the two groups timings together and see who heard/saw what first. You can add other tests.

      But the basic idea behind science is “look at reality”. Not “participate in a network of people agreeing with each other.”

      • nimim.k.m. says:

        >How, exactly, you would test your beliefs in Popperian fashion?

        This might not be exact. But, rough idea, video the start of a thunder storm.

        Eh, by that point I was no longer thinking about the lighting example in particular, but the more general problem. But rereading, what I wrote my text did not reflect my internal thoughts very well.

        First, looking at what the reality and the reality alone without any preconceptions is quite difficult. (Consider any serious scientific disagreement.) Second, even if you trust yourself to be a good scientist (a bold assumption, looking at the silly mistakes many quite smart people have made, but let’s overlook that for now), I don’t believe anyone has enough time and resources to form an informed opinion about every dispute of interest by themself. If you want to inspect the nature of the reality by yourself, especially if you choose any non-trivial aspect of it (unlike the lightning example), congratulations, you’ve found yourself a new career in terms of time commitment required (in other words, seeing by yourself is synonymous with conduct research). Even then you probably have to collaborate with other people to get anything done, and about everything else, you are always to some extent at the mercy of the rest of the scientific community.

        • Tracy W says:

          I’m wondering if I misinterpreted what you are saying. I thought you were wondering how you could test the scientific reliability of the whisper community.

          But my understanding of Scott’s description is that you’d start off with the testing of reality and find it throws up results incompatible with the orthodoxy and only then might you, if lucky, get in touch with the whisper network.

          There’s an asymmetry in science. Seeing reality clearly might be hard. But learning that you’re wrong in one of your views about reality is fairly common. Happens to me all the time.

          • nimim.k.m. says:

            I’m trying to argue that in real science, “testing of reality” is not as straightforward as it sounds. Scientist does not work alone, Scientist does not form the concepts and methods they work with in isolation.

            Furthermore, “testing of reality” is not always an option. Replicating studies requires serious effort (measured in labor). Analyzing the studies often requires a great effort (measured in mental work). And very few of us are scientists conducting research, so we can barely evaluate the works of others to understand anything about the word.

            So, the psychological biases I was talking about in the top-level surely play a role.

            Also in particular, the simple hypothetical example about thunder is unfair, because the questio about speed of sound vs speed of light a problem we have already solved ages ago and the answer is “obvious” to us. It obfuscates all the difficult parts of scientific process.

          • Tracy W says:

            @nimim.k.m: I think you are thinking about this testing of reality in the sense of testing an existing description of the world to see if it works, while I’m thinking of it in the sense of starting off by noticing that something doesn’t seem to fit the description. And then you start wondering if it’s you who is observing it wrong.

            I agree that deliberately testing reality is not as easy as it sounds. But, given things like the replication crisis in psychology, I think it’s pretty important.

            Unless something is used by practical types (eg plate tectonics and oil exploration) or is very controversial (like global warming) so you can look at what the attackers are saying and assess the strength of their arguments. Although that again comes back to assessing reality.

            So all our knowledge is contingent.

  28. Sigivald says:

    Did Giordano Bruno die for his astronomical discoveries or his atheism? False dichotomy: you can’t have a mind that questions the stars but never thinks to question the Bible. The best you can do is have a Bruno who questions both, but is savvy enough to know which questions he can get away with saying out loud. And the real Bruno wasn’t that savvy.

    One small problem with that ref. the Stalinist comparison.

    The Stalinist USSR pretended to like dissent, and imprisoned, punished, or killed people for doing so, dressing up their punishment as for treason or espionage or “capitalist wrecking”.

    The medieval and renaissance Church(es) did not pretend to like dissent about doctrine, especially core doctrine like “Christ is the son of God” or “Christianity is the correct, only true religion”.

    They executed people quite publicly for refusing to recant heresy, detailing exactly what they were heretical about (if not exactly what the details of their heresy were, to prevent contagion) and expressing as clearly as possibly exactly what beliefs were safe to express.

    So while there’s still an obvious, necessary conflict for someone of a scientific bent in such a context, there’s nothing much hidden and no serious learning curve about what you need to Pretend To Assent To to be safe.

    The Church’s whole “thing” was leading people to orthodoxy, after all.

    (Eponymous, just above, is quite right – and remember that the usual punishment for (not-uncommon!) heresy was not execution, but re-Neducation. By which I mean, “being forced to recant the heresy and publicly assent to the Church’s opinion”.

    It’s the ones that refuse to recant that get burnt at the stake, almost entirely.)

    • reasoned argumentation says:

      The Church’s whole “thing” was leading people to orthodoxy, after all.

      Imagine how horrible it would be to have a system where the payoff is to finding new ways to find heresy and never explicitly stating what a person has to state to be orthodox.

      • Le Maistre Chat says:

        Moldbug said near the beginning of one of his interminable essays “I would rather live in Popper’ s Open Society than medieval Christendom, but I’d rather live there than under a state religion that lies that this is the Open Society.”

        • reasoned argumentation says:

          During the disinvitation from some tech conference or another matter he also specifically stated on Hacker News that he’d affirm whatever statement of orthodoxy was required if someone would provide it – no one did (he’s since been banned from Hacker News).

          • Nornagest says:

            Would you? Provide a statement of orthodoxy that you can ritually recite to be allowed in and you’re basically saying that all you care about is the ritual. I’m having a hard time thinking of belief systems that wouldn’t undermine.

          • reasoned argumentation says:

            https://en.wikipedia.org/wiki/Nicene_Creed

            (Part of) Moldbug’s point was that there’s no formulation of progressive orthodoxy that can be stated without falling apart due to the constant progressive pressure to find more and more heresies – or that doesn’t blatantly contradict reality.

          • Nornagest says:

            Would you believe Mencius Moldbug if you got him to say something starting with “I believe in egalitarian ideals, in the will of the people as the sole source of political legitimacy, in the equal worth and dignity of all human beings regardless of race, gender, or national origin…”?

          • reasoned argumentation says:

            He’s said the second half of that unprompted.

            No one was taking him to task over the first part. If that’s the orthodoxy, he’s within it – he’s not, therefore it’s not.

  29. Eponymous says:

    It’s tempting to imagine a world where Servetus, Bacon, and Bruno followed Aaronson’s advice. They pursued their work in optics, astronomy, anatomy, or whatever other subject, but were smart enough never to go near questions of religion. Maybe they would give beautiful speeches on how they had seen the grandeur of the heavens, but the true grandeur belonged to God and His faithful servant the Pope who was incidentally right about everything and extremely handsome. Maybe they would have ended up running great universities, funding other thinkers, and dying at a ripe old age.

    How do you know that the great majority of medieval thinkers who harbored theological doubts didn’t pursue exactly this course? I would guess that, given the consequences for heresy, the true population of heretics must have been several orders of magnitude greater than the ones who were caught.

    This is a general problem with the Kolmogorov option. It’s a reasonable guess that it has been pursued by most intelligent people throughout history, given the evident insanity of humanity. While perhaps wise from a private perspective, the social consequences are grim. As a species we are trapped in a grand prisoners’ dilemma, in which those of us who see the truth are condemned to stand by mutely as our fellows pursue their folly.

    The only upside is that, knowing this, we can guess that sanity is not quite as rare as it seems; that surely there are others who see the world as we do; that we are not alone.

    • Eponymous says:

      More simply:

      Given the incentives and the frequency of public heretics, it’s a reasonable guess that private heretics have been quite common throughout history. This should cause us to lower our estimates of the likely benefits of pursuing this option today.

    • YehoshuaK says:

      The only upside is that, knowing this, we can guess that sanity is not quite as rare as it seems; that surely there are others who see the world as we do; that we are not alone.

      I’m curious. Why is it self-evident that you are the sane ones? Perhaps, contrariwise, you are insane, and your insanity prevents you from noticing that you are insane?

  30. Machine Interface says:

    Oh also: at the same time the Church was condemning Galileo to house arrest, it was quietly and uncontroversially adopting heliocentric-based maths to construct its new calendar and calculate the dates of religious celebrations with more accuracy. It’s also worth noting that even as Galileo was proposing his model, his theories were in fact already outdated — Kepler had beaten him to the punch and Kepler’s models was actually more accurate (it’s no coincidence that there are laws of motion named after Kepler, but not after Galileo). Kepler was never worried.

    So it’s more that the condemnation of Galileo was an “accidental swatting of a scientist” — it didn’t even make a dent in the development of the heliocentric model; no important knowledge was lost or delayed.

  31. NoahSD says:

    There are two separate issues here.

    First, there’s the question “Given that a society kills people who believe certain things, how does the specific list of such deadly beliefs affect our view of their morality?” I think the question answers itself. Killing people for their beliefs is just really horribly wrong no matter what beliefs happen to be on your list.

    Second, there’s the question of how killing people for certain beliefs affects science. I think this is a complicated and interesting question, but it’s important that it not be conflated with the first question. Some of this stuff even ends up being dumb luck. E.g., from a Christian perspective during the Dark Ages, the idea that pagan gods exist might have seemed roughly as absurd and offensive (maybe even significantly less absurd and more offensive?) as the idea that there are other planets revolving around other suns (possibly with other intelligent life!). But, killing pagans probably didn’t hold science back much.

    • Anonymous says:

      Killing people for their beliefs is just really horribly wrong no matter what beliefs happen to be on your list.

      Why?

    • B_Rat says:

      E.g., from a Christian perspective during the Dark Ages, the idea that pagan gods exist might have seemed roughly as absurd and offensive (maybe even significantly less absurd and more offensive?) as the idea that there are other planets revolving around other suns (possibly with other intelligent life!).

      Yeah, that’s why they punished the guy who suggested other inhabited worlds, Nicholas Cusanus, by making him Cardinal and Papal Legate.

  32. Alex M says:

    Let’s be honest here Scott. When you describe the Andrey Kolmogorov situation, are you secretly talking about the Google “diversity culture” and James Damore? Because everything that you’ve described sounds like a thinly veiled parable.

    • Nick says:

      Read the rest of the thread, when you can; Scott has acknowledged that he had some things in mind, but not any particular one. There’s no need for “honesty” here seeing as his point is more general than that.

      • suntzuanime says:

        “There’s no need for honesty here” is pretty much the central thesis of the article, you kind of have to question whether the people who are like “but seriously, are you a heretic” are acting in good faith.

        • 天可汗 says:

          It’s simple risk evaluation — the same reason goons infiltrate and destroy everything. To the extent that your faction is on top, you want to make sure nothing ever changes, because change is almost certainly going to be bad for you. There’s always room for refining the apologetics of your faction, of course, but there’s a reason these people never engage beyond the level of smug one-liners and threats.

          Even the existence of communities that haven’t been wholly annexed to the orthodox faction is a threat. Remember that Cass Sunstein article about how it’s actually bad for people to have lives because Hitler? The obvious interpretation of that is that he’s trying to say atomization is good for oligarchs because it lowers the risk that a competing movement will arise. The same logic applies here.

          I don’t think this is a conscious strategy. Members of the same subculture tend to share behavioral traits, and selection applies over subcultures — that’s all. But it’s probably a bad idea to let Inquisitors into your group sex club.

          (In the Discord I run, I permaban anyone who’s so much as dated one of these people. That’s maybe a bit extreme. But the geek social fallacy of inclusion leads to some major implicit blackmail problems, and I don’t see many people who are serious about avoiding them.)

  33. timoneill007 says:

    “The Church didn’t lift a finger against science. It just accidentally created a honeytrap that attracted and destroyed scientifically curious people. And any insistence on a false idea, no matter how harmless and well-intentioned, risks doing the same.”

    Unfortunately all this strange article has done is cherry pick some examples, several of which are highly dubious (Bacon), of “scientifically curious people” who got destroyed for purely theological reasons and, using a decidedly wobbly modern analogy, argue that this meant the Church somehow “accidentally” suppressed science. This ignores two things: (i) the lines between theology and “natural philosophy” (what we call “science”) were fairly clear in these periods and (ii) plenty of people were still merrily pursuing “science” on the “natural philosophy” side of those lines, despite this suppsed “accidental” suppression. These scattered (and dubious) examples don’t support the weight of this line of argument.

    This whole rather contrived argument seems to be constructed simply to prop up a weak version of the old Draper-White Thesis while admitting the fact that the whole “the Church suppressed science and burned scientists at the stake for science” thing is a total myth.

    • sconn says:

      What if the Church never suppressed science — but did suppress political science and economics? Because given that the church did (at the time) have official teachings against both democracy and capitalism, it almost certainly slowed the development of both.

      Of course maybe you see that as a good thing?

      • bapgwh says:

        I think it’s possible you have an exaggerated idea in your head about how strongly and uniquely the church preached against Usury. Moses, Plato, Aristotle, Cicero, Cato, Seneca, Muhhamad, etc. all denounced usury multiple times, e.g. Cato “And what do you think of usury?”—”What do you think of murder?”

        Italy, especially Northern Italy, was the richest place in all of history by per capita GDP by 1500, possibly going back to the 1300s, and many other Western European Countries were the richest they had ever been per capita by the 1400s, probably 1300s.

        Switzerland practiced Democracy in the Middle Ages and beyond to name one counterexample.

        • Nick says:

          When it comes to political science and economics, it’s worth mentioning the School of Salamanca. See also Graeber’s book Debt* re usury; as I recall, it’s discussed a fair bit, although it’s been a few years for me. Lord Acton once called Aquinas the first Whig; this is a distortion of Aquinas’s views, of course, but one cannot underestimate the importance of advancing ideas like the rule of law.

          *You can find a pdf free pretty easily.

        • YehoshuaK says:

          Moses, Plato, Aristotle, Cicero, Cato, Seneca, Muhhamad, etc. all denounced usury multiple times,

          I don’t know about the others, but Moses’ opposition to lending-at-interest is much exaggerated. To be specific, the Torah specifically limits the prohibition against lending-at-interest to loans between two Jews. Borrowing-at-interest from a non-Jew, or lending-at-interest to a non-Jew, is permitted.

          This seems to me to imply that lending-at-interest was never seen as wrong-in-itself (unlike murder or theft, both of which are forbidden whether the victim is a Jew or a non-Jew). I understand the prohibition against lending-at-interest between Jews as meant to encourage brotherly behavior and reliance on God, not as a statement that “lending-at-interest is inherently wrong.”

  34. Scott thanks for this post. I have literally this week gone through an experience similar to that of Kantorovich where debating gender differences with a feminist led to a whole lot of drama. Only by using the “dark arts” and virtue signalling was I saved from being burned at the stake. In the future I plan to remain engaged in the war of ideas but to be far more strategic about who I engage with and how.

  35. Calvin says:

    When you’re talking about “truth seeking” and open-mindedness as a valuable trait which goes against orthodoxy (whether it is lightning-thunder, or religion, or stalinism), you argue that the problem is that certain valuable institutions which prize truth-seeking will be adversely affected because they produce more people than average who challenge the orthodoxy.

    Isn’t the solution to this problem to have truth-seeking be taught in a more fundamental/universal institution? The orthodoxy values the truth-seeking trait, so they wouldn’t oppose teaching it more widely – but that creates the foundation for a generational shift where the orthodox false belief will no longer hold any sway.

    What I’m talking about is teaching the traits of inquisitiveness, truth-seeking and open-mindedness in K-12 (kindergarten to grade 12) and in basic parenting. You can still teach the orthodox false belief to children along with the truth-seeking trait, and trust that they will come to learn the truth in time. And when the first Kantorovich comes along, he would actually be effective in overturning the orthodoxy.

    I’m partial to this method since it mirrors my own experience with religion/agnosticism, growing up in a religious environment and then coming to my own conclusions about religion after being taught the value of the scientific method and of questioning my own beliefs.

  36. The original Mr. X says:

    So, if the Church was so anti-science, either consciously or by inadvertently setting up honey traps for medieval edgelords, why is it that (a) all these people existed, and (b) Europe c. 1500 was one of the most advanced regions on Earth?

    • Nick says:

      Prediction: every time Scott writes a post like this, or the Dark Ages one, or whatever, the number of Catholics in the comments section fervently correcting him goes up by 1. 😀

      • Deiseach says:

        the number of Catholics in the comments section fervently correcting him goes up by 1

        Nobody expects the Spanish Inquisition, which is how you’ve all ended up tied to stakes with the flames licking at your toes! 🙂

    • Scott Alexander says:

      …the whole thesis of Part I of this post was that the Church wasn’t anti-science, and that was a myth.

      • The original Mr. X says:

        In practice, “The Church inadvertently set up honey-traps for medieval scientists” should have pretty much the same stultifying effects as “The Church was anti-science.”

        • Scott Alexander says:

          If Marxism was so anti-science, how come [list of great Soviet scientists]?

          I’m not saying this is an overwhelmingly large problem that will prevent any progress at all. Just that it makes things worse. I think there was a pretty big upswing in science in Protestant countries after Protestantism, though I could be wrong.

          • Nornagest says:

            Marxism’s a great example, actually. It wasn’t anti-science — in fact it thought of itself as having a scientific foundation, enthusiastically adopted the trappings of science, and made a point, at least in its Leninist manifestations, of sponsoring scientific institutions.

            But that doesn’t mean it was necessarily friendly to any particular avenue of scientific inquiry. As much as it liked science, it couldn’t infringe on Marxist doctrine without compromising its own legitimacy, so right away there was this little separate magisterium that you couldn’t touch as a researcher. And the more totalizing it got, the bigger that magisterium became and the more toes you could step on as a scientist by researching the wrong things in the wrong ways.

          • Matthias says:

            Protestantism was more likely to be successful in areas with relatively stronger burghers and centralizing monarchs, and the most prominent country with both of these that remained Catholic, France, was scientifically very successful.

            Changing attitudes towards censorship may have played a part here – the religious wars soured a lot of intellectuals on the desirability of everybody burning each other at the stake for being indexically heretical – but it’s not obvious that was stronger in Protestant contexts than Catholic ones, and Protestant states probably had more coercive capacity. Probably a more important role is that of state patronage in paying intellectuals to do things of immediate worldly importance – the Enlightenment’s temporal and geographic correspondence with mercantilism and absolutism is not coincidental.

            (Robert Wuthnow’s excellent Communities of Discourse (pdf) does a comparative look at the rise of Protestantism, the Enlightenment, and Marxism, if you are into that kind of thing.)

          • The original Mr. X says:

            If Marxism was so anti-science, how come [list of great Soviet scientists]?

            That’s not a valid comparison, because:

            (a) Scientists in Marxist countries have generally had little to do with their governments, whereas the people in the list I gave were all churchmen, in many cases quite high-ranking ones. If lots of Marxist countries had leaders and senior officials who were also enthusiastic scientists, the claim that Marxism is anti-science would become rather difficult to maintain.

            (b) Christian Europe was the only part of the world where science as we know it took off. I guess you could argue that the rest of the world just had even more anti-science structures or something, but that just starts to look pretty circular and unfalsifiable. The analogy here, I guess, would be trying to claim that Marxism was anti-science if Marxist countries were producing far more scientific advances than non-Marxist ones.

            I think there was a pretty big upswing in science in Protestant countries after Protestantism, though I could be wrong.

            I haven’t seen any evidence for a big upswing in science before the late 17th century. I suppose this is technically “after Protestantism”, but it’s sufficiently after Protestantism in most countries that I don’t think you can just assume that it’s the natural result of the dead weight of Rome being removed. Plus, Protestant countries of this period were every bit as ready to enforce doctrine orthodoxy as Catholic ones (or even more so — the Elizabethan and Stuart attempts to enforce religious orthodoxy were far more brutal and wide-ranging than the ones which prevailed in England before the Reformation, for example), which by your argument ought to mean that science wouldn’t take off in Protestant countries.

          • reasoned argumentation says:

            Marxism’s a great example, actually. It wasn’t anti-science — in fact it thought of itself as having a scientific foundation, enthusiastically adopted the trappings of science, and made a point, at least in its Leninist manifestations, of sponsoring scientific institutions.

            It is actually (a great example) – but of Goodheart’s law. It had to be hugely anti-science in practice since the justification for it was science (that and representing “the workers”). The Catholic Church didn’t because it didn’t base its claim to legitimacy on science.

            Orthodoxies that claim legitimacy on the basis of being scientifically correct about specific claims are especially vulnerable to spiraling into very strong science denial.

          • Matthias says:

            If lots of Marxist countries had leaders and senior officials who were also enthusiastic scientists, the claim that Marxism is anti-science would become rather difficult to maintain.

            Elite recruitment in Leninist states tended to be biased towards engineers, whose similarity to scientists can be played up or down depending on how you feel about engineers, Marxism, science, &c.

          • The original Mr. X says:

            Elite recruitment in Leninist states tended to be biased towards engineers, whose similarity to scientists can be played up or down depending on how you feel about engineers, Marxism, science, &c.

            Well, I don’t think anyone’s ever claimed that Marxism is anti-engineering, regardless of its attitude towards other/proper (delete as you see fit) sciences.

          • Jaskologist says:

            Didn’t the Soviets end up purging the engineers as wreckers, too?

          • John Schilling says:

            (a) Scientists in Marxist countries have generally had little to do with their governments,

            Who was paying their salary, living expenses, and/or research costs, if not their Marxist governments?

        • Nick says:

          Not so, since we can expect most of them were genuine believers or had the good sense not to speak out.

  37. reasoned argumentation says:

    Catholicism doesn’t seem quite true, but it’s not doing too much harm, really, and it helps keep the peace, and lots of people like it. Just ignore this one good prosocial falsehood that’s not bothering anybody, and then you can do whatever it is you want.

    Catholicism looking better and better compared to progressivism, eh? The trinity is almost perfectly calculated to not have real world implications and the Church was pretty good about getting dogma out of the way when it contradicted reality. Progs are the opposite. The stuff you can’t talk about is the most important stuff and they continually expand the areas where you’re not allowed to notice reality.

    • suntzuanime says:

      My favorite Catholic doctrine that’s calculated to not have real world implications is transubstantiation, where they’re like “this wine definitely actually becomes Christ’s blood, but still behaves in every way like wine”.

      • Anonymous says:

        Mine’s the 1312 definition of the soul. 😉

        • theofloinn says:

          You mean as the substantial form of a living thing? Heck, that goes back to Aristotle. And if you had paid attention in Greek or Latin class, it would be obvious on its face. “Anima” simply means “alive.” It’s not a “ghost in the machine,” a la Descartes and the Scientific Revolutionaries.

          And transubstantiation is simply a logical complement to transformation. Everything we do wrt modern natural science has to do with forms — the weight, color, thickness, pH, etc. of things — but not with substance itself. It would be like examining a letter like “H” and determining simply from its physical shape and properties that it meant a breathy sound (as in English), a nasal sound (as in Russian), or the sound “mi” (as in Cherokee). That is, the form of the symbol does not determine its substance (or even that it has one: the markings may have been made at random by rainwater.) Hence, it is possible for
          Forms to change without affecting the substance.
          Substance to change without affecting the form.

          • Nick says:

            This is all true. But don’t downplay the actual miracle involved: in transubstantiation, God is changing the substance without affecting the accidents one bit, which doesn’t (to my knowledge) otherwise occur in nature. Aristotelianism allows for the possibility of transubstantiation, but without revelation, we have no reason to suppose it would ever occur.

    • vV_Vv says:

      Indeed.

      That’s the difference between a mature religion that has been around for thousands of years and a fairly novel secular religion that got caught into a holier-than-thou signaling spiral.

    • sconn says:

      Catholicism does have a few beliefs that absolutely do have real-world implications: for instance, that it’s a sin to lend at interest; that the death penalty is required by divine justice; that any form of contraception is opposed to the natural law; that one may never break the moral law even to save lives; that error has no rights; that baptism is absolutely required for salvation … I could go on.

      You may notice that these are some of the teachings that have changed over time. I suspect the reason is that a belief that has real-world consequences is the only kind anyone ever wants or needs to change. It costs a person nothing to agree that Mary was conceived without sin; there’s no possible way you could disprove it, so why not just say the church is right? If you’re the Vatican, though, and you’re making money off charging interest, you have a very strong interest to suddenly be inspired to change the definition of usury. Meanwhile there are good reasons why ethical people might have big problems accepting other bits of the moral law, because it isn’t just a matter of saying the right formula at no cost to you.

      • Nick says:

        It’s not a sin to lend at interest; the sin of usury is a bit more complicated than that, and this teaching has not changed, although it has grown much more difficult to determine what does and does not qualify as usury. The death penalty is not required in any circumstances by divine justice, although it is permissible in some circumstances; this teaching has not changed, pace the rather strange recent remarks by Pope Francis. Baptism in the usual sense is not absolutely required for salvation; this strict interpretation of extra ecclesiam nulla salus is the false teaching of the heresiarch Fr. Feeney.

        If this seems mean, understand I’m just upholding this thread’s tradition of undermining meta-level arguments at the object level. 😀

        • sconn says:

          So we’ve always been at war with Eastasia, is that it?

          The Church has had and still has teachings which have real-world applications. Some of these are commonly believed to be unchanging; others are not. After a teaching has changed, all Catholics agree it was the kind that can change. Lending at interest, for instance, was once considered a sin; now it is not, but that doesn’t erase the struggle of a would-be merchant who couldn’t get a loan back in 1400.

          My point is not to disprove infallibility (though, don’t get me wrong, I don’t believe in it; once I committed to seeking truth instead of trying to explain away inconsistencies, I soon could no longer be Catholic) but to point out that some Catholic teachings, both past and present, do have real-world implications. It is not cost-free to accept them. I know a number of women who have endured grave complications to their health from obeying church teaching on birth control, for instance.

          In the past, I think we can safely say Church teaching delayed the advent of democracy and capitalism. These things only got going after people ignored the Church on those topics and went ahead anyway. Eventually the Church caught up. So I don’t think we could give medieval thinkers the advice “just say you believe whatever they tell you to, and confine your research only to stuff that matters, rather than the theoretical stuff the church teaches.” If they’d done that, there are a lot of developments that wouldn’t have happened.

          • Nick says:

            I actually don’t disagree with your point re which things people want to change. What I disagree with is that there are object-level cases of actually changing supposedly unchanging teachings, which is what people are all the time wanting to do; and as a consequence, I don’t think they should bother pursuing this, seeing as such thing are regarded as unchangeable for a reason. Claiming “after a teaching has changed, all Catholics agree it was the kind that can change” and then giving an example at which I’d already linked a long and in-depth article arguing against exactly that does not constitute much of a response.

            If you’re not interested in the object-level debate, that’s fine. I’ll say again I don’t actually disagree with your meta-level point, but I do obviously disagree about what conclusions one ought to draw from it; Catholicism being true, the doubters would be better served by understanding said teachings than by going against them, with privately or publicly. I should also admit that my last sentence was pretty misleading in this regard, seeing as I don’t disagree with said point, and my purposes are really at a tangent to what you were arguing for in the first place.

      • reasoned argumentation says:

        Catholicism does have a few beliefs that absolutely do have real-world implications: for instance, that it’s a sin to lend at interest; that the death penalty is required by divine justice; that any form of contraception is opposed to the natural law; that one may never break the moral law even to save lives; that error has no rights; that baptism is absolutely required for salvation … I could go on.

        You’re trying to take this into the object level by pointing out that Catholic orthodoxy doesn’t match progressive orthodoxy. We all already knew that.

        Disputing your points would require arguing that the Catholic Church was in accord with reality where progressives aren’t – which was exactly the discussion that the post was attempting to avoid but the items you listed above are either true of nature or empty beliefs – which are exactly the types of beliefs that your orthodoxy should enforce.

      • vV_Vv says:

        Catholicism does have a few beliefs that absolutely do have real-world implications: for instance, that it’s a sin to lend at interest; that the death penalty is required by divine justice; that any form of contraception is opposed to the natural law; that one may never break the moral law even to save lives; that error has no rights; that baptism is absolutely required for salvation

        @Nick contested some of these things above, but even for the uncontested ones, note that these are not considered dogma. You can be a Catholic, even a Catholic priest, and publicly disagree with any of them without being excommunicated or even particularly socially shamed.

        Catholicism has various level of authoritativeness for its beliefs: from acceptable belief to teaching to article of faith (dogma). Progressivism, on the other hand, possibly due to Puritan Protestant influences, tends to elevate all its beliefs to unquestionable dogma.

        • sconn says:

          You can *now.* You couldn’t in 1400. I don’t really feel like hunting through the historical record for everyone who got in trouble with the church for lending at interest, or every witch who was burned at the stake for providing a woman with contraceptive herbs, but I’m sure you could take a look and easily find this stuff.

          Meanwhie I know three different Catholic internet personalities who have lost their jobs in the past year for posting stuff which, while technically not heretical, angered the Catholic organizations they worked for. It’s not burning at the stake, and it’s not official teaching, but it still happens.

          • You couldn’t in 1400. I don’t really feel like hunting through the historical record for everyone who got in trouble with the church for lending at interest, or every witch who was burned at the stake for providing a woman with contraceptive herbs, but I’m sure you could take a look and easily find this stuff.

            I’m not–why don’t you go look and report back to us? Witchcraft trials are mostly a post-medieval phenomenon, starting around the early 17th century as best I recall.

            What’s the evidence for contraceptive herbs in 1400?

            Lending at interest was indeed forbidden, but medieval lawyers had a variety of ways of evading the prohibition, at least one of which was borrowed from Islamic lawyers facing the same problem.

        • Deiseach says:

          I don’t really feel like hunting through the historical record for …every witch who was burned at the stake for providing a woman with contraceptive herbs

          Because you won’t find one, or at least not for that particular “offence”. Pop culture Burning Times 9 Million Nature Healers Mystic Earth Mothers Martyrs crap, plus the progressive notion that the only sin in existence is to express any kind of demurral from “everyone should be having as much consequence-free sex in as many combinations as possible!”, has resulted in this muddled thinking that (a) all that local churches did for a thousand or more years was burn witches (b) only women were burned (c) they were only burned because they were free-thinking heirs of the old liberated pagan traditions that wanted to help other women free themselves from the chains of the patriarchy by using their advanced medical knowledge (herbal remedies being every bit as good as modern medicine, doncha know) and alternate methods of accessing cosmic spirituality (getting high) (d) let’s all sing about the Golden Age of Matriarchy!

          The fact that most witchcraft accusations came from neighbours and within the community, and was not stern-faced clergy sniffing out abortionists, is conveniently forgotten for the political point.

  38. Jayson Virissimo says:

    Did Giordano Bruno die for his astronomical discoveries or his atheism? False dichotomy: you can’t have a mind that questions the stars but never thinks to question the Bible.

    Adopting Copernican doctrine for Neoplatonic reasons hardly counts as an astronomical discovery, nor is pantheism equivalent to atheism.

  39. meh says:

    I remember seeing a segment about Bruno in the new Cosmos series. They portrayed his view of an infinite universe as being factually correct, but as being divinely inspired. It was odd, and seemed counter to the point they were trying to make.

  40. Edward Scizorhands says:

    This is the weirdest analogy for NeoGAF but somehow extremely appropriate.

  41. James Miller says:

    What’s the value of a few people in respectable positions speaking out? Let’s say that none of these people are world class thinkers so the social cost of them being squashed isn’t too high. Plus, let’s add that these people have tenure which according to the official rules protect one against being fired for one’s beliefs. Finally, imagine that these rebels are on the autism spectrum and so care far less about being socially scorned than typical people do.

    • romeostevens says:

      More tenured professors going the way of Jordan Peterson sounds net good. Keep in mind one of his core points is that the people who flipped out at him are a paper tiger. Demonstrating that creates common knowledge.

      • reasoned argumentation says:

        Here’s the thing to keep in mind with this post though – the host doesn’t want it demonstrated that the current anti-heresy forces are paper tigers (in the sense that they have significantly more power if you try to placate them than if you ridicule them) in a lot of cases – he’s specifically used this:

        https://imgur.com/r/energy/up6yu

        to make the point. He believes that even if a bunch of present orthodoxy is false (and even if he’d like the right to subtly question it) that it’s still good. This post is a way of encouraging an increase in the power of orthodoxy enforcement by way of discouraging open defiance of the orthodoxy.

        • cuke says:

          You believe that Scott is writing this post to strategically and nefariously undermine dissent? I’m trying to wrap my head around that idea and why you would keep coming here if you think he’s not communicating in good faith.

  42. Skivverus says:

    I’m reminded of the logic puzzle with eye colors.
    You know, the one where no one can see their own, everyone can see other people’s, everyone wants to find out their own so they can [insert action here], and where, assuming perfect rationality on the part of the participants, everyone does nothing for a while, followed by everyone with the relevant eye color doing [insert action here] at once.

    I suspect a similar mechanism is at work when it comes to culture and bias. No one can see their own (status/bias), everyone can see other people’s, everyone wants to find out their own, and changing (culture/minds) tends to not go anywhere for a while and then shift relatively abruptly.

  43. Irenist says:

    Two issues are conflated here:
    1. Censorship stifles free inquiry.
    2. Hypocrisy about censorship makes it hard for the less savvy to know what’s off limits.

    (1) was certainly present in medieval and Renaissance Christendom. But this post is about (2), and that was largely absent.

    (2) was absent because the Church WASN’T hypocritical about heresy. It was proud it burned heretics. Unfettered free speech wasn’t a hypocritically praised value like among SJWs today–it wasn’t a medieval value at all. (The medieval university is best thought of as an oasis of LESS fettered free inquiry, not unfettered.)

    The Church was very explicit on what was orthodoxy and what was heresy. They had ecumenical councils and an Inquisition about it–both with published proceedings. And an Index of Forbidden Books. They even told Galileo exactly what phrases to use if he wanted to pursue his research in peace. ‘Just call it “hypothetical” and we’ll leave you alone.’ That was the S.O.P. of the Inquisition: “Here’s your heresy; here’s *exactly* the verbal formula you need to say to recant it and be set free.”

    Christendom had many, many failures. And this post is about a very interestingly subtle failure mode. Unfortunately for the value of the post, though, Christendom didn’t have this particular failure. Just lots of other ones.

    • Douglas Knight says:

      If the story of Galileo is so simple, why can’t Inquisition apologists keep their story straight?

      • Nick says:

        The story of Galileo isn’t simple. People write whole books on the Galileo controversy. Is anyone actually disputing that? I grant that there’s misinformation both ways, though.

      • Anonymous says:

        Because history is curved. We’ve long passed the point at which Galileo disappears beneath the horizon, and is interred into mythology, with randomized memetic embellishments.

        (Not that the Galileo story is simple, like Nick explains above. The bare bones, here, is of considerable length.)

    • quanta413 says:

      Unfettered free speech wasn’t a hypocritically praised value like among SJWs today–it wasn’t a medieval value at all.

      I don’t think most SJWs praise unfettered free speech. I think they are usually pretty honest about making exceptions to it. I might not love everything about the social justice movement, but I think that the activists in it are usually pretty forthright about what things they view as unacceptable.

      • Sniffnoy says:

        Depends who you’re talking about. The core are as you say and do not. The periphery, the liberals who were lured in by the implicit promise that it’s just an extension of liberalism, liberalism done right, and have not yet realized that it’s actually fundamentally illiberal, do.

  44. John Schilling says:

    False dichotomy: you can’t have a mind that questions the stars but never thinks to question the Bible.

    Newton certainly questioned the stars, but his questioning of the bible seems to have been sincerely limited to “how can I find even more hidden truth in this awesomely truthful document!”

    I’ve got a cousin who is an actual Young Earth Creationist, with a successful scientific career. In a geology-adjacent field, even (soil hydrology).

    And we’ve had a recent discussion on the extent to which people, even the clever rationalists here, can and do compartmentalize beliefs when matters of religion or politics get in the way of things they have to get right for scientific or otherwise professional reasons.

    The best you can do is have a Bruno who questions both, but is savvy enough to know which questions he can get away with saying out loud. And the real Bruno wasn’t that savvy.

    A distinction without a difference, perhaps. But, looking at the very long list of great scientists who nobody seems to have noticed questioning the religious or political orthodoxies of the societies they worked in, I think you may be greatly overestimating the extent to which a scientific mindset necessarily turns people into fearless/dangerous edgelords unless carefully guided otherwise.

    • Randy M says:

      Newton certainly questioned the stars, but his questioning of the bible seems to have been sincerely limited to “how can I find even more hidden truth in this awesomely truthful document!”

      Juxtaposed like that, it kind of makes sense. Newton examined the motions of the heavens and determined gravitational equations. Ultimately this is reducing the complexity of seemingly random or at least anomalous
      observations down to mathematics, but it is also for hidden patterns that others overlook. I think it might not have been that wild a leap for him to go into prophecy and numerology.

    • Deiseach says:

      The best you can do is have a Bruno who questions both, but is savvy enough to know which questions he can get away with saying out loud. And the real Bruno wasn’t that savvy.

      Is it really much of an improvement to have someone who questions the stars and questions the Bible because he believes the Earth is ensouled and intelligent? Bruno was happy to use the Copernican system as a tool to elucidate his philosophy, but he was much more interested in Hermeticism and metaphysics than the physical cosmology of the solar system. He would not have much to discuss with deGrasse Tyson, alas!

      Review of a translation of Bruno’s The Ash Wednesday Supper in which he sets forth his ideas and expounds on the Copernican theory:

      In the Preface to her influential Giordano Bruno and the Hermetic tradition, Frances A. Yates tells how she once planned to make an English translation of La cena de le ceneri with an introduction emphasising the boldness with which Bruno accepted the Copernican theory. But as she read his works more carefully she began to marvel at the nature of his astronomy and her eventual conclusion was that Bruno, far from being the herald of the new scientific age, was the last of the great Renaissance Hermeticists.

      From “The Ash Wednesday Supper”:

      Consider, then, that as the male moves to the female, and the female to the male, so every planet and every animal moves, more or less clearly, towards the Sun and the other stars as to their vital principle. The magnet moves to the iron, the straw to the amber, and in sum, everything seeks its like and flees its contrary. This happens through the sufficient internal principle which makes it move, and not from an external principle as we see in those bodies that are moved against or beyond their own nature. Therefore the Earth and the other stars move, according to their peculiar local differences, in virtue of an intrinsic principle which is their own soul.

      “Do you think”, said Nundinio, “that this soul is sensitive?” “Not only sensitive”, answered the Nolan*, “but also intelligent, and not only intelligent like our soul but perhaps even more so”.

      i.e. Bruno himself, as he came from Nola.

    • Scott Alexander says:

      I think in a more repressive age, Newton’s Biblical research absolutely could have gotten him in trouble, the same way Bacon got in trouble for looking into “prophcies”

      • Douglas Knight says:

        He kept his mouth shut, so why do you think it was safe in his time?

        Also, he became a definite Arian heretic, and his refusal to take holy orders did get him into mild trouble. It would have been much worse if he hadn’t endorsed the 39 articles before he became a heretic.

      • Le Maistre Chat says:

        It DID. He chose not to take Anglican holy orders at real cost to his career because he secretly sincerely believed in Arianism.
        Another data point on scientific genius being orthogonal to heresy: Gottfried Wilhelm Leibniz was at least as great a mathematician and theoretical physicist, while being an orthodox Lutheran who cared about reunification of the Church.

        • Nick says:

          Nitpick: Leibniz very much cared about the reunification of the Church and was willing to go to bat for both Catholics and Lutherans on theological matters, but we don’t really know how orthodox he was. His philosophical beliefs were certainly really out there, and he had an affinity for the sort of mechanistic philosophy that’s hard to square with mainstream Christianity. There’s a great essay on Leibniz’s work to reunify the Church that discusses his religious beliefs at the beginning:

          The philosopher seemed not to have been a pious Protestant no more than a fervent Catholic. One cannot, however, consider him as an atheist, though he hardly ever visited the church. He told his assistant that he was a priest of natural rights and noted that that was all he found in the Bible. According to many rumors Leibniz refused blessings in his deathbed. These facts and rumors earned the philosopher a bad reputation and in his last years in Hanover he was very unpopular.

          After reading Leibniz’s writings (The Monadology in particular) one cannot help thinking that Leibniz was religious, but in his own way – his conception of God is perhaps closer to platonism, the mystical theology of eastern religions or pantheism than the doctrines of Christianity. Leibniz believed that in every religion there is a mutual seed of truth. It seems that Leibniz had his own, philosophical version of religion, which is a combination of Protestantism and Catholicism. Leibniz was deeply religious, notes Jordan, but hated dogmatism and the short-sightness of clergy and kept himself neutral because of that. According to Leroy Loemker, Leibniz thought that his philosophy is an argument on behalf of Christianity.

          • Le Maistre Chat says:

            Ah, that essay complicates things. I thought he was simply a “I believe in Christianity for philosophical reasons, belong to X denomination, and am sad that there are such things as denominations” type.

        • The original Mr. X says:

          Given that Newton became a Professor of Mathematics anyway, I’m not sure his declining to take holy orders can really be said to have hindered his career..

      • John Schilling says:

        I think in a more repressive age, Newton’s Biblical research absolutely could have gotten him in trouble, the same way Bacon got in trouble for looking into “prophcies”

        Newton’s Biblical research was divided into two categories: Stuff that was pretty unambiguously safe to publish in any era, e.g. “Knock it off with the apocalyptic prophecies; here’s mathematical proof that the Anglican church will be administering God’s affairs in England for at least another four centuries!”, and stuff he carefully didn’t publish until he was safely dead.

        I do not believe this is coincidence. Whatever era you put Newton in, however repressive, the stuff he publishes in his lifetime will be the stuff that doesn’t get him in too much trouble. Just like most of the scientists working in the more repressive eras limited their published work to stuff that wouldn’t get them in too much trouble.

        Being a capable and motivated scientist does not mean being an irrepressible edgelord who can’t not speak forbidden truths unless carefully guided by a whispering minder. That describes a tiny fraction of history’s scientists. And probably a similar fraction of history’s non-scientists.

      • Deiseach says:

        Roger Bacon got himself into the middle of a nasty fight between two factions of the Franciscans, the Spirituals and the Conventuals, about the direction of the order after the death of St Francis. Part of that were the prophecies mentioned, which probably relate to Joachim of Flora and the Third Age of the Holy Spirit and the upcoming end of the world and spiritual kingdom.

        If you’re going to praise Bacon’s scientific open-mindedness on the basis of “looking into prophecies” (and he wasn’t investigating them to debunk them, he believed and promulgated them), then you will have to do the same for the Heaven’s Gate cult or the latest guy who is calculating the Apocalypse is going to happen on 12th November this year from his interpretation of the Book of Daniel.

        I believe Dan Brown has a new book out, another volume in his tireless questioning of orthodoxy and the secrets the Church is trying to cover up from us!

        • Nick says:

          I believe Dan Brown has a new book out, another volume in his tireless questioning of orthodoxy and the secrets the Church is trying to cover up from us!

          Yeah, I saw it advertised somewhere the other week. Origin is the name, I think. I’m not going to read it, personally; everything’s been downhill for us since we lost our albino assassin monks. 🙁

      • Steve Sailer says:

        Newton kept lots of his ideas, religious and scientific, secret for a long time. One thing to keep in mind is that secrecy back then wasn’t always just defensive, it was also offensive: there wasn’t a tradition of publishing discoveries, because you might want to keep them secret so you can have a monopoly on resulting technology.

        A third reason to hold back your discoveries would be to get them into shape so they wouldn’t be controversial when you revealed them. Newton had most of the Principia worked out many years before (although not quite all of it). When he finally did publish his big book, it was almost instantly accepted by everybody who mattered. It was triumphantly persuasive, in part because Newton took pains to format it in such a way that the top few hundred people in Europe would understand immediately. That took a fair amount of effort, which Newton wasn’t always willing to take away from his other pursuits.

        Here’s John Maynard Keynes’ interesting essay on what he discovered about Newton by buying at auction a trunk of Newton’s unpublished notes:

        http://www-groups.dcs.st-and.ac.uk/history/Extras/Keynes_Newton.html

    • vV_Vv says:

      I’ve got a cousin who is an actual Young Earth Creationist, with a successful scientific career. In a geology-adjacent field, even (soil hydrology).

      But Young Earth Creationism is not enforced by the authorities. The problem is not so much having objectively wrong beliefs (although they can harm if you don’t compartmentalize them properly), but in the censorious environment which discourages truth-seeking in general.

  45. Eli says:

    And the better a school or professor is, the better they train their students to question everything and really try to understand the natural world, the more likely their students will speak up about the lightning issue. The government will make demands – close down the offending schools, fire the offending academics. Good teachers will be systematically removed from the teaching profession; bad teachers will be systematically promoted. Any educational method that successfully instills curiosity and the scientific spirit will become too dangerous to touch; any that encourage rote repetition of approved truths will get the stamp of approval.

    Some other beliefs will be found to correlate heavily with lightning-heresy. Maybe atheists are more often lightning-heretics; maybe believers in global warming are too. The enemies of these groups will have a new cudgel to beat them with, “If you believers in global warming are so smart and scientific, how come so many of you believe in lightning, huh?” Even the savvy Kolmogorovs within the global warming community will be forced to admit that their theory just seems to attract uniquely crappy people. It won’t be very convincing. Any position correlated with being truth-seeking and intelligent will be always on the retreat, having to forever apologize that so many members of their movement screw up the lightning question so badly.

    And this is why societies which use authoritarian means to maintain orthodoxies usually rot and die over the long term, while open societies based on personal rights, social rights, and the autonomy of the epistemic sphere tend to flourish. Only the latter avoid the need to purge anyone who gets too clever and knowledgeable about the real world instead of about politics.

    I had thought we knew this stuff. It’s really basic stuff.

    Such a whisper network would be in the best interests of the orthodox authorities. Instead of having to waste their good scientists, they could let the good scientists could join the whisper network, learn which topics to avoid, and do good science without stepping on orthodox toes. But the authorities couldn’t just say this. Maybe they wouldn’t even think of it, and nobody (except maybe Kantorovich) would be dumb enough to try to tell them. Individual secret policemen are always going to see the written law – “arrest heretics” – and consider the whisper network a legitimate target. Kolmogorov is doing the Lord’s work, but that won’t give him a pass from the Inquisition.

    Sounds like 1984’s “Brotherhood”, or Warhammer 40K’s Radical Inquisitors.

  46. Machine Interface says:

    This has been pointed out I think, but to drive the point home, it seems like an error to equate “obsessional truth seeking” and “extreme curiosity” with “scientific virtue”. Being obsessed with something at the expense of everything else is usually unhealthy, even if that thing is truth, and being obsessed with finding the truth doesn’t actually mean you’re good at finding the truth.

    On the contrary, I’d wager that, out of 10 truth-obsessed people, maybe 1, generously, will be a great scientist. The 9 other will be conspiracy theorists, crackpots and gurus. When you’re obsessed with finding the truth, it’s easy to assume that the truth is hidden, that everyone is deluded, and from there that the truth is *being kept* kidden, that everyone is *actively being* deluded.

    As someone else pointed out, Bruno is an unintentionally good example of this: his scientific, astronomical work was not his own, but was in fact plagiarized (and poorly understood) from Nicholas of Cusa, who was never “punished” for his beliefs, and in fact died as the vicar general of the Papal States. Which means Bruno’s curiosity *only* lead to mystical nonsense. The Church in his case didn’t execute someone who happened to be a scientist, for reasons orthogonal to his science. They executed someone who was 100% a crackpot.

    In a more general manner, what society doesn’t have some manner of orthodoxy? Even hypothetical, ideal societies have some orthodoxy; even micro-societies like say, the rationalist community, have their orthodoxy — the vast majority of the rationalists seem to take moral and aesthetic realism for granted, they know that “good” and “beautiful” are totally real and objective features of reality, and if we are reason sufficiently hard enough about them we will totally be able to determine what exactly they are! We can at least assure with very high certainty that truth is good, curiosity is a virtue, and freedom of speech is as sacred as it is vital!

    • 天可汗 says:

      There was a wacky ultra-right New Age type on Tumblr ages ago who’d post word salad about Agharta or whatever, and right when everyone started figuring he was trolling he was like “Aryans are immune to AIDS” and everyone was all “haha ok, troll” so he was like “no, really, there’s a mutation that confers immunity to HIV and it’s estimated that it’s most common in the Nordic countries” and I remembered that a year or so later and looked it up and it at least seemed from five minutes of reading to be in accord with the scientific consensus of the time.

      And I know alt-med types who responded to contracting some disorder or other that their doctor told them nothing could be done about by sitting down, doing some alt-med research, coming up with something they figured would work, and actually, verifiably curing themselves.

    • Tracy W says:

      But if you’re an extreme truth seeker, at some point you’ll probably start thinking things like “How do I know everyone is deluded? How do I know things are being kept hidden? How can I test this?”

      Or, at least, if someone asks you these questions, then you’ll start querying yourself too. John Nash reportedly even doubted his delusions, which strikes me as a case of dedicated truth seeking.

      Crackpots and conspiracy theorists often seem to lack that self-doubt aspect of truth seeking.

    • cuke says:

      I wonder if the big five personality attribute of “openness” might be a corrective for the “obsessional” part of truth seeking. It seems to me there’s a difference between diligence/doggedness and epistemic closure.

      I further wonder whether a person’s level of emotional reactivity is the major contributing factor to whether they can pursue “truth” in a rigorous but open-to-dissenting-views way or whether they cannot tolerate the emotional experience of dissenting views which then sends them deeper into epistemic closure. I suspect “hot-headed” folks may be more prone to “my way or the highway” kind of thinking, while it’s possible cooler minds can pursue a curiosity in a very persistent and curious way while still remaining open to multiple outcomes. Or another way to say this is that anxiety and fear impede learning, so a person who is driven more by fear than curiosity in their pursuit of truth is less likely to find truth.

      This is just random theorizing without any particular reference to reality. Or maybe except the reality of my observing patients who I work with on these kinds of issues for their mental health.

  47. John Schilling says:

    But politically-savvy Kolmogorov types can’t just build a bubble. They have to build a whisper network.

    While this might be useful, I don’t think it is at all necessary. Useful in that “our” whisper network will be optimized for the things we desire, like providing a covert channel for discussions of forbidden subjects. But not necessary for the minimal task of ensuring that people stay clear of things that will get them fired or burned at the stake while leaving them free to pursue other intellectual pursuits. For that purpose, the enemy will build an adequate early-warning network for us, and it won’t have to whisper. Because this…

    The Church didn’t lift a finger against science. It just accidentally created a honeytrap that attracted and destroyed scientifically curious people.

    …is I think 90% wrong. The Church’s “accidental honey trap”, and Stalin’s, attracted and destroyed a very tiny minority of scientifically curious people. As do our society’s honey traps. The Church, and Stalin and Social Justice and all the rest, don’t carefully refrain from criticizing or correcting WrongThink until someone has crossed the hidden line and must then be irrevocably purged in some unpleasant way. They incorporate plenty of nonlethal warnings, which suffice to give almost everyone fair notice of what can and cannot be safely said and to do so without destroying people’s lives. Why would the Church want to destroy the life of a perfectly good scientist who could figure out how gunpowder works and so better destroy the heathens, when they can simply correct him instead?

    So, yes, you can focus on the relative handful who didn’t get the message and blundered into something really ugly. But let’s take another contemporary example. The dismal quality of the romantic advice and guidance given to the nerdishly inclined in our society has been discussed here at length before, as has the involuntary celibacy that often results. And yet we very rarely actually see people being arrested because they didn’t know the true secret rules and believed the fake rules instead. What we see, are people who figure out from warnings that come well before “arrested for rape; your life is now over” that this is a subject that is not safe for them to deal with and they should instead go do great work in e.g. quantum computing.

    The Church had Galileo et al, Stalin had Kantorovich, Google had Damore, and the way they treated those people was wrong. But that still leaves an awful lot of people who were “merely” silenced in a few specific areas and went on to e.g. develop almost all of the science and technology underlying the modern world, and I think mostly without the help of anyone’s whisper network.

    • Deiseach says:

      It just accidentally created a honeytrap that attracted and destroyed scientifically curious people.

      I’m going to dispute that they were scientifically curious people. The ones listed got into trouble for heresy and occultism and a lot of stuff that nowadays would be considered “woo” and would get them laughed out of many science-adjacent spaces if they went on there. Suppose Michael Servetus tried persuading the people over at Less Wrong 2.0 that God is a unity, not a trinity. How would he be received? What would the response be to him? “Gosh, what a scientifically-minded thinker” or “You are basing your theory on exploded notions”?

      Servetus didn’t get into a fight with Calvin over the circulation of the blood, it was over the Trinity and the doctrine of predestination. Servetus could have been an atheist, but he wasn’t; he was every bit as much a religious believer as any Unitarian (of the original crop) or else he wouldn’t have bothered corresponding with Calvin. Nobody gets run out of town for thinking in the privacy of their own mind that standing for the anthem and saluting the flag is stupid primitive nationalist nonsense, but they go along with the crowd for the sake of peace. It’s the guys protesting over it that stir up the heated feelings.

      There were plenty of scientifically-curious people who didn’t get into trouble with the Church, and plenty of religiously zealous believers who were as unhappy with the new trend of science (Luther wasn’t any too big a fan of heliocentrism either) who did. It’s too glib to make the equation that “got into trouble with the Church = scientifically curious, proto- or actual scientist, possible non-believer or atheist”.

      • 天可汗 says:

        One of the few good things about Neal Stephenson’s Baroque Cycle is that it does a good job of conveying how non-obvious it was that certain things would pay off and certain other things wouldn’t.

        And then he goes and undermines the point by downplaying Newton’s wacky religious speculation. At least in what I’ve read so far.

        • Nick says:

          One of the worst sins of historical fiction is treating its characters as basically just moderns. I have to give Stephenson a partial pass on this, since it was the very time and the very people through which our modern world was taking shape, but I still thought they had a little too easy a time accepting mechanistic, even atheistic ideas—Daniel being the most obvious example to my mind.

          • 天可汗 says:

            All his books after Snow Crash are about how much he likes MACTs — maybe theology is just too East Coast for him.

          • reasoned argumentation says:

            There’s no real subtext there though – by the end of the trilogy Daniel confesses to being a literal atheist.

            It’s perfectly in line with actual historical development (if compressed) put into a single character. Going back before Waterhouse’s life – the Anglican Church was supposed to be just like the Latin Church but with an Archbishop appointed by the King. Unfortunately that turned out to not be a Schelling point so by Waterhouse’s time another had grown up – the Anglican Church wasn’t living up to doctrine and the superior holiness of the dissenters was justification for them to rule (an Anglican goes to a dissenter service with Waterhouse at one point and remarks that it’s all politics and Waterhouse replies it just sounds like religion to him). That wasn’t a Schelling point either though because once you open the door to religious entrepreneurship the dissenter church gets outflanked by men who are even holier than Jesus and God – at this point they’re atheists who are only sure of one thing – that they’re morally superior to you and that that moral superiority is why they should rule your country.

          • Nick says:

            That wasn’t a Schelling point either though because once you open the door to religious entrepreneurship the dissenter church gets outflanked by men who are even holier than Jesus and God – at this point they’re atheists who are only sure of one thing – that they’re morally superior to you and that that moral superiority is why they should rule your country.

            You had an interesting thesis above, but I don’t get this bit; are you saying Daniel considers himself one of these holier-than-thou I-deserve-to-rule atheists? Because that doesn’t sound like him at all. Actually, it doesn’t sound like any of the characters.

          • reasoned argumentation says:

            Stephenson writes about Puritans lots of times. Waterhouse and his clan don’t evolve in this direction but others do.

          • Nornagest says:

            A lot of Stephenson’s villains tend to be holier-than-thou types with odd, idiosyncratic belief systems — not necessarily Puritan as such, but sharing some of the same mentality. Andrew Loeb in Cryptonomicon might be the purest example.

      • Saint Fiasco says:

        Believing in “woo” is just what scientific curiosity looked like back then. They are only seen as stupid now because a lot of people looked into it.

        Think Newton and the “shoulder of giants” thing. Newton himself believed a lot of woo things like numerology and alchemy, because those things hadn’t been debunked yet by a bunch of giants.

        • Anonymous says:

          Your alt-identities would be harder to spot if you ceased to use that unique shibboleth.

          • Saint Fiasco says:

            My alt is shnissugah, but I lost that password.

          • Anonymous says:

            You mean you’re not the only other person here who uses “woo” as an euphemism for nonsense?

          • qwints says:

            It’s a widely done thing in the skeptic community.

          • suntzuanime says:

            It’s not that unique is it? It’s associated with an unpleasant sort of pseudo-intellectual who wants to ridicule more than understand, but there are more than a few of those out there, it’s not one local individual’s coinage. It comes from the “skeptic” community, I think, the RationalWiki type fellows.

          • Anonymous says:

            I stand possibly corrected.

          • Nick says:

            I’m guessing Anonymous has HFARationalist in mind, who used it a lot and was recently banned. It would certainly be a sign it’s him, but it’s not nearly as conclusive as Sidles’ Spinoza stuff or “ratiocination.”

          • Saint Fiasco says:

            I literally put woo in human quote tags to make clear that I was quoting Deiseach. Maybe I should have put it in blockquote tags instead.

          • Anonymous says:

            I retract my accusation, and apologize.

        • Deiseach says:

          So when are you going to rehabilitate as Real Scientifically Curious Seekers Doing Real Science Not Dead-End Metaphysics all the alchemists who didn’t have Newton’s good fortune in producing useful mathematical works?

      • John Schilling says:

        I’m going to dispute that they were scientifically curious people. The ones listed got into trouble for heresy and occultism and a lot of stuff that nowadays would be considered “woo” and would get them laughed out of many science-adjacent spaces if they went on there.

        Right, but that’s only because we’ve got an extra few centuries of evidence to firm up the walls between “science” and “woo”. Even Newton’s alchemy and biblical numerology were, at the time, perfectly reasonable things for a scientist to look into.

        As to the broader point, the Catholic “honeytrap” caught people for reasons mostly orthogonal to scientific curiosity, but some people are scientifically curious and so some of them got caught. And those are the ones we remember, at least in contexts like this.

    • vV_Vv says:

      The Church’s “accidental honey trap”, and Stalin’s, attracted and destroyed a very tiny minority of scientifically curious people. As do our society’s honey traps. The Church, and Stalin and Social Justice and all the rest,

      Is this true? After the high profile trials of Galileo and the other science-minded heretics, scientific research pretty much died in Italy and in all the other Catholic countries, with the exception of the highly anti-clerical revolutionary France. I don’t think it ever fully recovered.

      As for Soviet science, sure, good math and rocket science, but severely underdeveloped in many other fields, not just in genetics where you would expect it given Lysenkoism.

      The damages of socjus are yet to be seen. My guess is that the scientific and technological frontier will shift to Asia, while Europe and America will become gradually more backward.

      • The original Mr. X says:

        Plenty of the people on this list seem to postdate Galileo.

      • theofloinn says:

        After the high profile trials of Galileo and the other science-minded heretics, scientific research pretty much died in Italy and in all the other Catholic countries

        Which other ones?
        What heresy?

        Late 1632. Leander Bandtius, Abbot of Dunisburgh, (and owner of a particularly fine telescope) notes a large red spot on Jupiter.

        1636. In Harmonie Universelle, Fr. Marin Mersenne diagrams the construction of reflecting telescopes in configurations similar to the Gregorian and Cassegrain telescopes.

        1637. Galileo Galilei publishes Dialogues Concerning Two New Sciences in Holland with the complicity of Cardinal Piccolomini (in whose palace he had begun work on the draft) The two sciences are strength of materials, in which he describes the square-cube law, and the physics of motion, in which he confirms De Soto’s law and Bradwardine’s medieval observations. He even uses Nicole d’Oresme’s graphical proof of the Mean Speed Theorem. Without attribution, of course.

        1640. Jesuits Francesco Maria Grimaldi and Giovanni Battista Riccioli drop weights from the Torre di Asinelli in Bologna and time the fall using a pendulum. From this they calculates the acceleration due to gravity as g = 9.144 m/s². (The modern value is 9.80665 m/s².) Both men have craters named for them on the Moon for the excellent reason that they were the ones who named the lunar craters: one sector centers on Ptolemy, one on Copernicus, and a third on Tycho.

        12 Mar 1641. In a letter dated, March 13, 1641, Vincenzo Renieri, a professor at the University of Pisa, reports to Galileo on experiments he conducted the day before in which he dropped balls from the Tower of Pisa. Renieri is an Olivetan monk, an order co-founded by one of the Piccolominis of Siena, and it was at Archbishop Piccolomini’s palace that Renieri met Galileo (1633). When Galileo dies next year, he will leave all his unfinished scientific work for Renieri to complete; but Renieri himself will die shortly after (1647). Galileo’s friend and biographer, Vincenzo Viviani, also a friend of Renieri, will ascribe the Tower of Pisa experiment to his master, starting a legend that lives to this day.

        1651. Riccioli publishes his masterwork Almagestum novum. In one section, he presents both major theories — Copernican and Tychonic — and gives arguments for and against each one:

        49 arguments in favor of Copernicanism, with rebuttals to each, and
        77 arguments against Copernicanism, with rebuttals to each. There are two for which he can find no credible rebuttals: the apparent absence of stellar parallax and the apparent lack of Coriolis effects.

        This is the book Galileo was supposed to write, weighing the pros and cons of each model.

        1665. Riccioli publishes Astronomia Reformata (Reformed Astronomy), a condensed and updated version of the New Almagest. It incorporates Keplerian ellipses into the Tychonic model. It includes reports on Bandtius’ observation of the Great Red Spot, on the Jovian cloud belts disappearing and reappearing, on the appearance of Saturn’s rings from time to time. (Cassini was Riccioli’s student.)

        1687. Newton presents his theory of Universal Gravitation. It’s hard for the Late Modern to grasp what a stunning achievement this is. Suddenly, everything makes sense! He does not use calculus to do this. The Principia is carefully structured in correct Aristotelian form, with axioms and deductive logic, to ensure that it is true scientia. There is one elegant solution to all the planets, to all the motions! (Copernicus had solved each planet sepatately; hence, each orbited a different center.) Kepler’s laws can be deduced from the principle. Finally! A simple, elegant reason why Kepler’s model ought to be true!

        Just one problem; or rather two:
        • There is still no obsrvable @#^$% parallax.
        • There is still no observable *#^%$ Coriolis effect.

        1728. Attempting to detect that old bugaboo, parallax, James Bradley detects instead stellar aberration in γ-Draconis (Phil. Trans. Royal Soc., 1729). The effect is small, and detectable only with special instruments, but it counts as a proof that the Earth is moving.

        1744. After Bradley’s paper is translated into Italian, a “corrected” copy of Galileo’s Dialogue is printed in Italy. Not a word is changed, but the term “if” is inserted in various marginal topic headers. This would have been all that was necessary had the original recommendation of the Extensor been followed in the Galileo trial.

        1758. Copernicanism is removed from the Index.

        Jun-Sep, 1791. In a series of experiments, Giovanni Guglielmini, a professor of mathematics at the University of Bologna, drops weights from the Torre dei Asinelli in Bologna — the same tower used earlier by Riccioli and Grimaldi — and finds an eastward (and southward) deflection. Concerned with windage, he repeats the experiment down the center of the spiral staircase at the Instituto della Scienze and finds a 4 mm Coriolis deflection over a 29 m drop; thus providing direct empirical evidence of the rotation of the Earth.

        1806. Giuseppi Calandrelli, director of the observatory at the Roman College publishes “Ozzervatione e riflessione sulla paralasse annua dall’alfa della Lira,” reporting parallax in α-Lyrae. This provides a simple direct observation of the revolution of the Earth.

        1820. Giuseppe Settele, astronomy professor at the Sapienza (now the University of Rome) incorporates these findings into the second volume of his Elementa di Ottica e di Astronomia, and tells his colleague, Benedetto Olivieri (who is then Commissary of the Holy Office) that this provides the empirical demonstration requested by Bellarmino back in 1616. Olivieri agrees, and convinces the Office and Pope Pius VII.

        12 Aug 1820. Although not enforced for many decaded, the injunction is now formally lifted.

        But you’ll notice that scientific work even in astronomy did not stop in Italy, where key empirical discoveries were made.

        • Steve Sailer says:

          Scientific accomplishment slowed down in Italy a generation or so after Galileo, as it sped up in other parts of Europe.

          Any suggestions why?

          • Douglas Knight says:

            Italy’s relative position in everything declined. Nothing particular about science.

            Murray: 1400-1600; 1600-1800.

          • The original Mr. X says:

            Worth pointing out, too, that the Church’s temporal power declined quite a bit during the period 1600-1800, so if Italy’s place in the scientific ranking table went down, it’s unlikely that suffocating ecclesiastical influence had much to do with it.

      • bapgwh says:

        That’s a pretty weak and I think untrue about Post Galileo Catholic Europe. Belgium, Northern Italy, the Catholic states and cantons of Germany and Switzerland, Austria, etc. Still contributed greatly to science. Since you specifically called out Revolutionary France as being anticlerical, lets take a look at Frenchmen after Galileo and begore the Revolution, Antoine Lavosier is considered by many to be the greatest chemist of all time and he was part of pre-revolutionary France, was a Catholic,and even died due to his associations with the nobility during the Revolution. Blaise Pascal was a Catholic Polymath as well as Descartes, all great French Catholic Scientists after Galileo and before the Revolution.

      • vV_Vv says:

        After the high profile trials of Galileo and the other science-minded heretics, scientific research pretty much died in Italy and in all the other Catholic countries, with the exception of the highly anti-clerical revolutionary France. I don’t think it ever fully recovered.

        I’m retracting my claim.

        I think it’s more appropriate to claim that the quality and quantity of scientific research output decreased in Italy and other Catholic countries, but I’m not sure if there was an explicit causal link with the Church’s censorship activities.

    • B_Rat says:

      The Church didn’t lift a finger against science. It just accidentally created a honeytrap that attracted and destroyed scientifically curious people.

      …is I think 90% wrong. The Church’s “accidental honey trap”, and Stalin’s, attracted and destroyed a very tiny minority of scientifically curious people.

      Well, the Church one sure, so far I can identify maybe 4 guys in about twenty centuries. Stalin on the other hand needed just a few years to straight up eradicate entire fields of research, i.e. genetics and a good chunk of psychology. It was no trap, but the genuine goal. I wonder why most seem to miss this.

  48. Deiseach says:

    Let’s say they’re all true, let’s accept every excuse we’re given and accept the Church never burned anybody just for researching science.

    You didn’t really expect me to let this one go, did you?

    This is part of what pisses me the hell off (to use the cool, rational, technical term for my reaction) about the whole “The Church hates Science, boo hiss” attitude when recounting how the Inquisition Burned Scientists Bedtime Tales for New Young Atheists. No, the Church hates heresy, boo hiss. But (hardly anybody) nobody nowadays cares about heresy, everybody knows that religion is either bunk or Vague Therapeutic Deity Presence. So that can’t have been the “real” reason for burning “heretics”, ergo they must all have been free-thinking scientists!

    Imagine you are a member of the Well-Regarded Respectable Institute For Really Good Physics, and you guys have just voted to throw out a kook who has tried to sell you all on a perpetual motion machine that runs on room temperature cold fusion. A friend of a friend comes up to you at a party and asks “So, what’s the story with Maverick Thinker and why did he get the boot?”

    You explain why and what.

    “Yeah, but come on”, the other person persists. “What’s the real story? Why was he really kicked out?

    “Because his so-called theory violated the laws of physics! He was a nutjob!”

    “Oh, nobody believes in those ‘laws’, right? We all know that. You can trust me – what’s the real reason?”

    “No, that really is the real reason and by the way, yes we do too believe in the laws of physics”.

    They wink and give you a dig in the ribs. “Ah, still spouting the official line, eh? I know how it is!”

    Then they go off and four days later there’s a big social media post that goes viral and gets picked up by Vox and Slate and all the rest of them about how Maverick Thinker was kicked out of the Well-Regarded Respectable Institute For Really Good Physics because the girlfriend of the Head Guy left him for Maverick and Head Guy wanted his revenge.

    And every time you try to explain the real reason, the person you are talking to winks and nods and goes “Sure, the party line, I realise you gotta toe it if you don’t want to be tossed out as well, but everyone knows the real reason it happened”.

    How would you feel after that?

    Giordano Bruno – oh man, if ever I wished there were such a thing as a time machine! I would love Neil deGrasse Tyson and Seth MacFarlane and Discover magazine to have their pet martyr brought forth in time. Because this is not a guy who would immediately rush to embrace astronomy and cosmology as pure true Science, this is a guy who would write something like The Secret. Would invoke “quantum” for why crystal healing works on vibrational levels. This guy would be Gwyneth Paltrow and Goop. This guy is not a proto-scientist.

    But sure, shovel me up some more of those 18th century Protestant anti-Catholic polemics and 19th century atheism about how the ostensible reason for their condemnations (and not all of them were condemned; whether Bacon, a Franciscan friar, ever was put under house arrest or for how long is now debated, and there’s nothing to say he died in any other circumstances than as an ordinary member of a Franciscan chapter house. Also I am sure Calvin would charmed to find out that Reformation Zurich is lumped in with the Catholic Church as “all the one thing” – if Servetus was dumb enough to seek entry to Zurich, which was pretty much Calvin’s own personal theocracy, after arguing over theology with Calvin and sending him a book attacking Calvin’s doctrines, that’s not the fault of the Catholics) were not the real reasons, only excuses, but the real reasons were cunning priestcraft trying to hold on to temporal power and identifying SCIENCE!!! as the enemy that would break their hold of superstition and terror over the common man.

    It’s tempting to imagine a world where Servetus, Bacon, and Galileo followed Aaronson’s advice. They pursued their work in optics, astronomy, anatomy, or whatever other subject, but were smart enough never to go near questions of religion. Maybe they would give beautiful speeches on how they had seen the grandeur of the heavens, but the true grandeur belonged to God and His faithful servant the Pope who was incidentally right about everything and extremely handsome. Maybe they would have ended up running great universities, funding other thinkers, and dying at a ripe old age.

    You want to know guys who never went near questions of religion and just got on with science?

    COPERNICUS. Niklas “My brother’s a friar, my sister’s a nun, my uncle’s a bishop, and I have a church position myself” Koppernigk, whose system Galileo “borrowed” and “improved upon” (Galileo had rather a habit of doing that; I’m old enough to remember all the claims about Galileo that have since been quietly walked back on re: inventions and discoveries that were the property of other people). Did not end up hauled in front of the Dominicans to answer tricky questions because he wasn’t every bit as busy getting involved in politics, court shenanigans, and score-settling as he was in doing real science.

    GREGOR MENDEL. Oh, hey, another Augustinian friar and abbot of his priory!

    GEORGES LEMAITRE. Catholic priest and professor of physics at the Catholic University of Louvain.

    Speaking of which, where do you think all that list of BOLD FREE-THINKERS AND CHALLENGERS OF ORTHODOXY got their educations? Who knew secular liberal arts colleges existed back then – oh wait, no, they didn’t; universities were under the patronage of the Church. The evil, science-hating Church.

    You know where you’re going to find your conventional wisdom challenging, looking for trouble free-thinkers today? Not fighting the Church, nobody bothers with that (apart from the kooks who still believe the fairy stories and those who want to bend the perceived political power to their ends).

    It’s the anti-vaxxers. The perpetual motion machine guys. The Time Cube manifesto. The “curators” of “cutting-edge wellness advice” on sites like Goop who will tell you to stick jade eggs into your vagina. They’re challenging orthodoxy all right – and the orthodoxy and power of today is Science, not the Church.

    Does that make them real scientists, so?

    • Scott Alexander says:

      I’m not sure what you’re disagreeing with. I *said* that they were burned for heresy and not for science.

      Also, Copernicus published all his work posthumously, which hardly seems like a victory for “science was super-easy and unregulated in those days”.

      • Nick says:

        It looks like Deiseach isn’t disagreeing with you, just attacking the usual Conflict Thesis. If anything, she’s just insisting that folks like Bruno dying were no loss to science anyway, which may be an implicit denial that your whisper networks would do any scientists any good, but it’s potentially a purely object-level disagreement—in other words, it may be true for scientists now, but wasn’t true back then.

      • The original Mr. X says:

        “I’m not sure why Brutus is so upset at me. After all, I *said* he was an honourable man, didn’t I?”

      • Evan Þ says:

        She’s disagreeing with (among other things) your argument that the qualities that make good scientists good necessarily incline them to question the reigning orthodoxy. To this end, she puts forward three counterexamples: Copernicus, Mendel, and Lemaitre.

        This’s an interesting counter – but, @Deiseach, I think you need better evidence. Lemaitre lived in the 1900’s, after the Catholic Church had ceased to be the reigning orthodoxy; and like Scott said, Copernicus published posthumously. To go with Mendel, I’d start with Nicholas of Cusa from whom Bruno lifted his astronomical theories?

      • theofloinn says:

        Copernicus published all his work posthumously, which hardly seems like a victory for “science was super-easy and unregulated in those days”.

        Which regulation was that?

        1. It was the ridicule of the physicists that Copernicus feared. (He had already been satirized on the stage.) He said so in his preface.
        2. He was encouraged to publish by his bishop and a cardinal friend.
        3. Publication was posthumous because he was sickly and died before it saw print.

      • B_Rat says:

        Also, Copernicus published all his work posthumously, which hardly seems like a victory for “science was super-easy and unregulated in those days”.

        Copernicus had his theory first circulate in the first half of the ‘510s with his Commentariolus. In 1533 it got explained to the very Pope Clement VII and other notables in the Vatican Gardens with the result… of a pretty gift to the one delivering the lectures (a precious Greek manuscript) and a later letter of great encouragement by Cardinal Nikolaus von Schönberg. Also, in 1537 Copernicus was one of 4 candidates for the position of bishop. The poor oppressed schmuck.

        (The problem was more about him being in need of some proof of his ideas, which in the end just didn’t came. The theory was pretty much ignored, for actual empirical reasons, until Kepler’s work in the following century.)

        • Lillian says:

          Kepler is the real hero of heliocentrism. The popular account treats him as someone who improved on Coppernicus’ model, but in truth his was an independent development inspied by the low predictive value of the Coppernican and Tychonic models with respect to the orbit of Mars. It’s his model that proved correct, and if anything it’s descended from Tycho Brahe’s own model since Kepler relied Brahe’s more accurate Rudolphine star tables instead of the Augustine/Prussian tables used by Coppernicus.

      • Deiseach says:

        I *said* that they were burned for heresy and not for science.

        And then used a finger-wagging “if we believe any excuse”. I may be going too hard on what is a turn of phrase, but as I said – how would you feel if someone represented the reason Maverick Thinker was tossed out of your research group for producing a paper on how limitless energy can be obtained by harnessing the pixies at the bottom of the garden as power sources as “well the excuse Scott told me was that he was incompetent, but I’m pretty sure the real reason was that money he borrowed from Scott and never paid back”.

        • Christopher Hazell says:

          The thing that always gets me in these discussions is that there is a hell of a wide berth between “Being thrown out of the research committee” and “Being burned alive.”

          I guess my fundamental problem, as an atheist, is that I think it is morally abhorrent to set people on fire for having the wrong ideas. Any one of those people may well have been quacks, they may have been set on fire because they were quacks, but it seems fucking insanely monstrous to me to demand that the author of “The Secret” be burned alive.

          “Setting people on fire for having the wrong ideas didn’t damage scientific progress that much in the grand scheme of things because most scientists just figured out how to avoid being set on fire” just seems both weirdly nitpicky and also a counterfactual that is impossible to prove one way or another.

          Also, lastly, a LOT of scientists do extremely good work in one field while believing utter nonsense in other areas, so I’m not totally convinced by the argument that if someone believed in something stupid, it means they didn’t have anything worthwhile to contribute to scientific progress.

          • Conrad Honcho says:

            I guess my fundamental problem, as an atheist, is that I think it is morally abhorrent to set people on fire for having the wrong ideas.

            Consider Antifa types in favor of “punching Nazis” (or worse) and alt-right types who want to give Marxists “helicopter rides.” Some of it’s hyperbole, sure, but there is no shortage of examples from social media of people on the far left and far right anticipating some kind of civil war.

            Wanting to hurt or kill other people because their ideas are so out of step with what you know the correct ideas are is not an uncommon part of human nature.

      • Deiseach says:

        Also, Copernicus published all his work posthumously

        He died in 1543, which was when his major work De revolutionibus orbium coelestium libri vi (“Six Books Concerning the Revolutions of the Heavenly Orbs”), was published, but he’d been working on getting it ready for publication so I don’t think you can take from the fact that he unfortunately did not live to see it published that he was too afraid of consequences to get it distributed in his life-time. Astronomy was a part-time job or even a hobby for him as he had full-time duties in his position as a canon.

        In 1514 he was invited to attend (but did not) the Fifth Lateran Council to advise on reforming the calendar; hardly sounds like “dutifully keeping his head down so the Higher-Ups in the hierarchy won’t hear of him because oooh, he’s a scientist and he might get in trouble for his free-thinking curiosity”!

        And if I am to believe this Encyclopaedia Britannica article, there was a tangled history behind the publication effort; he co-operated on a first ‘trial balloon’ work with a Lutheran who came to study with him, which seems to have been received well; so that when his collaborator went back home, Copernicus entrusted the manuscript of “De Revolutionibus” to him for publication but he had to turn it over to someone else to publish it. That guy made so many unauthorised changes that it was worse than leaving the work unpublished. Indeed, the Evil Science-Hating Church Hierarchs so despised this kind of unorthodox questioning of the established theories that… they demanded a proper re-print be done:

        Believing as Rheticus did that Petreius was partially responsible for the publication of the unauthorized preface, Copernicus’s friend Tiedemann Giese, a Roman Catholic bishop, wrote to the city council of Nürnberg to demand that Petreius be compelled to issue a reprint.

        And to end with a tidy anecdote to balance out Galileo and Eppur si muove:

        Legend has it that a copy of De revolutionibus was placed in Copernicus’s hands a few days after he lost consciousness from a stroke. He awoke long enough to realize that he was holding his great book and then expired, publishing as he perished.

    • ksvanhorn says:

      Regardless of anything else, there is one very important sense in which the Catholic Church and every other religion is anti-science: their promotion of faith as an ideal. Faith — believing, regardless of the evidence, what you desire to believe, or what you think you should believe, or what others say you must believe — is an anti-epistemic practice, a fundamentally dishonest way of thinking, and is anathema to free scientific inquiry.

      • The original Mr. X says:

        believing, regardless of the evidence, what you desire to believe, or what you think you should believe, or what others say you must believe

        That’s not how the virtue of fides is defined.

    • Conrad Honcho says:

      1) Bravo.

      2) This is the same thought process that leads to blaming Islamic terrorism on economic marginalization or climate change. “Sure, they say they’re slaughtering the infidels in the name of Allah, but since I don’t believe in any religious mumbo-jumbo, I wouldn’t do that, so it must be because they’re oppressed in some way.” No, no, they actually believe in slaughtering infidels in the name of Allah.

  49. Eli says:

    Lucilio Vanini was a philosopher/scientist/hermeticist/early heliocentrism proponent who was most notable as the first person recorded to have claimed that humans evolved from apes – though his theories and arguments were kind of confused and he probably got it right mostly by chance. City authorities arrested him for blasphemy, cut out his tongue, strangled him, and burned his body at the stake. But nobody cared about his views on evolution at the time; the exact charges are unclear but he was known to make claims like “all religious things are false”.

    Getting four major things right (heliocentrism, evolution from apes, laws of physics run the universe, no gods or divine interventions into our reality) seems a whole lot like clear thinking and cleverness, not like getting things right by chance.

    Now, sure, heliocentrism was rising in evidence, physicalism lets you derive the lack of intervening gods very nearly by deduction, and physicalism is also a huge hint that points you towards looking at the evidence for evolution. But still! Each conclusion, taken on its own, requires a lot of observational evidence, but when you notice the logical links between them, the amount of looking you need to do is dramatically reduced. That’s important.

    • Deiseach says:

      Not to keep flogging the Galileo horse, but he also deliberately picked the strawman version of geocentrism in his book, plus he got into a pissing match with a Jesuit mathematician over the reason for the tides: Jesuit says it’s the moon, Galileo calls him five kinds of donkey for this – because Galileo was using the tides as one of his ‘proofs’ for heliocentrism, via the motion of the earth, and if that claim was shown to be false then a leg was knocked from under his theory.

      Galileo was not all about the pure true science, he was a gloryhound as well and every bit as invested in ignoring inconvenient facts when they didn’t support his theory. But we don’t hear so much about the part where he’s wrong as we do about the story of eppur si muove.

      And Galileo really wasn’t the brave challenger of religion or proto-atheist that some would like to make him out to be; ironically, in view of the trouble he got into, he probably was the kind of conventional believer of that day and our own, who vaguely assented to most things, didn’t feel particularly troubled by religious doubts, considered himself “I’m not perfect but hey I’m a good guy” and expected to end up in The Good Place. His two illegitimate daughters became nuns, and the 90s book by Davina Sobel has a selection of the letters sent to Galileo by his daughter Sister Maria Celeste.

      Please note this, from the New York Times review of the book, which is the point I made in my long comment:

      the same year the Dominican friar Giordano Bruno was burned at the stake in Rome for insisting, among his many heresies and blasphemies, that the Earth traveled around the Sun, instead of remaining motionless at the center of the universe.

      You see? The real reason Bruno was burned was not his “many heresies and blasphemies”, it was for heliocentrism. That’s the attitude of today, because the secular citizens of today who have little to no care for religion and which has little power or effect in their lives can’t conceive of it actually mattering, so the real reason must be something else – and for moderns, nourished on the myth of Science Versus Religion, that something else has to be science. That people back then might not have particularly cared tuppence if the sun went round the earth or the earth went round the moon but passionately cared if Christ was the Son of God – no, they can’t think themselves back into that mindset. They can imagine intelligent people like themselves back then, and if they are like themselves they cannot really believe all that nonsense, but with a cynical attitude that of course the peasants and rubes believe all that and it’s useful to pretend to believe it, but it’s only a show. So if you have an enemy you get rid of them by appealing to something the peasants and rubes can understand, and you pretend they said something false about God – that there is no God and so it is impossible to say something false about a non-existent entity – well, the simple folk can’t handle that truth, so it’s convenient to use it as an excuse.

    • B_Rat says:

      Getting four major things right (heliocentrism, evolution from apes, laws of physics run the universe, no gods or divine interventions into our reality) seems a whole lot like clear thinking and cleverness, not like getting things right by chance.

      Like with Bruno, the “get right” part has more to do with the imaginations stimulated by modern lectures of his works than with the author himself, a bit like the Vedas were “obviously” about Quantum Physics.

      BTW, “laws of physics run the universe” was not even controversial, the Middle Ages already had the “The world as a giant mechanical clock with God as its clockmaker” methaphor.

    • B_Rat says:

      Getting four major things right (heliocentrism, evolution from apes, laws of physics run the universe, no gods or divine interventions into our reality) seems a whole lot like clear thinking and cleverness, not like getting things right by chance.

      Like with Bruno, the “get right” part has more to do with the imaginations stimulated by modern lectures of his works than with the author himself, a bit like the Vedas were “obviously” about Quantum Physics.

      BTW, “laws of physics run the universe” was not even controversial, the Middle Ages already had the “The world as a giant mechanical clock with God as its clockmaker” methaphor.

      Also, heliocentrism’s opposition was actually “rising in evidence” during Vanini’s times, thanks to Tycho Brahe’s work.

  50. MugaSofer says:

    There’s a big problem with this post: we already have anonymous discussion fora. These are whisper networks free from the disadvantages of their hypothesized medieval counterparts, in that they are large and can contain vigorous debate; they can even be used to co-ordinate large-scale actions that would be socially or legally unacceptable.

    And yet 4chan, Dark Web fora etc – although they can be interesting – do not actually resemble the virtuous whisper network you describe. Instead, they are predictably filled with pedophiles, anti-semites, conspiracy theorists, trolls and outright con artists, all egging each other on to ever-greater fits of stupidity.

    The only topic I can think of that actually sort of fits this “whisper network” description is drug use; drug users are actually pretty good at coordinating and sharing information. But to say that drugs should be legal, and even to share information on them, is perfectly legal (and indeed quite popular in liberal circles!) Drug users only make use of the whisper network because it provides an avenue for selling and purchasing drugs; using it to share information is a side benefit.

    EDIT: one might also claim that these modern whisper networks are used to share the forbidden truth that SJWs totally suck, man. But again, this is not actually forbidden: hey, everyone, large portions of the SJ community are terrible people! They suck! See, nobody cares.

    There’s very little discussion – even to refute them – of the supposed “forbidden truths” that some rationalists tend to darkly hint at, like enpr naq VD (nf bccbfrq gb trarevp “juvgrf ner gur orfg enu” enpvfz), fbpvrgl crefrphgrf crqbcuvyrf varcgyl naq sne gbb zhpu, aEk, Rhtravpf vfa’g arprffnevyl onq, zra unir uvture VD inevnapr, rirelbar fubhyq or ov, aba-ercebqhpgvir vaprfg vf zbenyyl npprcgnoyr, vasnagvpvqr vf Bx orpnhfr nobegvba, etc etc.

    • Aapje says:

      @MugaSofer

      And yet 4chan, Dark Web fora etc – although they can be interesting – do not actually resemble the virtuous whisper network you describe.

      You seem to be cherry picking. Why 4chan and not SSC?

      • Anonymous says:

        Because the gardener rather dislikes the anonymi.

        • 6jfvkd8lu7cc says:

          He asks for a consistent identity; without insistence on a single consistent identity, or any links from that identity (identities) to any identity used in a different context.

    • 天可汗 says:

      People who are integrated into society tend to actively desire to believe orthodoxy and disbelieve heresy.

    • Scott Alexander says:

      I think “whisper network” is fundamentally different from “open forum which happens to be anonymous” for exactly this reason.

      One of the things I was thinking of was sexual harassment whisper networks, where as long as they’re actually done informally they seem to be pretty good, but where passing an official list around seems to be going worse. My guess is if there was a 4chan like “Accuse People Of Sexual Harassment Forum” it would go terribly, because the informal networks allow reputation as well as secrecy. If your boss tells you someone in the workplace is a sexual harasser, that means something, and you can have a reasonable conversation about it.

      • Jiro says:

        as long as they’re actually done informally they seem to be pretty good, but where passing an official list around seems to be going worse.

        Could that be availability bias. or related? It’s easier to notice that a group is acting badly when you have a list you can point to as evidence of them acting badly.

    • birdboy2000 says:

      You seem to be reducing one of the web’s most popular fora to the most obnoxious section of its politics, television, and video games boards. I’ve found that “whisper network” (in addition to one of my favorite communities to hang out in) to be a seriously impressive resource on a variety of topics – check out some general thread OPs from time to time.

      (Also, file sharing meets your drug analogy pretty well, and 4chan is certainly a hub for info on that topic.)

  51. Cecil Harvey says:

    I’m a Catholic, and I appreciate the distinction you are making, and think you’re being very fair and even-handed, even though I may disagree with you on some finer points.

    I don’t think orthodoxy of a society can ever be quelled or suppressed. We just trade one orthodoxy for another. And any orthodoxy believes the old Catholic maxim that “error has no rights.” It seeks to crush all dissent.

    In the west, we’re living under a strange humanistic progressive orthodoxy now. It purports not to believe in anything, except the cardinal sins of sexism, racism, climate change denialism, and failure to sufficiently celebrate homosexuality and transgenderism. It has its own liturgies, its own codex of forbidden literature, its own sacred tradition, its own rules of blasphemy.

    I don’t think it’s possible to make a society that doesn’t seek to quash all heterodox thought. It might be possible in limited circumstances or for limited times, but as a more-or-less permanent state, it doesn’t work. I don’t buy the whole conservative argument that this was the case until the 1950’s when liberals took over. It as just a different orthodoxy that was extant before the modern progressive orthodoxy conquered all. Until after WW2, blacks, Catholics, Jews, Asians, etc. were second- or third-class citizens. As were anyone who didn’t pay lip service a sort of bland, patriotic protestant Christianity that didn’t actually require people to live in a manner that was virtuous or sacrifice anything for the common good.

    • Winter Shaker says:

      I don’t think it’s possible to make a society that doesn’t seek to quash all heterodox thought.

      I think that the Fallacy of Grey might be relevant here: “That which I cannot eliminate may be well worth reducing.”

      Are you claiming that there is a sort of orthodoxy set point of human society, which we naturally regress to, even if the specific contents change, and that we cannot in the long run reduce the degree to which heterodox thinkers are persecuted? I’m not really sure how to get a handle on that question. I mean, I’d sooner be fired from my job than burnt at the stake or exiled to Siberia, so it certainly looks at a glance like current Universal Culture is better than Mediaeval Christendom or the USSR in that regard, but maybe we are persecuting a larger number of people for smaller deviations from orthodoxy, even if the punishment is typically less severe.

      • Cecil Harvey says:

        Sure, today it’s being fired from my job — which I would be, if my employer knew my thoughts on gay marriage.

        In Canada, Catholic Priests are being fined for “hate speech” when teaching traditional Catholic sexual morality. I don’t think we’re far behind that in the US. I honestly think that in 10 years or so, parents who teach Catholic morality will have their kids taken away from them by the state, which in my opinion is worse than being executed for dissent. I’d gladly be executed if I could successfully raise my children as believing Catholics. And I’ll die, rifle in hand, before allowing the state to take my kids away for indoctrination.

        • qwints says:

          I honestly think that in 10 years or so, parents who teach Catholic morality will have their kids taken away from them by the state, which in my opinion is worse than being executed for dissent.

          Care to bet? I think the odds of even one case of judicially sustained removal in the US for religious instruction by married parents before 2028 are less than 1% as it would essentially require a revolution or coup. Custody cases are a different story, and courts have already barred religious instruction by one parent when it was a change from the child’s previous religious instruction.

          • Cecil Harvey says:

            If you care to stretch the range to 10-20 years, I’d take that wager. I did, in a weasely way say “10 years or so”. I’d be willing to put a hard cap on 10-20 years, however. I’d be willing to bet, in this range, $1000.

            That being said, are either of us likely to be in contact in 20 years, and I’m not breaking anonymity for this. So it’s your prerogative to gauge my seriousness in this.

          • qwints says:

            We’d need a trusted third party to hold it and resolve the funds or to use something like an ethereum contract. You’re right that the time scale precludes us actually making a bet, both because of the difficulty of finding a trusted repository and because the time value of the money exceeds the value of the bet. Oh well.

            I’m making a note in a software calendar (interesting to see if that survives 20 years) to check 10/24/2037 to see if there’s been a case.

          • Cecil Harvey says:

            I’ll do the same.

            On the off chance I’m still using the same calendar in 20 years, and this post still also exists, I’ll look it up.

          • I think the odds of even one case of judicially sustained removal in the US for religious instruction by married parents before 2028 are less than 1% as it would essentially require a revolution or coup.

            How else would you describe the Texas child protection authorities taking several hundred children away from their mothers, including male infants, because they disapproved of the parents’ religious view (polygamy) and didn’t want it taught to the children?

            It’s true that it was judicially sustained only in the lower court and reversed in the appeals court and the Supreme Court. Do you regard that series of events as sufficient to justify your claim?

          • qwints says:

            I would not count a trial court ordering a removal as “judicially sustained” . Some judges do crazy and obviously illegal things in family court, and I wouldn’t be surprised at all if a judge had already sought to order married parents not to teach a religion.

          • I would not count a trial court ordering a removal as “judicially sustained”

            Fair enough. And you don’t think the fact that the action was taken on a large scale, approved of at the bottom level of the judicial system, widely approved of and supported in the national public discussion, cancelled only after both the appeals court and the state Supreme Court found it to be illegal, and resulted in no legal penalties to those responsible, suggests that it might well be judicially sustained somewhere in the U.S. sometime in the next ten years?

          • qwints says:

            No, I don’t. The trial judge was overturned at the first possible opportunity. The trial judge (illegally) ordered the removal because she believed there was rampant child abuse based on an intentionally false report.

    • Aapje says:

      I don’t think it’s possible to make a society that doesn’t seek to quash all heterodox thought.

      We can still fight for a society that does this the least and fight the quashing that has the most harmful consequences.

      • Cecil Harvey says:

        Of course we can. But it’s best to know when you’re fighting the long defeat.

      • Questioner says:

        “We can still fight for a society that does this the least and fight the quashing that has the most harmful consequences.”

        Sorry, but in my mind that just translates to “We can still fight for a society that crushes views I don’t like, and doesn’t crush ones I do like.”

        “So imagine the most irrelevant orthodoxy you can think of.”

        Ok, how about the idea that biological reality doesn’t matter, all that matters is what you’ve decided your “gender” is?

        Or, honoring James Damore, we’ll consider the orthodoxy that “men and women are exactly the same, and that any difference in employment of women is proof of sexism”? Oh, and while we’re at it we’ll carefully avoid noting that if men and women are exactly the same, then transgenderism can’t possible be real, since there’s no “male feelings” or “female feelings”.

        CA’s gearing up for criminal prosecutions of people violating the first orthodoxy. It’s really easy to get fired for violating the second one.

        And if you’re willing to watch people’s lives be destroyed for violating your orthodoxies
        1: You don’t get to legitimately bag on the Catholics / Protestants for doing the same
        2: You don’t get to whine when others decide to burn you at the stake. And IMO we’re not that far away from the pyres going both way

        (Note: No, I’m not going to hauling you up on a pyre. But unless I can see solid evidence you were fighting against the Left every time they hauled someone up on a pyre, I’m not going to fight to save you, either.)

  52. smopecakes says:

    I feel like a lapsed skeptic for not thinking of global warming in this context until it came up. *does the lukewarmer’s cross*

    • jhertzlinger says:

      That reminds me… Sometimes the most effective defense against a group of censors is to be protected by another group with its own censors.

      • smopecakes says:

        Interestingly, while I’ve heard of a person chastised for saying that certain events were not caused by global warming (and the people telling them not to fully agreed that they were right, just not being useful) the particular opinion I’ve seen socially censored by skeptic groups (Watts Up With That) is that CO2 does not have a warming effect.

  53. rahien.din says:

    IIa.

    you can’t have a mind that questions the stars but never thinks to question the Bible [or to send a personal letter to Stalin.]

    The Algernon argument at work.

    Potentials tradeoffs for “I have the ability to challenge large, fundamental concepts” may include “I don’t have enough sense to not write a letter to Stalin criticizing his policies” or “I will endorse lunatic occultism to the extent that I invent new colors” or “I will also directly challenge my world’s largest and most powerful institution on its home turf.”

    IIb.

    The Church didn’t lift a finger against science. It just accidentally created a honeytrap that attracted and destroyed scientifically curious people.

    This has always bugged me. Your implicit claim is “if not for the church, everyone in medieval society would have acted in a manner that I would find totally reasonable.” Whence the church and its disagreeable attitudes?

    This is not mere hand-waving. It all had to come from somewhere. Even if you claim “The-Church-people developed a system of non-negotiable arbitrary beliefs, which non-The-Church-people would never have developed on their own. But all of the non-The-Church-people acquiesced to these beliefs arbitrarily,” you’ve just moved goalposts so to let society off the hook. Society is on the hook.

    Moreover, the alternative scenario you propose is one not worth embracing. Arguendo, if not for The Church, the lay contemporaries of Bruno would have been entirely willing to update their worldview from “lights embedded in celestial spheres that orbit the earth” to the correct view of “other suns, around which other planets orbit, potentially inhabited by other life-forms.” What else would they have been willing to update? Refer to IIa. What would that look like on an individual level? What would that mean for society? My guess is it wouldn’t be all that great.

    There are correct and incorrect timings with any paradigm. Consider the Second Vatican Council. It was a big surprise when it happened in the 1960’s, and it caused some definite upheaval. But that upheaval was peaceful. If it had happened in to 1560’s, there may have been civil war. Even if Gregory XVI had proposed such reforms in the 1860’s, the proper response would have been “Wait a bit, Father.”

    I’m not even denying that the Church (in your examples) was acting to maintain its power and authority, for surely it was. But even this self-serving impulse is not necessarily bad/deplorable/the wrong thing. Society has inertia. Sudden changes in inertia cause problems. Resistance to these inertial changes is not even at the level of a Chesterton fence, because, it’s reasonable to expect that such resistance will someday be gone. It’s more like a Chesterton speed limit sign. Maybe “Chesterton speed limit sign” is the same thing as “externally-imposed epistemic humility.”

    My competing hypothesis : the iconoclasts you cite lacked the ability to respond to Chesterton speed limit signs (neither implicit ones, nor explicit ones). Refer to IIa.

    III/IV.

    when a heretical belief turns out to really be completely wrong – maybe occultism would be a good example here – a whisper network might be the only place where you could get high-enough-quality debate to be sure.

    There is a certain kind of website which the SSC filters are really, really good at rejecting. As a psychiatrist, Scott knows exactly which one I’m thinking of. It’s a spectacular (and pernicious) whisper network. The vaccination opposition movement is a great example of a whisper network that attained enough mass to go public. A whisper network, in these cases, is exactly the wrong place to go if you want high-quality debate or to flee orthodoxy.

    What you’re ultimately describing isn’t necessarily “truth-containing heterodoxy vs truth-denying orthodoxy.” What you’re describing is “competing orthodoxies, which may or may not contain truth.”

    • multiheaded says:

      Consider the Second Vatican Council. It was a big surprise when it happened in the 1960’s, and it caused some definite upheaval. But that upheaval was peaceful. If it had happened in to 1560’s, there may have been civil war.

      Certain people, some of them to be found around this comment section, are absolutely mad as all fuck about Vatican II. And that’s not a metaphor, I’m talking about a real politico-religious movement. They just don’t have the numbers or the clout to take that fight to the official Catholic church itself and prioritize lower-hanging fruit like agitating among lay believers. Or fund propaganda that’s not at all in line with the mainstream Catholic consensus.

      https://en.wikipedia.org/wiki/Society_of_Saint_Pius_X

      • rahien.din says:

        Consider the Second Vatican Council. It was a big surprise when it happened in the 1960’s, and it caused some definite upheaval. But that upheaval was peaceful.

        Certain people are absolutely mad as all fuck about Vatican II.

        You’re supporting my point : even the people who are as mad as all fuck are doing things like talking about it. In contrast to how this could have gone down in prior eras, they aren’t killing or dying or schisming over it.

      • Nick says:

        Quibble: traditionalists, sedevacantists, schismatics, etc, are very much a thing, but as far as I know there actually aren’t any on SSC.

    • B_Rat says:

      Arguendo, if not for The Church, the lay contemporaries of Bruno would have been entirely willing to update their worldview from “lights embedded in celestial spheres that orbit the earth” to the correct view of “other suns, around which other planets orbit, potentially inhabited by other life-forms.”

      The funny thing is that these is stuff Bruno explicitly took from “the Divine Cusanus” he admired, a guy that a century before had been made a Cardinal and powerful Papal Legate in the years after publishing his ideas.
      The way both this post and many of its comments try to understand human nature from history is often lacking in the “history” compartment.

  54. Chalid says:

    This post is kind of empty, since it contains nothing about the magnitudes of the effects involved.

    OK, you sketch out a mechanism by which “thunder comes before lightning” might have a corrosive effect on the whole truth-seeking enterprise, and you’ve used all the usual writers’ tricks to make it seem like a big big deal, but there isn’t actually any reason in your post that I should believe this is a significant effect. “Some people” will ask socially inconvenient questions and be silenced – how many? One person a century? How would I think about if this is going to be strong enough to matter?

    The medieval Church was a far stronger and more significant reality-distortion than anything we’ve got going on today. Maybe we could think of that as an upper bound on the strength of these effects in today’s society. What was the ratio of scientists silenced to scientists that continued working unmolested in that period? How much was progress actually impeded? Anyone have any way of estimating that?

    • Robert Liguori says:

      If we want another example of how bad the practical effects can get, we can look at the great leaps forward Soviet genetics took under Lysenkoism, as mentioned upthread.

      • David Shaffer says:

        Or modern social justice. Even if one were to assume that they’re right about all object-level issues, the attitude that their dogmas can be asserted rather than backed up with reason, and that anyone who disagrees is automatically evil rather than potentially honestly mistaken, is extremely corrosive to seeking truth. Even if they were entirely correct, getting in the habit of that kind of epistemic sloppiness would lead to mistakes down the line.

    • cuke says:

      Your use of “reality-distortion” got me thinking because with my patients I sometimes talk about depression (or anxiety, say) as creating a kind of distortion field that skews thinking in particular ways. Depression scales (given as questionnaires to patients) are in some sense a way to measure the magnitude and scope of the distortion field.

      So certain kinds of collectively-held dogmas, or authoritarian regimes, can also create distortion fields that can be measured. I was reading recently an instrument used by some international journalism group (can’t remember the source) to measure the magnitude of the distortion field around the media (how free is the press) in any given country… are journalists killed/jailed/fired for doing their jobs? what rights to free press are codified by the society’s laws? Does the government exert editorial control and through what means? and so on.

      We might haggle over exactly what the scale should be if the issue is “free pursuit of science” or some other issue, but certainly we can imagine for the sake of conversation that we could identify some of the indicators of a distortion field around science. Is scientific consensus forbidden to be taught in schools (evolution, etc); is research data being suppressed in government reports; are scientists being jailed? The research instrument about freedom of the press shows a continuum of suppression and reading it you can pretty quickly see where one’s own country might fall on that spectrum.

      To me, Scott’s post is valuable without providing the instrument that measures the magnitude of the distortion field because it captures some of the dynamics of how the distortion field is created, reproduced, and maybe undermined — regardless of the magnitude.

      People’s actions have consequences: in extremely authoritarian regimes, those consequences may be lethal; in less authoritarian regimes, consequences may include loss of reputation or job. Political savvy or politeness/diplomacy might buffer some of the worst consequences in any regime, while being a jerk or simply politically naive is likely to subject one’s actions to the worst consequences available in that regime. Various kinds of wealth or privilege can also act as buffer.

      It seems to me we can still have an interesting conversation about the dynamics of suppression without measuring its level, with the knowledge that different countries and times in history land in different spots on the spectrum.

    • quanta413 says:

      The medieval Church was a far stronger and more significant reality-distortion than anything we’ve got going on today.

      Is that actually true? Obviously it’s very hard to quantify, but I’m not convinced that the medieval Church was even as reality distorting as the U.S. government currently is. The medieval church could relatively easily be fled. It didn’t even cover all of Europe. Pagans, protestants, and Muslims were all relevant at one point or another and the Muslim civilizations were often more powerful and larger. The U.S. government can hunt you down almost anywhere in the world if it really wants to.

      My relatively weak understanding is that the medieval church was most active as a censor in theological disputes and only had very small branches of the organization dealing with things like inquisitions. The U.S. government on the other hand has deployed troops in how many countries and controls arms dealing with how many regimes?

      Sure the medieval church might burn you for saying or writing things, but it’s not like that was a huge business for them. The body count they wracked up wasn’t that high when you consider that it’s spread over a millenium.

      • suntzuanime says:

        I feel like you’re not comparing like to like. The US government is not going to hunt you down in another country for heterodoxy (there was that one guy Obama had murdered, but that’s pretty far from the norm and was more related to geopolitics than ideological orthodoxy). And the Catholic Church deployed troops in plenty of countries too, it was kind of a whole thing, if someone’s not complaining about the Inquisition they’re complaining about the Crusades.

  55. armorsmith42 says:

    The Death of Stalin was a very entertaining comedy and I recommend it. Before I suggest that it could be a rich source of metaphors for discussing this sort of thing, Does anyone have any audiobook recommendations for someone who wants to know how historically-accurate it is?

  56. Gabriel Conroy says:

    This might very well be my first comment on this site, and I fear it’s a rude comment, because I’m not going to address the post itself, but one of its premises, having to do with the list of scientists you offer. While I realize people mention the explanations you recite as evidence that the church* wasn’t “anti-science,” I suggest another way to think of those examples is that the church–and religion–are not *peculiarly* anti-science, or at least haven’t always or necessarily been so. That’s no defense for giving religious authorities the power to censor or punish thoughts, but it’s possible to use those examples of scientists to demonstrate a narrower point.

    *Of course, once we get to Servetus, we can’t really use “the” church anymore. Not that we really could before, but with the reformation there were more institutions that lay claim to that title.

  57. untimelyreflections says:

    I would just like to get on the record the premises of the argument as originally presented elsewhere

    > It is OK to imprison and torture and kill people who are rude to figures in authority

    > It is OK to imprison and torture and kill people who adhere to the wrong religious beliefs

    > As long as you don’t to this to people only because of scientific beliefs it’s OK.

    Worth pointing out too, it can be very hard to work out which are obscure scientific questions and which are politically sensitive matters. This is a fast moving target.

    Finally, this is very costly. Darwin held off publishing Origin of Species for decades for fear of the reaction. Several of Nietzsche’s books were not published in his lifetime for the same reason.

    Affirmative action would be very costly in an alternate universe where different groups had substantial genetic based differences which people pretended did not exist because of ideology. In that alternate universe, more people who were incompetent would be doing jobs they were not up to, with potentially high costs and even fatalities.

    • herbert herberson says:

      Affirmative action would be very costly in an alternate universe where different groups had substantial genetic based differences which people pretended did not exist because of ideology. In that alternate universe, more people who were incompetent would be doing jobs they were not up to, with potentially high costs and even fatalities.

      The degree to which this is true depends entirely on how widely/deeply implemented the affirmative action is. This is particularly true if we posit a third alternative universe where where different groups had substantial genetic based differences which people pretended did not exist because of ideology, but those groups also had a lengthy history of oppression that very clearly caused their descendents to suffer under various (economic/biochemical/social) disadvantages that in turn often interfere with the metrics used by society to identify/exploit/reward genuine ability even when contemporary oppression was not in play (which it sometimes still was).

      • reasoned argumentation says:

        Nope – not unless the metrics were broken and underpredicted performance drop due to oppression but oppression only caused underperformance on the metrics. You’d still get incompetents performing surgery.

        • herbert herberson says:

          For one, yes, the metrics are obviously broken. But since we’re only talking about a hypothetical alternative universe, let’s say for the sake of argument that they aren’t.

          But let’s also assume that this universe contains a process that can significantly correct the oppression-related causes of underperformance (or, at least the non-biochemical ones), and also assume that access to it is the most central example of affirmative action. The system that implements this process has trouble identifying high-IQ/conscientious/etc people whose lives are cluttered up with poverty and intergenerational trauma and other consequences of oppression, but can educate those individuals into a high level of competence. In this alternate universe, failing to adjust for the costs of oppression will cause the system to pass over some candidates from the oppressed group who would take great advantage from the education and become highly competent in favor of someone from the non-oppressed group who would take moderate advantage from the education and become moderately competent, thereby using its resources less efficiently, creating a less talented pool of educated persons, and postponing the day when the history of oppression is distant enough to be ignored.

        • reasoned argumentation says:

          The metrics aren’t wrong though. They correlate with results for all groups.

          You’re claiming that there’s some kind of oppression that could cause underperformance on any measurement used to predict future performance but at the same time doesn’t cause underperformance on the actual task. Of course, if that was the case you could simply add some kind of oppression bonus to the test and end up with a valid predictor – adding X points to an SAT score or subtracting Y seconds from a 40 yard dash time. Here’s the problem though – it doesn’t work. The metrics track performance across all groups and they are consistent in predictive power across groups as well.

          Not only do you have to believe in that insanity you also have to believe that this oppression-fueled underperformance causes metrics to turn up results that are random for the oppressed group – otherwise the “add X points to the results of the metric” would work – and it doesn’t.

          For one, yes, the metrics are obviously broken.

          No, they’re not. A broken metric is one that fails to predict performance – not one that turns up results you don’t like.

          • herbert herberson says:

            The SAT is only one small component of the relevant metrics, though. Assuming it is accurate (and I have no particular reason to think it isn’t), that still leaves you with grades, “extracurriculars,” how much your high school guidance counselor likes you, and all of the innumerable other factors that separates, say, rich suburbanites who do SAT-prep and send out fifty applications and do interviews and competently search for what few scholarships they need vs. the guys who would do great on the SAT if they took it but never did because none of the adults in their families had ever been to college or really gave that much of a shit plus they have that misdemeanor for pot instead of getting off with a warning and the local community college offers pretty good scholarships according to the guy who tabled at lunch last week.

            And that’s just for college! For most jobs it boils down to nothing more or less than credentialism, nepotism, and how personable you are to the person hiring. It’s all a very obvious joke.

            If you imagine education a flat and consistent benefit, a “+10 to skills,” then maybe you could disregard all that, say “yeah and their working-class families will tend to give them inferior working-class values and it is therefore rational to discriminate against them.” But I don’t think education works that way at all.

          • reasoned argumentation says:

            … and all of the innumerable other factors that separates, say, rich suburbanites who do SAT-prep and send out fifty applications and do interviews and competently search for what few scholarships they need vs. the guys who would do great on the SAT if they took it but never did because none of the adults in their families had ever been to college …

            I get that you can’t help yourself but you again just made a testable (and really strange) prediction – that the metric underrates performance for “oppressed” groups but the outcomes it’s intended to predict show no such difference. Somehow all these differences that lower MCAT scores don’t reduce performance as a doctor – this might be true in some hypothetical world but in this world there’s a linear relationship between MCAT scores and medical malpractice committed.

            You’re claiming that oppression has this weird effect of only suppressing metrics that are used to predict performance while not actually suppressing performance. It’s magical thinking.

          • herbert herberson says:

            First and foremost–I’m not making predictions! We’re just having a friendly conversation about a hypothetical world!

            However, I’d expect that even in your hypothetical world, there is a difference between a valid test with verified predictive value and a perfect test that cannot be improved through further adjustment.

    • chrisminor0002 says:

      The cost of Affirmative Action is independent of whether or not the differences are based in genetics. If racial differences are entirely the result of culture, that means something for our future and what’s possible, but it still means that Affirmative Action is imposing a cost on employers and more-qualified workers in this universe.

  58. Sergei Lewis says:

    To find the Soviet era whisper network, you might like to check out science fiction of the time; e.g. by the Strugatsky brothers. Start with “Hard to be God”, then go on to “Monday begins on Saturday”; between them they contain much of this post.

    • multiheaded says:

      You mean a tragic sci-fi novel about a just if quixotic effort to hasten the advent of progressive values in a benighted society, and a spirited love letter to the 1960s Soviet research community with all its idiosyncrasies, respectively?

  59. nate_rausch says:

    I have a question. How do social norms form to begin with, how do they change and what sustains the equilibrium?

    Our current days feel to me to be plagued by some social norms of religious-level conviction that are not most importantly wrong (in fact probably partly right), but more that they are coupled with a sort of holiness. Anyone who touches the holy grail must be destroyed-type situation.

    These seem to come and fall. Most of Eastern-Europe has seemed to throw out Marxist orthodoxy quite easily and without much disruption. It seemed to only be perpetuated by the authority remaining in power. With a more religous orthodoxy this is slightly more decentralized I suppose. But still there must have been spheres of influence. Some countries, some areas (Florence, Athens) where the orthodoxy for a time didn’t reach.

    If one where to take the whisper-network to Next-Level Whisper-network, could one perhaps create a place where the orthodoxy had no reach, and where truth-seeking could survive and generate memes until it figured out a way to overthrow or corrupt the religious authorities?

  60. sconzey says:

    Magnificent steelmanning of anticlericalism.

    You assume that we all agree that edgelording is an unavoidable trait in the truth-seeker. But edgelording doesn’t come from a desire to seek the truth, but from a desire to rub others noses in your superior smarts. (At least, that was my experience as a teenaged edgelord.)

    There is a very important difference between carefully researching the LQ and discovering ‘huh, it turns out lightning *does* come before thunder’ and then moving on to something else, and standing on a park bench shouting ‘WAKE UP SHEEPLE! I ALONE HAVE DISCOVERED A TRUTH THAT YOU ARE ALL TO STUPID TO DISCOVER!’

    I too experience curiousity as an intellectual itch, but that itch is entirely scratched by discovering the truth.

    Correcting people who are wrong? That’s an entirely different itch (but sometimes a socially useful one!), and it’s misleading to imply they are the same, because the question is then: when should you correct someone who is wrong? Which is a more interesting and useful question, but one where the answer is less morally clear-cut.

    • Tracy W says:

      How about a sense of irritation at bad arguments? Or terrible observational skills?

      I sometimes have a feeling of “You people, you are better than this! There’s no way you should be arguing this sort of sloppy stuff! I know you’ve got a head on your shoulders, for goodness sake use it!”

      • Aapje says:

        @sconzey & Tracy W

        How can you know whether you are right unless you have made your claim and given people an opportunity to shoot holes in the argument?

        • sconzey says:

          How can you know whether or not you are right…

          Because reality exists whether or not I have a winning argument in its favour.

          • Aapje says:

            What exists in your and my head is not ‘reality,’ it is a simplified model which can be more or less correct.

            You still need a method for building up a good model. I don’t think that very many people are capable of doing that without building up a narrative and having people try to poke holes in it.

        • Tracy W says:

          @Aajpe: I can’t even know then. The people shooting at my ideas might just not be that competent.

          But some claims I’m pretty darn sure about.

          • Aapje says:

            @Tracy W

            Of course, but one the whole, you should definitely be more confident of opinions that you make public and get no good counterarguments, than of opinions that you didn’t give anyone a chance to criticize.

          • Tracy W says:

            Sure. But I don’t follow what point you’re trying to make.

    • 天可汗 says:

      There is a very important difference between carefully researching the LQ and discovering ‘huh, it turns out lightning *does* come before thunder’ and then moving on to something else, and standing on a park bench shouting ‘WAKE UP SHEEPLE! I ALONE HAVE DISCOVERED A TRUTH THAT YOU ARE ALL TO STUPID TO DISCOVER!’

      Yes, that is in fact where the analogy breaks down.

      You see, the reason thunder comes before lightning is that lightning is the air’s response to the presence of the Thunder Devil. The Thunder Devil is angry. If we don’t keep the Thunder Devil at bay, he’ll destroy our entire country. Tall people are agents of the Thunder Devil; he pulls them up toward him because he likes them so much. So they have to pay punitive taxes, and if you and your friends see one on the street and beat him up, well, you know, I guess that’s “bad” and all but he did sort of deserve it for being in league with the Thunder Devil. And any town that has unusually many thunderstorms is in league with the Thunder Devil, so we’ll have to punish everyone in it, and anyone who disagrees is also in league with the Thunder Devil, and so on.

      I think the question of whether thunder comes before lightning would be rather more important in this scenario than in the analogy as originally stated.

      People seem to care about false beliefs in proportion to their connection to the rest of the associated belief-system. More people are emotionally invested in combating young-Earth creationism than are emotionally invested in combating astrology, because astrology is jabberwocky that doesn’t propagate through the belief-network and young-Earth creationism is jabberwocky that does. At the same time, you’ll face fewer penalties for disputing astrology in places where people tend to believe in it than for disputing young-Earth creationism in places where people tend to believe in that. And if you dispute that Kim Il-sung’s birth on Mt. Paektu caused a double rainbow to appear in the sky and winter to turn to spring and that his grandson controls the weather and doesn’t shit or piss, well…

    • Scott Alexander says:

      “I too experience curiousity as an intellectual itch, but that itch is entirely scratched by discovering the truth.”

      Oh, man, look at this guy who’s discovered the truth!

      …sorry. But the way it works for me is I discover something that seems funny, and it bothers me, and then I feel sort of bad when it doesn’t fit into an existing paradigm, and then I wish I could ask other people about it, and then other people refuse to even acknowledge that it seems funny, and then I try shouting it, and then I end up accidentally crossing the border into edgelordship. I think a big part of this post is that if you could just discover all the forbidden truths and be done with it, that would be fine. It’s when you don’t know which things are true, or forbidden, or how true or forbidden they are, that it gets troubling.

    • cuke says:

      Science advances because scientists share their results with each other and the world, their “truths” are tested and critiqued, and then they go back and search for better truths.

      If one’s job is a doctor or policy maker or whatever, it seems like being able to communicate how some evidence base is guiding action is important to being able to take said actions. If truth matters at all, it seems like it has to be communicated in multiple arenas. There’s no real opt-out option that I see. Am I misunderstanding you?

      If we’re all just quietly pursuing our own private truths and not speaking up about what we’re learning, I don’t see how society functions or significant problems get solved.

    • The Nybbler says:

      How about simple self-interest? If the people are using the wrong beliefs in a way that operates to your detriment? Say they’re denying your insurance claim for being hit by lightning because you should have heard it coming, or something. (yah, it’s a bit tortured, but I’m sure you can think of real-life heretical beliefs for which the orthodox belief is more naturally damaging).

      • multiheaded says:

        Note that, well… this is exactly what the orthodox explanation of your side’s driving motive is.

        I shall not stretch the weather metaphor, but this charge typically goes like, “you’re privileging a specific interpretation of the issues that frames your pursuit of self-interest as a reasonable default reaction while minimizing or denying responsibility for disproportionate contingent harm to others”.

    • Worley says:

      You’ve put your finger on a lot of truth.

      The meta-edgelord is: There’s an apostrophe afer “others” in “rub others’ noses in …”.
      I.e., “noses of others”.

  61. ian parkinson says:

    [closer reading of comments shows same point made already]

  62. Steve Sailer says:

    Scott quotes venture capitalist Paul Graham’s 2004 essay “What You Can’t Say,” which is an important exposition of why so much hatred is directed at cognitive dissidents:

    “No one gets in trouble for saying that 2 + 2 is 5, or that people in Pittsburgh are ten feet tall. Such obviously false statements might be treated as jokes, or at worst as evidence of insanity, but they are not likely to make anyone mad. The statements that make people mad are the ones they worry might be believed. I suspect the statements that make people maddest are those they worry might be true.”

    He goes on:

    “When a politician says his opponent is mistaken, that’s a straightforward criticism, but when he attacks a statement as ‘divisive’ or ‘racially insensitive’ instead of arguing that it’s false, we should start paying attention. … Labels like that are probably the biggest external clue. If a statement is false, that’s the worst thing you can say about it. You don’t need to say that it’s heretical.”

    Why discuss “divisive” topics? Graham offers threes reasons:

    “Curiosity”;
    “I don’t like the idea of being mistaken”;
    “It’s good for the brain … Great work tends to grow out of ideas that others have overlooked, and no idea is so overlooked as one that’s unthinkable …”

    But Graham doesn’t want his readers to sacrifice their fabulous careers over this. So, he advises, don’t say anything in the open. But then, again:

    “The trouble with keeping your thoughts secret, though, is that you lose the advantages of discussion. Talking about an idea leads to more ideas. So the optimal plan, if you can manage it, is to have a few trusted friends you can speak openly to. This is not just a way to develop ideas; it’s also a good rule of thumb for choosing friends. The people you can say heretical things to without getting jumped on are also the most interesting to know.”

    True. But oral discussions among a few refugees in the catacombs, while better than complete inanity, are hardly as good for the country as a whole as written discussions and public conferences.

    • Aapje says:

      Fortunately we have the Internet now and the persecution is mostly done by individuals, not the government, so you can be anonymous and still discuss controversial issues.

      Unless you use your real name on the Internet, of course…

      • PedroS says:

        The only reason I don’t use my real name is that no one would take me seriously if they knew my owners call me “Rex, the overgrown Chihuahua”

    • Matthias says:

      Labels like that are probably the biggest external clue. If a statement is false, that’s the worst thing you can say about it. You don’t need to say that it’s heretical.

      This statement is itself obviously false; not all lies are created equal, and oftentimes the hereticality (from an orthodox point of view, dangerousness) of a lie is the most salient feature. Blood libel is no more false than astrology, but there are good reasons to respond to claims of blood libel with aggressive shaming or even censorship, while astrological claims merit at best an eye-roll (and not just because they are as obviously wrong as “people in Detroit are ten feet tall” – after all, plenty of laypersons do find astrology plausible.)

  63. Bugmaster says:

    This “whisper network” idea sounds great, but it suffers from some crippling issues.

    First of all, despite being somewhat secret, it is still incredibly dangerous. A single bad actor who pushes the “defect” button on the whole thing — for any reason — can obtain the power to instantly destroy anyone and everyone who is desperately holding down “cooperate”. This means that the scientists who do choose participate in it would have to be either very brave, or very foolish. This shrinks down the pool of potential scientists quite dramatically.

    Secondly, some topics of research are directly opposed to established doctrine; there’s just no way around it. If the Church says that the Sun goes around the Earth, and if you can never publish any research that states otherwise, then essentially the entire discipline of astronomy is now permanently shut down (plus or minus a few engineering applications). Same thing goes for evolution and biology. This problem is exacerbated by the fact that it’s very difficult to predict ahead of time which Church dogmas will lock humanity out of the next technological revolution, vs. the relatively harmless dogmas that can be safely skirted around. This gets even worse when you consider the fact that Church dogmas tend to fluctuate over time. The harmless research you’re doing today might well become heresy tomorrow.

    Thirdly, take a look at the list of those persecuted scientists once again. Galileo. Bruno. Bacon. They have many things in common, and one of those things is, essentially, martyrdom. Today, we can still occasionally point to those people and say: “You see this ? This is why you don’t burn people at the stake for disagreeing with you”. Would we still have space travel and genetic engineering today if everyone on that list stayed quiet and minded their own business and maybe whispered a few forbidden words now and again ? Well… maybe we would, it’s hard to tell for certain… But the chances get a lot worse with every major scientist who chooses to play it safe.

    This last issue, BTW, is the classic “tragedy of the commons” scenario. If every one of these scientists chose to maximize their own benefit, following a perfectly rational and sensible strategy, then our entire civilization would suffer dramatic losses. I realize this sounds cruel, but personally I’m glad that some of them chose to risk it all, and tossed the dice.

    • theofloinn says:

      Three names across 400 years? And only two of them scientists? And one of those really a proto-scientist.

      1. It was not the Church that said the Sun went around the Earth. It was the established science since before the time of Aristotle. The proofs were empirical and impressive. Geomobility, otoh, was based on religious faith (Pythagoreanism). The Church Fathers of old had implicitly absorbed this scientific consensus in their interpretation of scripture; and practice was not to deviate from the interpretations of the Fathers unless there was demonstration to the contrary, as Bellarmino wrote in his letter to Foscarino. So if the geostationary theory were falsified, then it would be legitimate to read the scriptures in question as metaphorical and admit that the old philosophers (and hence, the Church Fathers) had not understood them rather than to claim that what was clearly false was true. Thus, he told Galileo in their one-on-one that, while he could continue to teach Copernicanism as a mathematical model (“hypothesis” in the language of the time), he could not teach it yet as an established physical fact. Not without empirical evidence. Galileo agreed because he expected to have that empirical evidence Real Soon Now. It proved, however, frustratingly elusive.

      2. Bruno was not a scientist. He was a hermitic mystic who liked to use scientific talk to sound highfalutin, much like a postmodern deconstructionist using topology. Read his Ash Wednesday Supper.

      3. Bacon was a Franciscan friar in the early Middle Ages. The prohibition was actually laid on a colleague at his friary, but it was applied in general to all the friars. No publications outside the order without prior review and approval. Bacon’s illness and the imposition of other duties interfered; but the three works for which he is best known were written at the express personal request of the Pope, bypassing Bacon’s superiors; so there was clearly no general animus against his science. Further, nothing was done against his friends and mentors Robert Grosseteste (he of the scientific method, of optics, etc.) and Peter de Maricourt (rules of magnetism), so again, no general animus against science.

      As we used to teach in problem-solving: a successful theory not only has to explain what happened, but also must explain what did not happen. If Bacon was “perseculted,” then why were not Grosseteste and Peter, who works were arguably more important, not also “persecuted”?

      • Nick says:

        This is tangential to your point, but one thing that’s often missing from these discussions (including Scott’s post, unfortunately) is any attention paid to the arguments these people put forward for their beliefs, as apart from the beliefs themselves. We’re too quick to vindicate Bruno when Bruno had bad or Not Even Wrong reasons for his beliefs; we’re too quick to condemn the Inquisition when, as you show in your Ptolemaic Smackdown series, their scientific objections to heliocentrism were good ones. The worst of this is when we, for instance, celebrate Democritus’ atomism as if it has anything to do with modern atomic theory, or equate Lucretius’ clinamen with Brownian motion.

    • Drew says:

      First of all, despite being somewhat secret, it is still incredibly dangerous. A single bad actor who pushes the “defect” button on the whole thing — for any reason — can obtain the power to instantly destroy anyone and everyone who is desperately holding down “cooperate”. This means that the scientists who do choose participate in it would have to be either very brave, or very foolish

      Can you? If there’s a centralized inquisition, sure. Then there’s an authority who can do proper investigations and apply sanctions themselves.

      In our current system, the enforcement is all social. So, any revelation needs to be clear enough that an outsider can understand it. And it needs to be shocking enough that people feel virtuous for joining in the critique.

      So long as the whisper network has a couple layers of indirection, it seems pretty easy to make the objection boring or incomprehensible to outsiders.

      In the past, people used latin or math. Now we’ve got math, screen names and jargon.

  64. Soy Lecithin says:

    I don’t think you meant to imply otherwise, but it might be worth pointing out that none of the thinkers you list were atheists (except maybe Vanini). Some of their “heresies” were positively religious, rather than doubt or skepticism. Your example of an atheist monk whisper network gives an incorrect picture. I don’t think this changes your point, of course. It just bothers me a bit to see a broader issue of religious liberty framed as religion versus atheism. Mormons, Quakers, Kurds, and peyote cultists have the same (maybe dubious) claim on Giordano Bruno that the New Atheists do.

  65. blacktrance says:

    If you overestimate the threat and the necessity of the whisper network, you strengthen the authorities by retreating from public defiance. If there’s some social impulse to ban truths like “lightning comes before thunder”, the best thing to do is to confront it as early as possible, as long as you can get away with it. You can be on the lookout for a whisper network just in case, but the first thing is to make sure everyone knows the truth and proclaims it while they can. Maybe even if you don’t defeat the thunder-firsters outright, they’ll have to create their own bubble where the truth is banned, instead of spreading it to the rest of society.

    But while some have suffered from Stalinist or religious oppression, we’re not likely to encounter anything that pervasive. What people actually want to avoid is some office worker coming in with a superior look and making annoying remarks like “You thunder/lighting-firster sheeple!”. They don’t care about orthodoxy, but they care about people who won’t shut up. If Alice and Bob go to thunder- and lightning-firster meetings, no one will object if they don’t make an issue of it.

    • The Nybbler says:

      They don’t care about orthodoxy, but they care about people who won’t shut up. If Alice and Bob go to thunder- and lightning-firster meetings, no one will object if they don’t make an issue of it.

      You say that now. But when Eve comes in and starts gossiping about how she overheard Alice and Bob talking about their lightning-first beliefs, who gets punished… Eve or Alice and Bob?

  66. Ialdabaoth says:

    Here’s my biggest problem with this whole idea:

    Suppose I live in the antebellum South. I KNOW Slavery is an abhorrent institution. However, speaking out against it will get me lynched. I’m a public figure who is researching important things – say, about medicine and human biology – and I’m also frequently asked to justify slavery on racial / human biological grounds. If I refuse, I lose my ability to do good work. If I acquiesce, then a hundred and fifty years later, everyone remembers me as a horrible racist. Which do I do?

    • Tracy W says:

      Move north.

    • suntzuanime says:

      The question of whether to actively support the edifice of lies is a different one from whether to passively avoid contradicting it. In my opinion, at least, a public intellectual or scientist has a duty not to promote claims they know to be false in their area of discourse. However, if you want to be a medical/biological researcher in the antebellum South but don’t think the time is ripe to overthrow slavery, I think the advice would be to be obscurantist about anything that would directly contradict the prevailing racial theories, and try not to get into situations where you have to directly make pronouncements on them.

    • Said Achmiz says:

      Consider not caring about what people think about you a hundred and fifty years later.

      • t mes says:

        You may also want to consider how well a “I was just following orders” defense will work out at your war crimes tribunal. In that parable, the turn around came much quicker than 150 years. So even morality and ethics aside, its not always clear cut.

        • Anonymous says:

          You may also want to consider how well a “I was just following orders” defense will work out at your war crimes tribunal.

          That question boils down to “did my side win?”. If your side won, you’re scot-free. If your side lost, you’re doomed unless you can prove you were an active traitor to your side (and then you can still easily get treason as its own reward).

        • SEE says:

          Seems to have turned out okay for the NKVD, and rather poorly for anyone who failed to cooperate with them.

          • Tracy W says:

            If the NKVD are the Soviet secret police they purged themselves numerous times and two of the NKVD leaders, Yagoda and Yezhov, were killed in the Great Terror.

            And cooperation didn’t save you from the NKVD.

          • Anonymous says:

            Were they purged on the basis of the purgees doing something really bad during wartime? Or on some other basis, like their political reliability?

          • Tracy W says:

            @Anonymous: in the Great Terror people were purged for all sorts of reasons. There were quotas. They’d torture people until they signed a blank piece of paper. And if you were an authority naturally you were known to the authorities.

          • Anonymous says:

            Which means they actually did okay with regards to not being killed specifically for their war crimes, even if a few of them ended up dead for that. If the Soviets had lost the war (and were waging it against someone who likes victor’s justice trials, rather than the more straightforward Nazis), you can bet that pretty much every single one the vast majority of them that the victorious enemies caught would be found guilty.

            EDIT: I actually went and checked.

            On October 1, 1946, the International Military Tribunal handed down its verdicts in the trials of 22 Nazi leaders – eleven were given the death penalty, three were acquitted, three were given life imprisonment and four were given imprisonment ranging from 10 to 20 years.

            86.3% conviction rate is pretty solid.

          • SEE says:

            @Tracy W – Purged themselves, sure. Got held accountable at a war crimes trial? Not so much.

            Nuremberg was a sick joke, with agents of the regime that invaded Finland and Poland sitting on the bench that convicted people of “crimes against peace” for invading Norway and Poland.

          • Anonymous says:

            Nuremberg was a sick joke, with agents of the regime that invaded Finland and Poland sitting on the bench that convicted people of “crimes against peace” for invading Norway and Poland.

            I agree. In fact, I would regard it as a sick joke even the USSR were not involved. I don’t object so much to the execution the top Nazis, as to calling it “justice”, as if it’s not a blatant case of vae victis.

          • CatCube says:

            The Nuremberg courts provided me one of those great opportunities, where it turns out that a great historical figure has articulated my own thoughts far better than I ever could. Per Chief Justice Harlan Fiske Stone:

            Jackson is away conducting his high-grade lynching party in Nuremberg. I don’t mind what he does to the Nazis, but I hate to see the pretense that he is running a court and proceeding according to common law. This is a little too sanctimonious a fraud to meet my old-fashioned ideas.

            I think it might have been better to go with Churchill’s first instinct, to just execute the Nazi leadership out of hand.

          • Lillian says:

            It wasn’t just Churchill’s first instinct. If i recall correctly the Soviet stance was to dispense with the trials and just shoot the top ten thousand Nazis. It’s a very Uncle Joe approach to the problem.

          • Tracy W says:

            @Anon:

            Which means they actually did okay with regards to not being killed specifically for their war crimes, even if a few of them ended up dead for that.

            I’m sure they were deeply relieved to have been shot or hung in the 1930s rather than shot or hung in the 1940s.

            If the Soviets had lost the war (and were waging it against someone who likes victor’s justice trials, rather than the more straightforward Nazis), you can bet that pretty much every single one the vast majority of them that the victorious enemies caught would be found guilty.

            600,000 people are estimated to have died in Stalin’s Terror according to Wikipedia. Your own quote mentions 11 death sentences handed to Nazis post-WWII.

            That’s quite a disparity.

          • cassander says:

            @Lillian

            Stalin wanted them tried, then shot. Stalin was a big fan of show trials.

          • Anonymous says:

            @Tracy W

            Whether or not those 600k committed war crimes or not did not enter into the equation of their survival chances. Except maybe in a positive way, given how ruthless people might have a better chance of surviving the purge, by being the very people doing the purging, rather than the victims.

          • Tracy W says:

            @Anon: but that comes back to what I said at the start, which was that the NKVD themselves were purged numerous times.

            As far as I can tell the way to survive the Russian Terror was to be very lucky. Ruthlessness didn’t help.

          • Murphy says:

            @Tracy W

            The Nuremberg trials were only a small fraction of all the trials of Captured Nazis.

            For example there’s the Auschwitz trial

            https://en.wikipedia.org/wiki/Auschwitz_trial

            with 21 executed by hanging.

            One notable person from that list was a doctor Hans Münch who later became known as “The Good Man of Auschwitz”

            he didn’t take Tracy W’s preferred approach of leaving (assuming he could) and instead stayed. He was required to perform experiments on human subjects and to analyse tissues from executed victims.

            He also had 19 concentration camp inmates testifying in his defense because he treated inmates well and concocted fake “experiments” to keep inmates from being sent to the gas chamber.

            I remember hearing an old story, I can’t remember the exact specifics but it was about an old southern judge being asked about being part of a system that often imposed racist and unfair laws and shouldn’t he have just left if he was a good person. His answer was something like “if all the good judges left then only the bad ones would be left”

            If Hans Münch had simply moved somewhere else the primary outcome would have been the people he saved ending up in the gas chambers.

      • Doesntliketocomment says:

        Consider then how you think of yourself, that night.

    • RiOrius says:

      Waffle.

      “Oh, that’s outside the purview of my research.” “My participants aren’t demographically suited for that sort of comparison.” “I’m not focusing on that: instead I’m working on such-and-such.”

      If you want to be the best possible antebellum professor in the South, with the most clout and research funding, yeah you’d probably have to toe the line, including making claims that we today would see as bad. But if you just want to be a decent researcher, make some progress in your field, and help people, keeping your head down should be enough to keep you off the radar (or whatever antebellum idiom is appropriate).

    • 6jfvkd8lu7cc says:

      Drown people in details without saying any words they can write down without spelling mistakes.

      In case of Kolmogorov, intellectual difference between him and ideology people was huge enough that he actually managed to make it dangerous for them to speak to him: when they tried to claim that materialistically there are no random events, he answered that the state guarantees randomness of distribution of prizes in the state lottery (it was a combination of bonds, a lottery, and a tax — purchase was obligatory — so this randomness guarantee was relevant to every person working in a city), and this randomness is good enough for his mathematical work and its industrial applications.

    • John Schilling says:

      Suppose I live in the antebellum South. I KNOW Slavery is an abhorrent institution. However, speaking out against it will get me lynched.

      Nit: I’m pretty certain that if you’re a white person living in the antebellum South, speaking out against slavery doesn’t get you lynched. If you’re tactful about it, it probably doesn’t even get you excluded from polite society.

      For that matter, if you’re a black person living in the antebellum South, you’ve probably got a fair bit of latitude on that front, though there were plenty of other things that would merit a lynching and with enough overlap that the point is probably moot.

      The edge case would be a white Yankee visiting the south and tactlessly speaking out against slavery. That might get you lynched, though I’d have to research the issue.

      • The Nybbler says:

        Lynching was more of a postbellum institution, but at least one abolitionist was murdered by a mob: Reverend Lovejoy

        (note it wasn’t in the South)

      • Evan Þ says:

        I’m pretty certain that if you’re a white person living in the antebellum South, speaking out against slavery doesn’t get you lynched. If you’re tactful about it, it probably doesn’t even get you excluded from polite society.

        Given the frequency of mobs against Northerners who were even vaguely suspected of being abolitionists, and how the Quakers were all but expelled from the Winston-Salem area, I severely doubt that. At best, you could say that you hoped slavery might be done away with at some point in the very vague and distant future, like Rev. Robert Lewis Dabney, but nothing more.

      • AnonYEmous says:

        maybe he just meant lynched in a social sense, like people use it today

        kind of hard to tell though since they did actually lynch people

    • AnonYEmous says:

      If I acquiesce, then a hundred and fifty years later, everyone remembers me as a horrible racist. Which do I do?

      The one which has current-day implications. What do you care what people think 150 years in the future?

  67. Steve Sailer says:

    A big difference between pre-modern Christendom and the Soviet Union was the latter was far more monolithic and politically centralized. A medieval thinker who was on the outs with, say, the Pope or the Dominicans could often find a principality with a friendly monarch who would hire him as court librarian or who had a Franciscan monastery in his duchy where his cousin was the abbot. This meant that life was full of interest for the dissenter, since the winds of politics could send him on his travels once again, but it was a lot less hopeless for a dissident than in an 8,000 mile wide superpower with armed borders to keep its subjects in.

  68. Steve Sailer says:

    Galileo’s problem was in part that the Catholic Church was drifting away from science during his lifetime as part of the Counter-Reformation. Before the Reformation, the upper ranks of the hierarchy were intellectually elitist and didn’t much mind highbrow speculation about astronomy (e.g., Father Copernicus dedicated his 1543 book to the Pope).

    But over the next 80 years, the Catholic Church decided to respond to the anti-elitism of the Protestant Reformation by moving in a very populist direction. Catholic art (e.g., Caravaggio, Bernini) became more melodramatic and less intellectual than during the Renaissance (e.g., Raphael painted the School of Athens on the pope’s apartment wall in the early 1500s, but this glorification of non-Christian scientists and philosophers was less in fashion during the 1600s.

    Galileo got trapped by this drift in Italy away from scientific thinking. He had Cardinals who supported him, but they were getting fewer in number as the logic of the Counter-Reformation worked itself out.

    In general, this tendency for authoritarian tendencies to become more extreme and thus pester scientists who formerly were given a pass is a common one. Jared Diamond, for example, did yeoman service to the conventional wisdom in the past, and has a good chance to make it to his grave without being subjected to a massive struggle session, but if he were a younger man he’d likely to suffer Watsoning at some point at the hands of the new Red Guard.

    • James Green says:

      Even after some googling I’m having trouble understanding the verb Watsoning, care to enlighten me? What happened to Watson (I think he was tar and feathered from the context, but there are other possibilities)? Which Watson is it named after, there are a lot of them?

      Edit: Thanks Evan, I had no idea that guy was still alive.

    • theofloinn says:

      Actually, a big source of the friction was Bellarmino’s insistence on empirical evidence for geomobility vs. Galielo’s insistence that it be taken on faith. The Church, alas, was firmly aligned with what was then the scientific consensus, the “99%” of scientists who agrees that the objections to the dual motions of the earth were profoundly unanswerable, given what was known or assumed by natural philosophers at the time. They weren’t about to overturn all of physics just on the say-so of a jumped-up mathematician who had more epicycles in his model than Peuerbach’s then-current version of Ptolemy.

      When Lembo, Galileo, and others observed the phases of Venus, that put paid to the Ptolemaic model and the Gilbertian model. (There were about nine models then in contention.) But astronomers then flocked to the Tychonic or the Ursine models because they not only matched the outputs of the Copernican model (being mathematically equivalent) but also fit the data better by explaining the lack of parallax and the lack of Coriolis effects.

      If you think it’s easy to demonstrate that the earth possesses a dual motion, spinning like a top while careening like a dervish around the sun without leaving the Air and Moon behind and without all of us losing our balance and staggering around like a politician dodging a question, we’d all like to hear it. But remember: no appeals to authority, just whatever evidences available in the early 1600s.
      +++
      As for evolution of new species: as long as people thought the world was eternal, there was no reason to suppose its furnishings were not also eternal. Jews, Christians, and muslims might believe the world had a beginning in time, but the default philosophy in science was Aristotelianism, and that held that the world was eternal, or at best, cyclical. Certainly, there were no species known to the medievals that were not known to Aristotle. For that matter, there were none known to Darwin that were not known to Aristotle, with the exception of a) species in regions that Aristotle had not known, and b) finer distinctions of species that Aristotle had not made. No one will look for an explanation of the origin of species if they have never seen an example of a new species originating. Even so, Aquinas did make one passing comment in the Summa theologiae, that IF any new species should ever appear, it would do so through the powers inherent in matter from the beginning; i.e., through natural processes. This was a follow-up on Augustine’s concept of rationes: “reasons” embedded in nature for future unfoldings and developments, often taken as the embryo for the concept of evolution (originally: the unrolling of a scroll).
      +++
      Bruno was not an astronomer, and his interest in cosmology was only insofar as it supported his hermetic mysticism. Read his Ash Wednesday Supper, a Seven-Way “Dia”logue. He clearly does not understand astronomy and is even contemptuous of Copernicus.

      One needn’t multiply entities by imagining some sort of “atmosphere” hostile to scientific curiosity when it is only necessary to imagine an atmosphere hostile to Bruno. Bruno was no more burned for his inchoate cosmology than Lavoisier was guillotined by the rationalists for his chemistry. Look up the cosmological thoughts of Nicholas of Cusa and what happened to him.
      +++
      My old history prof once cautioned us that when we studied the Battle of Salamis, we should do so in the expectation that the Persians might still win. IOW, you have to read history from pastward, and not looking back over your shoulder from the future.

      • quaelegit says:

        If anyone wants the long version of the first section, check out TOF’s amazing essay on The Great Ptolemaic Smackdown!

        ++++

        > for that matter, there were none known to Darwin that were not known to Aristotle, with the exception of a) species in regions that Aristotle had not known, and b) finer distinctions of species that Aristotle had not made

        What about microorganisms that people started finding with microscopes?

    • Douglas Knight says:

      You describe the change in the Church in maybe 3 different ways, implying that they are all the same, or consequences of each other. They look really different to me.

      The relevant changes are authoritarianism and centralization.
      Was this due to Protestantism?
      Maybe this was the result of copying Protestant authoritarianism, but not Protestant decentralization.
      A simple explanation is just fear. Fear that any dissent would snowball like Protestantism. And after Bruno fear that even scientific dissent might be tied into heresy.

      Maybe the Church copied Protestant populism, and maybe that’s what you’re seeing in the art, but is there any connection to the other trends?

    • Conrad Honcho says:

      What do you think the “new Red Guard” would find objectionable about Diamond?

      My objection to Diamond is that his thesis proves too much. Or, rather, “proves” the exact thing he’s arguing against. My interpretation of his thesis is that Europeans did better at civilization than Africa because the flora and fauna in Europe were, “because environment,” better suited for civilization than those in Africa. Europe had domesticable horses and wolves that could be turned into dogs because “the environment” made them smart and less aggressive where Africa had undomesticable zebras and hyenas because “the environment” made them less intelligent and more aggressive (ignore that people have, in fact, domesticated zebras and hyenas).

      It seems to me that if you accept this premise, that for some reason the environment of Europe makes for smart, temperate animals useful for civilization and the environment of Africa makes for less intelligent, intemperate animals not useful for civilization, one could then note that “man is an animal shaped by the environment,” cut out the several hundred pages of Just So stories in his book and make one very short and very racist screed.

      Is that what you would expect to happen to a young Jared Diamond?

      • herbert herberson says:

        I think Diamond over-simplifies, but if I recall correctly he has a good explanation for the distribution of domesticable animals that explains why they were in Eurasia but not Africa and the Americas (that I think is accurate):
        – in Africa, we evolved alongside megafauna from the very start, so they had the time to develop a deep-seated instinctual fear of hominids that prevented domestication
        – in Eurasia, we met them relatively late in our evolution, so they did not have the time to develop these fears. However, at that point, we still weren’t modern humans and therefore didn’t have the technology to wipe them out.
        – in North America, the first humans to meet the megafauna were fully modern humans who had sufficiently potent weapons and hunting techniques to, along with the concurrent deglaciation, cause a mass-extinction. There were, for example, horses in North America ~10,000 years ago. Some megafauna survived, but the selection process there rewarded aggression and skittishness, aka the same traits that discourage domestication.

        As to the original point, where Diamond gets both weaker in general and more vulnerable to accusations of un-wokeness isn’t the Eurasia vs Africa/Americas stuff (which was never that hard a question anyway) but rather his attempts to explain why Europe dominated the rest of Eurasia. Comes off as apologetics for imperialism to some, reminds others of colonial pseudoscience of why people from colder climates are more industrious (I don’t think it actually crosses any lines there, personally, even though I also don’t buy his arguments at all)

      • bean says:

        @Conrad
        The counterpoint to that theory is that evolution had a lot longer to work on dogs and horses vs hyenas and zebras than it did on European vs African humans.

        • Nick says:

          This. I do want to note, for historical interest, that environmental explanations were one of the commonest ancient explanations for differences among peoples. See chapter 3 of Kennedy’s book on this.

        • Conrad Honcho says:

          Counter-counterpoint, if it’s been 50,000 years out of Africa, well, we’re not talking about diverging into different species. Just a change in some characteristics. Every breed of dog differentiated in less time than that.

      • reasoned argumentation says:

        A young Jared Diamond published a paper about testicle size differences across races – i.e., a feature that’s under strong selective pressure that varies depending on the mating style of the race in question.

        https://www.nature.com/nature/journal/v320/n6062/abs/320488a0.html

      • hyperboloid says:

        @Conrad Honcho

        Europe had domesticable horses and wolves that could be turned into dogs because “the environment” made them smart and less aggressive

        Your deep, and subtle understanding of biology never ceases to amaze.

        African animals are not, on average, less intelligent than Eurasian animals. In fact the opposite appears to be true. Africa is the native home of elephants, every extant species of great ape, save for the orangutan; including, I might add, the genus Homo. And though you may not want to accept this, you sir are a Homo. If anything, it seems that there is something about the east African environment that is expectantly suited to the development of high mammalian intelligence.

        As to why there are more domesticable animals in Eurasia than Africa, there are to three possible explanations that come to mind:

        1. Eurasia is bigger, and has more species of larger terrestrial herbivores, and omnivores.

        2. All of the large domesticatable herbivores in Eurasia have a particular kind of follow-the-leader dominance hierarchy based herd structure.With these species It was very easy for humans to take over leadership and begin driving the vast herds that provided the raw material for domestication. This sort of behavior is more common in animals with long migratory paths, and because of it’s geography Eurasia has more animals with very long migratory paths. For example the longest migration in Africa is that of the plains zebra, about three hundred miles round trip across Namibia and Botswana, in comparison caribou can migrate three thousand miles annually.

        3. As herbert herberson rightly points out, humans are an invasive species anywhere outside of Africa, from an evolutionary point of view we’re basically talking bipedal kudzu. As such only animals in Africa actually co-evolved with hominids, and thus have developed behaviors to protect themselves from human predation. Behaviors that are likely to make them hard to domesticate.

        • Conrad Honcho says:

          I’m not saying this was my understanding of biology. I’m saying that was my understanding of Diamond’s argument.

          Thank you for the clarification, though, that was interesting.

  69. James Green says:

    When I was about 7 I started questioning the orthodoxy of Santa Claus. By the time I was about 9 I had learned to stop discussing my beliefs. And when I was about 10 my parents confessed to me that, yes, it had all been a sham the whole time. I was an eldest child.

    I find it hard to think of my 8 year old self as an edgelord though.

    —-

    Personal bugbear: saying such-and-such a person needs no introduction, and then introducing them right after that!

    Galileo in this case. I understand that it is really difficult not to do it, but I just want to get this off my chest.

  70. Tedd says:

    Previously on LessWong.

    (This is a great post and I’m in favor of it! It is much more than a rehash of the essay I’m linking. I just want to point people to prior art and maybe get them to read the rest of the sequences. Scott rewriting and expanding on them is a great good, but I don’t think he’ll get to all of them.)

    • Douglas Knight says:

      The two essays appear to me to be entirely disjoint. Servetus’s views on the Trinity were not entangled with his science.

      • Tedd says:

        Section III of Scott’s essay seems pretty much identical to my link, except with less assumption that it’s being done intentionally. It is, in fact, specifically making the point that one’s views on the Trinity *are* entangled with one’s science, via one’s epistemology.

  71. Steve Sailer says:

    Stalin occasionally responded to direct challenges by intellectuals and artists in a non-lethal fashion.

    Bulgakov, the author of “The Master and Margarita,” was a Soviet playwright who kept getting censored. He wrote a letter to Stalin in 1929 asking permission to emigrate. Stalin called him up and explained that he had to censor his works, but that he admired Bulgakov, so Stalin got Bulgakov a job as a theatrical director and kept him from getting arrested or fully purged throughout the 1930s until he died of natural causes in 1940.

    It’s a weird story, but a small number of talented people with immense courage managed to challenge Stalin directly and survive. Even Stalin didn’t feel like being Stalin all the time, evidently.

    • Bugmaster says:

      That’s a fair point, but I think that if your survival strategy relies on Stalin feeling magnanimous that day, then you’re still basically screwed (on average).

    • srconstantin says:

      Also remember that nothing whatsoever happened to Pasternak. I don’t know how! But he survived!

  72. ilkarnal says:

    Scientists tend to get in trouble in proportion to how political they get. I like some politically active scientists (meaning scientists who talk to the media, basically) and dislike others. I shudder at the way many good men are ruined. But the problem isn’t the ‘system.’ That’s like having an avalanche fall on you and saying the problem is gravity. I would argue that suppression of political opinions is the essence of politics. This is life – brutal competition, with any cooperation that lasts being cooperation TO COMPETE BETTER. To abstain from this sort of thing, which is considered ‘icky’ by many in this area, is in the end to abstain from life.

    Politics is war by other means. The answer to political problems is to get better at war. Scientists are often naive children on a battlefield, acting like their enemies are family or friends. They do not understand the essence of politics – that the other side seeks their destruction just as surely as if they were lobbing shells, they are merely restricted by a powerful protection racket. Each side seeks dominion for themselves and ruin or subjugation for their enemy, all the same.

    Conspiracies can have utility, but sooner or later the blade needs to be wetted, literally or metaphorically. If there’s no way for you to bring your enemy to ruin, sooner or later he will find you and bring you down. This is not a question of truth against lies, but of people against other people with fundamentally different interests. The instant a person is saying something true, his statement is not just about truth but also about him. And the ‘about him’ part is the most salient part when someone is worried about being trampled into dust by this fellow and his compatriots. And everyone except his compatriots has that worry. Whether they recognize that or not, their instincts are not as idiotically naive as they are.

    The question of how to be good at politics needs to be approached methodically, systematically. Operational security is surely some part, but nothing like the whole. ‘Rationalists’ and those adjacent need to learn how to fight in this realm effectively. They need to understand that the ’emotional’ side of arguments is not noise but signal, that that signal is coming from fairly well tuned evolved heuristics, and that embracing a value-system that does not emphasize caring about this sort of thing, caring about their own proliferation, means winking out of existence pretty quickly. The truth is a tool in this game of proliferation and annihilation, a means and not an end. Elevating truth in this naive fashion actually just condemns it to relative darkness. If you use truth as a hammer to smash your foes’ skulls, you will have great riches and forge many hammers. If you sit around worshiping the hammer, someone will come along and bash you with a rock, and in all likelihood the hammer will garner a great deal less respect.

    TL;DR don’t avoid politics, git gud.

    • John Nerst says:

      This objection comes up occasionally, and I think most here have heard it already. It doesn’t gain traction partly because to exhort people to be more dishonest in pursuing their interests when their interest is for everyone to be more honest is a bit paradoxical. How do you do that?

      It’s a valid point but more cynical that you strictly need to be. Moloch can be kept at bay, but that needs work and everyone can’t be consuming social capital (defecting). It needs to be produced too. The modern world exists because people did.

      I tried to re-read Foundation recently, and this reminded me of this quote, brilliant in its simplicity:

      Q. Can the overall history of the human race be changed?
      A. Yes.

      Q. Easily?
      
A. No. With great difficulty.

      • Peffern says:

        I think In Favor of Niceness, Community, and Civilization is relevant here.

      • ilkarnal says:

        Honesty and dishonesty really aren’t that important. It’s how you say it, not what you say. And it’s what you look like and what you have as much as it is how you say it. Trump didn’t win by lying – his lies probably hurt him moderately. If he told the truth in the same manner he told lies, he would be ahead. It is how he said things, not what he said. And then, what he looked like and what he had, what he could marshal as social proof.

        Look, deep down everyone is a preening arrogant bastard. There are varying degrees of this, but everyone has it. If you were told – you’re an ugly worthless nothing, would that offend you? Why? Because some very significant part of you is not a truth-deducing and projecting machine, but wants to be a beautiful valuable important thing. Nerds don’t lack this essential human element, they are just often much worse at fulfilling it. They’ve got other things to concern them, and I would agree that a lot of those things are important, and a lot of people completely devote themselves to beautiful-valuable-important and lose out on the outside world. But clearly, beautiful-valuable-important remains VERY CENTRAL to the value-system of even nerdling humans.

        Nerds will countersignal effort put towards beautiful-valuable-important, but while it is easy for people to say ‘Oh, it’s not that important to be beautiful..’ Essentially everyone feels something hurt inside them when they are called or considered ugly. People might say ‘oh, I don’t need to be important‘ but something inside them still hurts when they hear or realize that they are irrelevant.

        I’m not saying that you should surrender to the value-structure of this society. I’m saying if you want to make something better, you sure as shit need to pay attention to the nature of value-structures and your own position in them, and you will need at least moderate success within this value-structure to move to create new and successful, lasting ones. This notion that you can ignore such childish and superficial things is actually just nonsensical. It implies not a defeat of the existing value-structure but a complete surrender to it.

        What is the aesthetic of the future? Recently, this battle has turned out not necessarily to our advantage. The answer is not to retreat. There is nowhere to retreat. The answer is not to surrender, because in the long run success within this structure is not in our blood. The answer is to fight, and to build, within this value-structure because that is what now exists as the thing we can fight in, and then as success piles on success – fate willing – a value-structure that subtly and then not so subtly advantages us.

        You care about this fight – caring about the degree to which you win is highly variable, but caring about losing is universal. Too much ground has been sacrificed by our forebears, who could not imagine losing and thought they were just raising their boot slightly off a poor helpless victim’s neck. The boot isn’t quite on the other neck yet, but we’re getting there. The first to feel it are those who are at the middle-bottom of the ruling coalition that now falters.

        I don’t like the whole old, failing ruling coalition. I like some narrow parts of it very very very much. Those parts are the first to be sacrificed by its fall. I DESPISE those incipient coalitions that rise to take its place – they represent a bottomless downfall. The goal is not to reverse or restore the old ruling coalition. It is to recognize that the outside forces that seek to tear it down are not rescuers of those who struggled within the old ruling coalition, but enemies who seek the defeat and subjugation of all within that old ruling coalition. Luckily they are weak, only threats to the heirs of glory when they sit befuddled and blinded.

        The old ruling coalition must have its leaders cast out, and proceed in a new direction. This can only be accomplished through politics. There is a feeling in the air, that those narrow parts of the old coalition that I mentioned earlier are waking up to the impending disaster. I want to push forward on this as hard as possible. This is where hope lies.

        • Aapje says:

          I disagree that it’s so neat where the oppressor becomes the oppressed and vice versa. Quite often, people who were under the boot of the old coalition end up under the boot of the new coalition.

        • carvenvisage says:

          Look, deep down everyone is a preening arrogant bastard. There are varying degrees of this, but everyone has it. If you were told – you’re an ugly worthless nothing, would that offend you? Why

          Your premise is that if someone explicitly, clearly, attacks you (to a relatively extreme/blatant extent), that having some kind of emotional reaction makes you “arrogant”.

          If someone summons up bile and sickness in themselves to convey it over the empathic link you generously extend them, to hear what they have to say and think, you’re craaaaaaazy if you object.

          The foundation offered for your worldview is the idea that self defense is arrogant, that an attack isn’t an attack.

          _

          This is a good illustration of the importance of questioning. Heliocentrism is a side effect. The main thing is discoveries like ‘if someone attacks you for no reason, disliking it doesn’t prove them right’.

          Given humans’ slow starts, and their chance of landing in a bad starting zone, basic essential things like that aren’t self evident. Grasping fearlessly after truth is not primarily a matter of reaching great heights, but of establishing basic integrity and sanity, from the depths of weakness and ignorance we all share as our starting condition.

    • tsutsifrutsi says:

      And if you’re on the autistic spectrum and will never be able to successfully bash anyone’s political skull in—or realize that your actions might be interpreted as attempts to do so—but will successfully prove many a math theorem (some of which might be interpreted as attempts to so bash)?

      • John Nerst says:

        If you (general you) don’t realize that many things are really about power you are indeed stupid. If you don’t realize that some are not you are stupid as well.

        Sadly the incentives are asymmetrical because PD type situations are a thing.

      • sconn says:

        Then you need at least one trusted friend to help you navigate this stuff. No one person has all the talents necessary to handle the world; relying on one another is necessary. Parents, colleagues, and academic advisors often take on the role of warning naive brainy people “hey, don’t say X, phrase it like Y, otherwise you are inadvertently making a political statements.”

      • Brad says:

        I find it curious that the essay didn’t say anything about the autism spectrum. I’m not someone that likes to armchair diagnose people, much less people that lived centuries ago, but when you see things like:

        If you think it’s impossible to be that oblivious, you’re wrong. Every couple of weeks, I have friends ask me “Hey, do you know if I could get in trouble for saying [THING THAT THEY WILL DEFINITELY GET IN TROUBLE FOR SAYING]?” When I stare at them open-mouthed, they follow with “Well, what if I start by specifying that I’m not a bad person and I just honestly think it might be true?” I am half-tempted to hire babysitters for these people to make sure they’re not sending disapproving letters to Stalin in their spare time.

        and

        The idea that everything in the world fits together, that all knowledge is worth having and should be pursued to the bitter end, that if you tell one lie the truth is forever after your enemy …

        it makes you think that the essay should be thinking about a neurodivergence angle and how it fits in.

        • Edward Scizorhands says:

          I was going to say “of course he talked about autism” but looking back he just said things that were obviously the description of autism and my mind put it together.

          The autistic people will never figure out the unspoken rules that we aren’t allowed to talk about, especially when they are in the progress of changing.

          • Edward Scizorhands says:

            Incidentally, my number one job stress is that some of the written rules are supposed to be ignored, and some of the written rules are supposed to be followed, and I don’t know how to figure out which is which, or even know how to safely ask.

            I have a really nice job if you put this one thing aside, but this one thing frequently causes me to think about quitting, so I’d love to resolve it.

          • Brad says:

            I’m not sure if this is the place for it, but I’ll go ahead and say it anyway:

            I think there’s a lot of room for making accommodations for people with autism. But the best way to make that case isn’t to suggest that the way of doing things that would be best for autistic people is actually far superior to the way that works for non-autistic people and to more than hint at that the way that non-autistic people do things is not just suboptimal but maybe evil. And why don’t you guys just do the reasonable thing, which is our way.

            We like our unwritten rules and our polite fictions. They have real value for us. Some acknowledgement of the fact that there are trade-offs here would go a long way towards building the trust necessary for compromise.

          • carvenvisage says:

            @brad

            Even if you want to warp everything around your particular dishonest way of living, total cultural hegenomy, just for your own mildest improvement to comfort, you still don’t have to be shitty about it. -If a 70lb woman walks into a martial arts gym populated on avg by 180lb men, she isn’t expected to lift as much weight as anyone else, and there is no reason someone who is socially retarded should be treated worse.

            ..but if someone is just a no good believer in truth, they will have to be broken on the wheel, that’s different..

            Your comment is addressed at the latter crowd with the pretense (or, generously, confusion) of being addressed at the former. “I know things are hard for (actual) autistics, but could they at least not be frustrated and confused about it, so I can feel like making.. deign to make, an effort?” No, of course not, you know that. The issue is that they don’t understand/aren’t gracious with social things.

            So it pretty much follows that the whole entire point of your comment was to equivocate objecting to bullshit with autism.

        • Nornagest says:

          Eh… autism certainly lends itself to a particular type of that, but I know plenty of neurotypical people with more curiosity than tact, too. I think it’s at least 50% culture.

          • Brad says:

            There’s certainly lots that could also be said about cultures that undervalue tact, kindness, and understanding and celebrate abrasiveness, but I don’t think it could be added in a reasonable number of words to this essay. Whereas a paragraph here or there on autism and scrupulosity and how they interplay with these forces could have enhanced the essay without dramatically changing it. In my opinion of course.

    • Emanuel Rylke says:

      99% of the time this kind of thinking will slowly corrode you to dust. But in the 1% rest it is vital.

    • Tracy W says:

      I would argue that suppression of political opinions is the essence of politics.

      I’d be interested to hear this argument and how it accounts for the longevity of disputative organisations like the British Parliament or the Swiss Cantons.

      ;Scientists are often naive children on a battlefield, acting like their enemies are family or friends.

      Funny then that they’ve managed to extract millions in funding, the support of public education, recognition and fame.

      If there’s no way for you to bring your enemy to ruin, sooner or later he will find you and bring you down.

      We can see this in the history of France and England. They were at each other’s throats for centuries but eventually one got the upper hand and brought the other down.

      Or the UK and the USA: the Brits couldn’t bring the American Revolutionaries to heel so the Yankees destroyed the British state. Burnt down Big Ben, disposed the monarchy and lugged the Crown Jewels back to the Smithsonian.

      And the ‘about him’ part is the most salient part when someone is worried about being trampled into dust by this fellow and his compatriots. And everyone except his compatriots has that worry.

      Based on past performance I think an American President could console themselves with thoughts of their future lucrative public speaking career.

      • sconzey says:

        the Yankees destroyed the British state. Burnt down Big Ben, disposed the monarchy and lugged the Crown Jewels back to the Smithsonian.

        I know this is hyperbole, but the US did pursue a strong anticolonialist policy postwar, with the Suez Crisis as an archetypical example of the relationship between the two nations at that point.

        And as any Brit or Frenchman will tell you: it ain’t over yet!

        • Tracy W says:

          Anti-colonialism is hardly bringing your enemy down. Indeed, from an economic viewpoint, it’s probably beneficial for a country to lose its colonies.

          As for “it ain’t over yet” that is of course true in and of itself but that way lies an un-disprovable hypothesis.

      • armorsmith42 says:

        > Back to the Smithsonian

        I just want to point out this particularly wry bit of ironic argument. To those who didn’t see it, look up the original funding source for the Smithsonian.

        • The Nybbler says:

          Yet, for some reason, the only Crown Jewel there is French.

        • quaelegit says:

          “A British guy bequeathed his wealth to the U.S. to build a museum” doesn’t strike me as ironic. Like yes, it’s a wealth transfer from Britain to the U.S., but a private citizen’s wealth bequeathed voluntarily.

    • albatross11 says:

      The problem happens when you have something that’s both a political controversy, and also a question of fact whose answer matters.

      Is Saddam really developing nukes that will threaten our safety? Do vaccines really cause autism? Is human action really changing the climate?

      Those are factual questions, but they have a huge amount of political stuff tied up with them. The political struggle often overwhelms any attempt to answer the question of fact with logic or data, and we end up making really bad collective decisions as a result.

  73. hnau says:

    One thing that bothers me about this post is how the “at-risk thinkers” category blurs the line between “people who advance truth in a particular area” and “people who like to question received wisdom”. In my experience the two are largely orthogonal.

    Many of the greatest scientists of [whatever we’re calling this period] were utterly orthodox. Many other great scientists– including some of those you cite– were way into the occult, or alchemy, or spiritualism, or any number of other weird and not-strongly-correlated-with-truth belief systems. In any era we can find plenty of thinkers who are geniuses in one respect and crackpots in another.

    Being prone to defy conventional wisdom may be statistically helpful for achieving Great Discoveries in certain contexts, just because those tend to require being at the edge of the ideological bell curve. And certainly it’s good for societies to keep that kind of person around and not burn them at the stake. But if you try to parlay that into a judgement of “this kind of person will have lower-than-average expected distance from the truth in any given area”, then you might be disappointed. Selection bias is almost certainly at work here.

    • theredsheep says:

      I want to second this. I’m kind of confused about the whole “in favor of truth” thing being used to describe people who keep magic books. I think it’s more that some people just demand independent thought, and sometimes they’re very right, and sometimes their beliefs are simply bizarre and stupid, and sometimes both at once, but they’re so convinced of their own genius that nobody can gainsay them. Aside from Newton’s passion for alchemy–well after most other people were rolling their eyes at it–wasn’t it Linus Pauling who started the bizarre idea that gorging on vitamin C will make your immune system invincible? Jack Parsons: brilliant rocket scientist, dedicated practitioner of black magic, got snowed by L. Ron Hubbard of all people. Et cetera.

  74. void_genesis says:

    Surely comedy plays a pivotal role here in providing a socially acceptable space to explore the edge of socially unacceptable ideas? You can often get away with obviously joking about something that you could never say in seriousness, and the more people hear the jokes the less taboo the topic becomes, until eventually it breaks out into common sense.

    • CatCube says:

      The problem is, going back to the thrust of the OP, is that being successful at comedy takes a very high degree of social skill. You have to be especially savvy when dealing with very controversial social issues. Chris Rock had a high degree of success riffing on the N-word…Michael Richards less so.

      • eh says:

        Comedy as practiced by actual comedians and broadcast to huge audiences is hard, but comedy as practiced by a group of friends making jokes like “hey, look, it’s the thunder” when a desert course accidentally arrives first is not hard.

        Most people can probably think of examples in their lives where comedy has been used to obliquely approach a sensitive topic. I interpreted void_genesis as having meant this kind of low-key plausible deniability.

      • Aapje says:

        @CatCube

        Chris Rock also has a trait that results in him getting the benefit of the doubt for ‘racial humor’ much more than a person who doesn’t have that trait.

        In our society, on some topics the people who most clearly see a specific fault in the popular narrative due to the typical life experiences for their group, are also the people who get the least leeway in criticizing the narrative, directly or through humor.

        Since people often judge prevalence by how many people speak out and often get offended and try to suppress claims that don’t fit the narrative, you can easily get self-reinforcing narrative + taboo combinations. So people can come to think something is false because they rarely hear about it. Then the assumption is often that the people who do talk about are lying and have nefarious goals, so most people don’t talk about it as to not be accused of being nefarious. This then leads people to conclude that it’s not a real issue as they hear little about it and see only a few antisocial people speak out (because the pro-social people tend to care a lot about being accused of being nefarious).

        What also often happens is that society (self-)segregates into subgroups who each develop their own narrative + taboo combination, so then we get bubbles with their own realities.

        PS. I don’t think that Michael Richards was actually riffing on the N-word. AFAIK he attacked loud audience members with racial slurs. That’s not ‘a bit.’

        • CatCube says:

          Chris Rock also has a trait that results in him getting the benefit of the doubt for ‘racial humor’ much more than a person who doesn’t have that trait.

          Knowing that is part of the “social savvy” that I was referring to.

          And I think that Richards was under the impression that his attacks were “jokes.” Comedians going after audience members that offend them (hecklers, interruptions, etc.) can be pretty vicious. However, he tried to freestyle racial slurs and it (rightly) bit him. Daniel Tosh ran into a similar wall when somebody objected to a set on rape, and he made an extremely poor extemporaneous “joke” about the heckler getting raped.

          • Conrad Honcho says:

            I’d say Lisa Lampanelli is the better counter example to Richards. Her whole act is “insult comic who will totally GO THERE.” One cannot be in her audience and get mad about being called an ethnic slur because that’s her act. If that sort of thing bothers you, you don’t go to her show. Richards…not so much.

        • Aapje says:

          @CatCube

          Knowing that is part of the “social savvy” that I was referring to.

          Yeah, but my point is that a white person who is just as socially savvy as Chris Rock still doesn’t have the same ability as him to get away with certain jokes.

          If we would wave a magic wand and make everyone equally imperfectly socially savvy, you’d still have people who don’t realize that certain jokes can only be made if you have a certain immutable trait. So you will have persons who make that joke while being oblivious that it can only be made by some people. Then if that person happens to have the right trait, he will get lucky that his obliviousness is not punished by society, while a person who is identical except for having the wrong trait, will get unlucky to have his obliviousness is punished by society. Neither of these people is any wiser than the other or did anything different, yet one gets punished and the other doesn’t.

          So there is a fundamental imbalance here that is separate from a person’s social ability.

          As for Richards, he himself said that:

          “Somebody interrupted my act,” he remembers, “and said some things that hurt me, and I lashed out in anger … It was a selfish response. I took it too personally … I should have been working selflessly.”

          That doesn’t sound like an attempt at humor, but rather extreme anger making someone use the most hurtful words he could come up with (which doesn’t necessitate believing that it’s right to refer to black people with the N-word, but merely the belief that it is going to be very hurtful to most of them, combined with ‘red mist’).

        • A friend told me that after she went on hormone replacement therapy, she felt more empathetic and nurturing. I think this counts as one of those traits that allows someone to state unacceptable truths. I certainly wouldn’t go around in polite society suggesting that the amount of estrogen in one’s body can maybe affect the way one thinks.

  75. t mes says:

    The dog whistle is getting louder and louder… I think your position is wrong. Maybe the status quo isn’t dialed in perfectly but I think the evidence points to the “Truth” being closer to the mainstream consensus. I don’t have access to any information you don’t and I use the word “think” for a reason but your side is constructing narratives out of incomplete/ify data and engaging in storytelling as least as much as the other side.

    • Scott Alexander says:

      I don’t want to lie and say I don’t have specific things in mind when writing this. But I think this post stands on its own as a generic claim about why white lies can be bad at the social level.

      • t mes says:

        It does stand on its own just fine. Nevertheless, we still both know what the other is talking about (more or less). So consider it a critique from the other side that has pierced your whisper network. Maybe the thunder does come first for some of these unspeakable issues.

        • Scott Alexander says:

          I think part of the function of whisper networks is to hold debates on these issues – that was kind of what I meant to point to with my Jerusalem example. You can’t trust any debate outside of a whisper network because you know one side has its hands tied behind its back, but a debate within a whisper network can actually be productive (in either direction).

          • Evan Þ says:

            Ideally, yes. But, that depends on the good debaters showing up to the whisper networks instead of steering clear of them, on people being sincere there instead of keeping one eye out for spies, on people recognizing that their interlocutors are sincere instead of just unwoke, and on a lot of other things.

            And also, it depends on the whisper networks actually being able to produce good science. If you wanted to make an argument about evolution in the Middle Ages – well, you couldn’t, because the evidence wasn’t there. Even if you somehow knew exactly where to dig for fossils, your whisper network didn’t have the money to send out an expedition to East Africa. It’s the same thing if your forest band of radicals can’t afford to send out an expedition looking for Jerusalem.

          • Tracy W says:

            @Evan P, was this truly the case in medieval Europe? I mean I know Darwin went to the Galapagos, but the evidence he cited included European animal stock and pigeon breeding and some basic stuff about the number of offspring a female animal can produce.

          • Evan Þ says:

            @Tracy W, sure, and you could run Mendel several centuries earlier if you had a good pea garden and some time. But from there you need to wave your hands and extrapolate a lot to get to natural history, especially before geology’s gotten off the ground.

            (Pun unintentional.)

          • t mes says:

            I totally agree. Maybe we should set up the first “great whisper debate” for those who are interested in amicable discourse. Would be fun experiment if the terms were to discuss on a dog whistle and subtext basis only.

          • Steve Sailer says:

            Debates in public are, on the whole, much better than debates in private: e.g., Jensen vs. Flynn in public advanced science more than most private debates.

          • Protagoras says:

            @Tracy W, Selective breeding tends to show that evolution could produce species, but in order for it to actually be the explanation for the species we see, the Earth has to be extremely old, which was not generally believed to be the case in the middle ages. Also, just recognizing that it is a possible explanation doesn’t on its own rule out other explanations. So the Galapagos stuff is extremely important because the geographical distribution of species makes sense on Darwin’s account, but is quite mysterious otherwise. Looking only at animals from one region of the world it is much harder to see conclusive evidence for evolution.

          • vV_Vv says:

            If you look at medieval or even Renaissance paintings of Biblical stories, or Greek-Roman myths, or drawings of exotic locations, they all tend to resemble the artist’s local environment in many details: not only in anthropic elements such as architecture and clothing styles, but also in natural elements such as plants, animals and general landscapes.

            Before the Age of Sail, very few people traveled long distances, and therefore everybody imagined the world pretty much as their backyard. People with some education might have been vaguely aware that certain regions were warmer or colder, and might have vaguely heard of specific exotic animals, but nobody could really imagine the immense variety of lifeforms that existed on earth.

            In the 18th-19th centuries as more and more people traveled, some of them, the Naturalists, started to notice such variety, and compiled extensive collections of accurate drawings, descriptions and preserved specimens. And they noticed that this variety was not random, but followed patterns, and therefore set their minds to make sense of these patterns, to find the underlying rule. And finally one of such Naturalists, Charles Darwin, found the rule.

            Could have people come up with good arguments for evolution centuries before Darwin? In theory yes, in practice probably not. Even if there was sufficient evidence available, it was not salient enough for people to notice that there were even patterns to be explained.

          • sconn says:

            Right. But that holds true even when you debate inside the whisper network. I find that when I debate edgy views, the people who hold them say, “Well, the only reason you could possibly say that is that you are brainwashed by the establishment!”‘ And of course, establishment people are likely to have trouble hearing the edgy views either, because their lizardbrain is saying “the only people who could possibly think that are TERRIBLE PEOPLE.”

          • If you wanted to make an argument about evolution in the Middle Ages – well, you couldn’t, because the evidence wasn’t there.

            I don’t agree. You couldn’t provide nearly as good evidence for evolution as we can now. But the essential insight isn’t the evidence, it’s the logic. Once one sees that it is clear that the evolutionary explanation is at least possible, is one way in which the world we observe could be explained.

            Which is enough to undercut one of the strongest arguments for religion.

          • Murphy says:

            @DavidFriedman

            I somewhat disagree, while you could construct experiments in the middle ages reasoning from first principles is prone to awful failure.

            Especially if you’ve got a few false first principles mixed in like a deity who wouldn’t let X happen.

            (Experiments, if constructed poorly and constructed with bad base assumptions can also be misleading )

            “If a soiled shirt is placed in the opening of a vessel containing grains of wheat,” he wrote, “the reaction of the leaven in the shirt with fumes from the wheat will, after approximately 21 days, transform the wheat into mice.”

            So also some animals are produced from animals of
            a similar form, the origin of others is spontaneous, and
            not from similar forms ; from these and from plants are
            divided those which spring from putrid matter, this is the
            case with many insects ; others originate in the animals
            themselves, and from the excrementitious matter in their
            parts ; those which originate from similar animals, and have
            both the sexes are produced from coition

            If you’ve not had genetics and Mendelian inheritance explained to you, if working from first principles there’s many, many, many sort of reasonable positions you can come to, at least positions more reasonable than dirty rags and grain spontaneously generating mice.

            Let imagine a hypothetical world that looks very similar to ours but with a few of the sliders adjusted.

            Imagine a world where horizontal gene transfer was much much more common by many orders of magnitude. It’s not against the rules in our world but without experimental evidence you couldn’t reason from first principles that it wasn’t a thing.

            Without experimental evidence you couldn’t reason from first principles that putting a really wonderful healthy Ewe in with your so-so flock wouldn’t cause all the lambs born to all the other Ewes be healthier as well.

            because in a world with lots of extra horizontal gene transfer that could be the case.

            In a world with lots of beneficial symbiotes and probiotics that could also be true.

            In a world with a [Deity] who happens to bless certain people and animals and favor all those near them that would also be true.

            In a world where that Ewe is unusually bright and tends to nudge the flock towards better pasture and shelter that would also be true.

            Most problems have a very large plausible hypothesis space.

            Reasoning from first principles without enough experimental evidence and simply picking one of those plausible hypothesis is a way to get the wrong answers most of the time.

            Hindsight bias devalues science. When you already have the answers in front of you it seems obvious that you could have reasoned them from first principles without all those expensive experiments.

          • vV_Vv says:

            Yes, at some point this becomes like speculating if they could have discovered quantum mechanics in the Middle Ages.

            In principle yes, it was physically possible to construct all the relevant equipment and carry out the experiments, in practice it would not have been the Middle Age anymore if they were able to do this sort of thing.

          • Murphy says:

            You could establish some of the basics of inheritance through experimentation.

            Indeed Mendel’s experiments are somewhat of a demonstration of that.

            The more complex details however would be far out of your reach.

            What you are extremely unlikely to do without screwing up is just guessing from first principles from looking around yourself.

          • B_Rat says:

            You couldn’t provide nearly as good evidence for evolution as we can now. But the essential insight isn’t the evidence, it’s the logic. Once one sees that it is clear that the evolutionary explanation is at least possible, is one way in which the world we observe could be explained.

            Which is enough to undercut one of the strongest arguments for religion.

            The problem with this sort of reasoning is that one could look at what actually happened in history and observe that even a certain nobody named St. Thomas Aquinas wrote:

            Species, also, that are new, if any such appear, existed beforehand in various active powers; so that animals, and perhaps even new species of animals, are produced by putrefaction by the power which the stars and elements received at the beginning.
            — Summa theologica, I.73.1 reply3

            This example is specifically singled out as a possible objection to the idea that “the completion of the Divine works ought to be ascribed to the seventh day”. The Aquinate answers that even if new animals may develop, the powers (natural laws) by which they are created were there by the seventh day.

            Thus it was possible to study the rules according to which new species appeared, yet no Darwin in the Middle Ages.

          • Ilya Shpitser says:

            One thing I seriously worry about is generally in what you call “whisper network debates” the orthodoxy has a huge intellectual firepower advantage. This is perhaps not surprising, but what it means is correcting wrong orthodoxy is difficult because folks arguing against it are outnumbered and outgunned, even if we posit a “whisper network” setting where people can mostly speak freely.

            I have generally not been hugely impressed by heterodox thinkers on essentially any controversial issue. But maybe I can’t see enough sigmas out! Or maybe I need to learn their language better? I don’t know.

            The other thing is, the “heterodoxy ecosystem” is polluted by a lot of bad stuff, like conspiracy theorists (and other variety of poor epistemic hygiene), genuine assholes, foreign state actors, and so on.

        • 27chaos says:

          I can think of several things Scott might be talking about, not just one.

          • Sgeo says:

            I can think of one thing that it’s probably not based on prior posts, but which this might look like a dog whistle for without that context.

          • Scott Alexander says:

            This post isn’t about any one thing, though I have a couple of examples in mind.

          • Luke the CIA Stooge says:

            Come on Scott, don’t play coy, we all know what this post is about and i find your pathetic attempt to hide behind intellectual discussion disgusting.

            You think Lance Armstrong did nothing wrong and you want to take the Tour de France back to the bad old days.
            You probably even keep a copy of My Comeback on your dresser, and wear your wristband to bike rallies with your pathetic racer friends.
            You disgusting, fit piece of shit.
            Do you ever even sit in front of your laptop in your bedroom like a well adjust person? Or do you spend your entire life outside getting exercise bemoaning the fact that every girl always found you hot?

            I can’t even.
            Tip your bike helmet and leave. Your not welcome on the internet! go back to polite society and wallow in your contentment with cyclist friends.

            But you’ll always know deep down your a bad person!
            Cyclist!!! Oppressor of skateboards! bike Supremacist!

            How can you people read this guy he’s literally Lance Armstrong!!!

        • vV_Vv says:

          Maybe the thunder does come first for some of these unspeakable issues.

          If so, then why is it heresy to say otherwise?

          Nobody is burned at stake, sent to the gulag or fired from a near-anagram of it for saying that the earth is flat, are they?

          This does not mean that all heretic beliefs are correct, of course. Occult magic is really nonsense and Jerusalem does really exist, after all. But if the powers that be have to resort to censorship to defend their orthodoxy, then this is evidence that they probably don’t really have a good argument for it.

          • sconn says:

            No, I don’t see that at all. There are loads of reasons why orthodoxy might engage in censorship besides that. For instance, the proof for their view might be extremely complex and difficult, while the price for being wrong might be very high. For instance, a doctor might lose his license for using some fringey treatment. The establishment does not want to argue him out of doing it, because he might not listen and in the meantime, he might be giving people bleach enemas and baking soda IV’s … people who are in no position to evaluate the effectiveness of this treatment.

            The Catholic Church censored heresy, not because they didn’t think you could disprove it, but because they admitted that the proofs were difficult, and in the meantime ignorant people might come to believe it … and the cost was an eternity in hellfire. They sincerely believed the risk of contaminating ignorant people’s beliefs was *that high,* and therefore they couldn’t rely on argument, which often fails even when proof is strong. (E.g. flat earthers still exist.)

            I’m sure you can easily come up with parallels about ignorant people being convinced by wrong arguments with dangerous results even today, and about whatever edgy views you’re thinking about right now.

          • t mes says:

            A common reason is the position is aggressively antisocial. Generally speaking, nobody wants to hear a defense of slavery or pedophilia and it’s not because they are afraid of the deep truths that are to be uncovered.

          • vV_Vv says:

            The Catholic Church censored heresy, not because they didn’t think you could disprove it, but because they admitted that the proofs were difficult, and in the meantime ignorant people might come to believe it … and the cost was an eternity in hellfire.

            I’m sure this was their rationalization, but in reality we know that the theological positions of the Pope and all his Cardinals were no better than those of any random raving mystic. And they must have known, at some level.

            Today we can observe many objectively false beliefs which can cause serious harm if acted on. Most of these beliefs are not illegal or even particularly socially unacceptable. Think of anti-vax, or denial of the negative effects of tobacco, or alcohol, or drugs, and so on.

            On the other hand, as far as I can tell, many factual beliefs that can get you “burned at stake” if publicly stated, are, in their steelmanned forms, at worst epistemically controversial and at best objectively true.

            Or at least, if slam dunk arguments that debunk them exist, I’ve never heard of them. And the defense that these argument exists but they are too complicated and difficult to state sounds like an excuse if these arguments are in fact never stated. If the censors can’t explain why they hold certain beliefs, how can they be sure that they are not wrong? How can they be so sure that they are willing to go such great lengths to enforce the orthodoxy and suppress dissent?

            My conclusion is that the censors don’t care about the truth, they care about power. As far as they know the orthodoxy could be false, but they know that they stand to lose if this becomes common knowledge, therefore since they can’t argue for the orthodoxy using the tools of rational discourse, they enforce it using coercion.

          • The original Mr. X says:

            I’m sure this was their rationalization, but in reality we know that the theological positions of the Pope and all his Cardinals were no better than those of any random raving mystic. And they must have known, at some level.

            Arguments that people who disagree with us really, deep down, know that they’re wrong and we’re right, are rarely very good.

          • vV_Vv says:

            Arguments that people who disagree with us really, deep down, know that they’re wrong and we’re right, are rarely very good.

            My argument is not that they know that their beliefs are objectively false, my claim is that they know that they can’t make a good case for their beliefs, thus they resort to censorship.

          • Nick says:

            vV_Vv,

            I’m sure this was their rationalization, but in reality we know that the theological positions of the Pope and all his Cardinals were no better than those of any random raving mystic.

            Prove it.

          • The original Mr. X says:

            My argument is not that they know that their beliefs are objectively false, my claim is that they know that they can’t make a good case for their beliefs, thus they resort to censorship.

            Claims to the effect of “People who disagree with me know, deep down, that they don’t have any good arguments” aren’t generally very convincing, either.

          • stucchio says:

            >Claims to the effect of “People who disagree with me know, deep down, that they don’t have any good arguments” aren’t generally very convincing, either.

            I’ll disagree with this.

            Consider alternate topics where factual support is a lot stronger – for example, global warming or evolution – they (or at least a very similar group) very vociferously argue those facts. The same is true on a variety of fairly questionable economic topics, where they at least perceive the facts to be on their side.

            Strangely, that willingness to engage with facts vanishes on certain other topics.

            “When the law’s on your side, pound the law. When the facts are on your side, pound the facts. Otherwise, pound the table.”

          • Matthias says:

            Strangely, that willingness to engage with facts vanishes on certain other topics.

            You’re going to have to get more explicit, because on all the issues I can think of you trying to darkly hint at (race, gender, sexuality, religion, whatever) both “sides” are very eager to cite empirical claims that support their position. You can argue that these are fake alternate universe facts, like with creation science or the like, but you seem to be explicitly saying they’re very different from that, and my own experience (as someone who has spent far to much time online yelling at people) is utterly different from that. Maybe it’s a filter bubble thing.

          • stucchio says:

            Mathias, my claim is that on topics like race and gender, they do not in fact appeal to any facts. For example, no one that I read correctly stated a claim of James Damore and then showed why it was incorrect.

            Similarly for Charles Murray, or Bret Weinstein, or any of the others.

            Or similarly, see the comments here: https://www.chrisstucchio.com/blog/2016/are_gays_or_guns_more_dangerous.html

            Lots of criticism, but no empirics.

            If you think I’m incorrect, can you actually link me to an article which a) correctly states a claim James Damore made and b) attempts to use data/theoretical argument/etc to refute *that argument* rather than attacking Damore or attempting to debunk a straw man?

          • lvlln says:

            stucchio’s comment reminds me of something PZ Myers said in response to a comment on his blog on a post related to a culture war topic: “Your math is fine. It’s your humanity that is broken.

            To the credit of his commentariat, he received immediate pushback.

          • vV_Vv says:

            @qwints

            Most of these “scientific arguments” against Damore cited in the link you provided are vitriolic personal attacks.

            But to the extent that they make actual scientific arguments, all they can argue is that Damore’s case may not have been as strong as it is presented. In some cases there is conflicting evidence. This is hardly a refutation.

            So the intellectually honest response would be to recognize that Damore rised some potentially valid points about our collective epistemic state that have important policy implications, and while not all of his arguments may be 100% correct, it is important to discuss these issues and evaluate the evidence, and if necessary do more research.

            Instead the response is that Damore is branded as heretic and all the arguments that he made are denounced as blasphemy not to be ever uttered again.

          • Aapje says:

            @qwints

            That’s a gish gallop, but I’ll address one of the criticisms from the site you linked:

            American businesses also have to face the fact that the demographic differences that make diversity useful will not lead to equality of outcome in every hire or promotion. Equality or diversity: choose one. In my opinion, given that sex differences are so well-established, and the sexes have such intricately complementary quirks, it may often be sensible, in purely practical business terms, to aim for more equal sex ratios in many corporate teams, projects, and divisions.

            First of all, this is primarily a moral argument, not a scientific argument. Do you favor equality of opportunity or equality of outcome? Having a different preference than Damore is not actually an argument that he made a mistake in his reasoning.

            Secondly, the critique has the assumption that jobs will be done better if you have workers with disparate abilities. This is highly doubtful. You probably want all surgeons to have fine motor control. You probably want all front-line soldiers to not have emotional breakdowns on the battlefield. You probably want all programmers to be highly systematizing. If one wants to argue that the upside of more diversity are greater than the downside, the first step is to acknowledge the possibility of both upsides & downsides. The second step is to compare them somehow. The critique does neither and instead assumes that no downsides exist.

            The critique doesn’t address the implications of Damore’s claim that the main cause of the disparity is that women are interested in programming far less often, rather than companies refusing to hire them. This means that those companies don’t actually have the ability to hire equally good female programmers. The critique seems to assume that companies have the ability to solve the issue, but it offers no good arguments why this assumption is true.

            Finally, Damore actually did argue that we should look into possibly changing the job of programming to make it more attractive to women and/or playing to their strengths more. He merely doesn’t believe that discrimination in hiring is just. Again, Damore’s dislike of gender discrimination is a moral preference, not a scientific issue.

          • stucchio says:

            Qwintz, I have read some of those critiques, though not this roundup. The roundup is actually quite odd – I’ve read some of the people included here and it’s quite strange how nymag fails to quote the parts where they completely support Damore’s factual claims.

            For example, here’s Geoffrey Miller. Nymag didn’t see fit to quote this part of Miller’s response:

            I think that almost all of the Google memo’s empirical claims are scientifically accurate. Moreover, they are stated quite carefully and dispassionately. Its key claims about sex differences are especially well-supported by large volumes of research across species, cultures, and history…his memo would get at least an A- in any masters’ level psychology course. It is consistent with the scientific state of the art on sex differences.

            It merely quoted something Miller said tangentially which doesn’t even really disagree with Damore.

            Similarly, here’s what Schmidt says:

            …sex differences exist in negative emotionality is not an “incorrect assumption about gender.” It is an empirically well-supported claim (at least, based on the best psychological science we have so far)….Culturally universal sex differences in personal values and certain cognitive abilities are a bit larger in size (see here), and sex differences in occupational interests are quite large2. …should we be able to openly discuss and be informed by some of the real psychological sex differences that account for variation in men’s and women’s workplace performance, and might lead to less than 50% of technology employees being women? In the right context, I vote yes to that, too. Apparently at Google, internal discussion boards intended for open conversations about diversity and science-based thinking are not the right context for discussing evidence about psychological sex differences.

            Many of the others (e.g. Cynthia Lee, Rosalynd Barnett) do not dispute his science, but merely attack him and discuss their own negative emotional reactions. For example, Barnett critiques Damore’s

            …claim that women’s biology makes them less able than men to work in technology jobs…We can say flatly that there is no evidence that women’s biology makes them incapable of performing at the highest levels in any STEM fields.”.

            They are very explicitly NOT arguing against anything he said.

            The only one I haven’t had a chance to read carefully is the Quora post. Maybe later this week.

          • qwints says:

            @stuchhio. Fair. Let me not one point from the quora article that I think meets your test as it “a) correctly states a claim James Damore made and b) attempts to use data/theoretical argument/etc to refute *that argument*”

            Damore (p.5):

            Note that contrary to what a social constructionist would argue, research suggests that “greater nation-level gender equality leads to psychological dissimilarity in men’s and women’s personality traits.” Because as “society becomes more prosperous and more egalitarian, innate dispositional differences between men and women have more space to develop and the gap that exists between men and women in their personality traits becomes wider.”/blockquote>

            Sadedin (4th heading)

            At first glance, this seems compelling. Let’s look more closely at this paper. Specifically:

            “these changes appear to result from men’s cross-cultural personality variation. In more traditional and less developed cultures a man is, indeed, more like a woman”

            Hmm. That sounds a little different, now. In fact, Table 2 shows that, after controlling for human development index, the only gender equality-related factor that predicted gender differences was the ratio of female smokers. In other words, gender equality in general doesn’t change women’s personalities, or the difference between men and women. Rather, human development index changes men’s personalities much more than women’s.

            That doesn’t support the claim that gender-liberal societies allow men and women to express innate differences more freely. If that interpretation were correct, women and men should diverge in gender-liberal societies independent of egalitarianism. Instead, men change personality in more egalitarian societies regardless of gender issues; women don’t.

          • stucchio says:

            Qwintz, the point you cite is her NOT disagreeing with Damore.

            Damore said that higher developed societies result in a larger D = M – W. She’s simply saying “totally true, that’s because M changes, he’s so wrong.”

            I think she’s also glossing over other arguments Damore made regarding the correlation between gender equality and % women in tech. Specifically that Iran/Saudi Arabia/etc are great at getting women into tech, France/Sweden/etc not so much. That’s very explicitly a case where W changes rather than M.

            But I’m not particularly sure that was in the memo – it’s possible I’m remembering Damore making this argument elsewhere (e.g. in one of his interviews).

          • lvlln says:

            @Aapje

            The critique doesn’t address the implications of Damore’s claim that the main cause of the disparity is that women are interested in programming far less often, rather than companies refusing to hire them.

            Did Damore actually claim this? It’s been quite a few weeks now since I read his memo in full, but I recall concluding that his claim was that the disparity was not evidence of companies refusing to hire women (or some other form of sexism), because given what we know based on the latest available social science research, men’s and women’s average preferences would tend to disproportionately lead more men toward positions that Google is looking to fill, which would cause such a disparity even sans sexism. I don’t recall him actually claiming that this really was the main cause or a cause at all – merely that it was a sufficiently plausible that it was a cause, such that we can’t just dismiss it as not a cause in favor of “sexism” as the likely or only cause of the disparity.

      • MugaSofer says:

        But you’re not attacking “white lies”; you’re attacking the idea that any claim, no matter how abhorrent and obviously false, should be unacceptable to publicly proclaim.

        “Society should not have false beliefs” isn’t exactly controversial, since (as we all know) our society loves truth and welcomes honest criticism. “Society should not censure people with false beliefs, no matter how harmful they would be if implemented, no matter if people are actually implementing them” is super controversial even at the meta level.

        • 天可汗 says:

          “Society should not have false beliefs” isn’t exactly controversial

          Yes it is, for several reasons, and from several directions.

          On the one hand, “Christianity provides enough benefits to its believers to outweigh the costs they might incur from it being false, if it is false” isn’t controversial, but what that translates to is “if X is false, society should still have X false belief”.

          On the other hand, comrade, what is truth anyway?

          • Randy M says:

            It’s been argued in the comments here before that certain hypothesis shouldn’t be suggested even if true because it would give people justification for philosophies of superiority and inevitably lead to oppression or killing.

          • vV_Vv says:

            And questioning the Holy Trinity, even if it does not make any sense, would give people justification for philosophies of Satanism and will inevitably lead to sacrificing children to the devil.

          • theofloinn says:

            what is “truth” anyway?

            In Old English, “truth” or “troth” meant “faithfulness.” Hence, a “true friend” or lovers who pledged their troth to each other (became “betrothed”). It is the Germanic equivalent of the Latin word fides (faith), inasmuch as one must be faithful to something just as one must be true to something.

      • adder says:

        I know I’m late to the party, but this post has been haunting me all week. It’s been haunting because, well, even if Scott does have a few different things in mind, it is indeed a dog whistle for something(s). And, if Scott is a Kolmogorov, then we can never quite know for sure what it’s a dog whistle for. I find this chilling, and makes me wish I could pick Scott’s brain in person. But of course, if he’s a Kolmogorov, he would be wary to reveal any more in person than he does now.

        Let’s say, e.g., that this post serves as a dog whistle for the one topic not to be mentioned on this site. Maybe Scott thinks it’s important to have whisper network about it because (A) he’s really concerned that ignoring this topic will result in society missing the opportunity to save many from fatal disease that could have been prevented if we better studied human populations. Or maybe he wants a whisper network because (B) he believes that acknowledging this topic and research will allow society to forcibly modify populations in ways that (I think) everyone basically finds horrendous. I like to think that Scott’s actual view is closer to the (A), and he’s said as much. But if he’s a Kolmogorov how would we ever know?. This community attracts a nontrivial number of commenters that express sentiments being closer to (B), or at least close enough to (B) that they can’t viably be spoken publicly. And, while I do think that Scott’s commitment to free speech is consistent with holding (A) while allowing (B)-thinkers to participate, how are we supposed to know that this forum isn’t for the (B)-thinkers to get their ideas out there?

        If Scott announced that he agreed with the consensus that thunder comes before lightning, but that he would never ban a lightning-first-er from his site, it might be the only site where a lightning-first view can get expressed. And all the lightning-firsters would think to themselves and say to each other “Of course Scott says he’s a thunder-firster. But he’s a smart guy. He knows lightning comes first. He just can’t say it straight out; he’s got a professional career and a hugely popular web site. Instead he tends a nice garden for us to get out and say so. He’s supporting us, but maintaining plausible deniability.

        Is that what (B)-thinkers are saying to each other? And are they right? I really have no way of knowing.

    • Sniffnoy says:

      Whether Scott happens to be right on the object level on claims not discussed here doesn’t really have to do a lot with the more general harmfulness of this sort of thing…

      • t mes says:

        I agree. The “general harmfulness of this sort of thing” was not the subject of my reply however.

        • ruelian says:

          I’m confused as to your position: you agree the behaviors under discussion are generally harmful, but in this case you still support them? I’d like to understand your reasoning on this…

          • t mes says:

            I was speaking about some of the pet “unspeakable” topics in the ssc-sphere, not about a surface level reading of the post which I agree with in the same common sense way almost everyone will agree.

          • Shion Arita says:

            Ok, since you’re the one who’s bringing it up, what are some of the pet “unspeakable” topics in the ssc-sphere. It shouldn’t be a problem for you to list them since you’re saying they’re being discussed openly here. I have some ideas about what you might have in mind, or what scott might have in mind when writing this, and I understand why Scott doesn’t want to say what those were, because the general case is more important.

            However, since you’re talking specifically about this aspect of it, it would be nice to know directly what you’re talking about.

    • Nornagest says:

      Sounds like something a heretic would say.

    • veeloxtrox says:

      As someone who grew up in the Midwest and recently moved to Silicon Valley I found Scott’s post helpful. It feels like there are some subjects that are held very dogmatically and when SV gives statements about basing decisions off data it sends me looking for the data behind the dogma. Yet, I have received backlash for questioning the dogma in what I attempted to be a good faith discussion.

      I read the whole post as Scott reflecting on this issue as a whole in a kind of meta level. Yes there are specific issues that have happened recently in SV that probably inspired this post. Yet this post is just as helpful to me in SV as it could be to someone who grew up in SV and moves to Small Town USA and is having to slowly (and awkwardly) figure out where the dogma is and what cannot be questioned.

      • Conrad Honcho says:

        Can you pop my bubble for me? What are some examples of Small Town USA dogma that can’t be questioned? The obvious response is “JESUS” but there’s atheists and Jews and a big Hare Krishna collective around my town and nobody seems to get shunned.

        • Brad says:

          How about that working for the military is a good thing? (You can even call it working, you have to call it serving.)

          • Conrad Honcho says:

            No, there’s plenty of people who will bitch about ‘nam or how stupid the Iraq war was.

            Now when someone says their kid is serving in the Marines you can’t scream “baby killer!” at them, but that’s out of politeness.

            Now I do think if you started burning the flag or something, that would get you shunned. You can defend others burning the flag, barely, but definitely no flag burning allowed.

          • Brad says:

            Of course you can’t scream baby killer. But can you calmly and politely express dismay that someone’s kid is serving in the Marines because you think it is terribly immoral to join the Marines and go kill innocent people? Or will that get you just about the same reaction as going to a cocktail party in Silicon Valley and calmly and politely explaining that you think that all other things being equal a company should hire a white person over a black person because you should have a higher prior that they are more intelligent?

          • Nornagest says:

            I am under the impression that “serving” comes from the way military enlistment is structured, i.e. as an irrevocable service contract. Historical apprenticeships worked the same way, where you sign up for a fixed term of years and from that point you’re stuck for the duration unless you run away (which is illegal) or the master craftsman decides to dismiss you for some reason, and “serving as [say] a tanner’s apprentice” makes linguistic sense to me.

            Contrast firefighting or EMT work, which carry the same public-service status but which are structured as conventional jobs. “Working for the fire department” sounds more natural than “serving in the fire department” to me, although I have occasionally heard people tell firefighters or EMTs “thank you for your service”.

          • Conrad Honcho says:

            Brad, if you were at a Silicon Valley party and someone said their son joined the marines, would you calmly and politely express this dismay to him or her? How would that be received?

            Also, you incorrectly model Small Town USA racism. This is the land of “I don’t care what color you are, black, white, purple or green, can you get job done?” What you’d get instead is the Soft Bigotry of Lowered Expectations. Something like, “This new guy they hired is great. And he’s black!”

          • Brad says:

            @Conrad Honcho
            I believe you misunderstood what I was saying. I don’t suggest that a small town red tribe member invited to a cocktail party would say such a thing. It would be one of these gray tribe edgelords. I was only using the reaction such a thing would recieve as a point of comparison.

            Speaking of which I don’t think you ever answered the question.

          • Conrad Honcho says:

            I was saying I don’t think the question is reasonable, because at the Small Town USA backyard BBQ, nobody’s going to suggest not hiring blacks over group racial differences either.

            So, I think you would get a better reaction to explaining to Small Town USA guy that joining the Marines is immoral, because the Small Town USA guy is already pretty much aware that’s how some of them city slickers think and he disagrees. Worst he’d probably just tell you to shut up.

            Speaking poorly of blacks at the SV party is going to get you chucked out and excommunicated. Doing that at the Small Town USA BBQ would probably cause some uncomfortable silence, or a response along the lines of the Soft Bigotry of Lowered Expectations. “Don’t think like that because my work just hired a black guy who’s really smart.”

            Basically I think you misunderstand the nature and overestimate the virulence of flyover country racism.

            I think the better answer for “absolutely do not do that” out in the country is flag burning. You will get your ass kicked for that.

          • Brad says:

            Speaking poorly of blacks at the SV party is going to get you chucked out and excommunicated.

            Perhaps it is you that has the wrong model.

          • Conrad Honcho says:

            Perhaps so. I didn’t realize that sort of thing was socially acceptable in SV.

  76. rechelon says:

    One thing you’re not really addressing is how to avoid those whisper networks from spiraling into their own epistemic traps. The whole “and don’t go that far, because that’s silly” stuff seems rather glossed over and a major issue. Underground whisper network are particularly prone to compounding bad claims. See for example hundreds of years of esoteric nonsense that leads to smart people getting all into the occult and terrible terrible means of finding knowledge and then sharing them with each other.

    “Hey buddy, saying IQ is real will get you shunned badly by the other students and possibly sanctioned by the teachers. Also if you think that’s bad, wait till you hear about how global warming is fake, blacks are subhuman, and we should really be able to rape 6 year olds.”

    Whisper networks seem particularly prone to turning into singularities of tribal epistemic closure because trust piggybacks and people have bad instincts on the whole when it comes to rationality and vigilance, even when they have good instincts for curiosity.

    • Paul Crowley says:

      This seems really important. It is not a coincidence that very often the people who publically say “Lightning comes before thunder!” are very often awful people who you would legitimately shudder to be associated with.

      • promotoriustitiae says:

        I suspect the idea that any network like this has been healthy is wrong, though retrospectively the correct portions just shine brighter. The point of the post seems to be that they are, nonetheless, important enough to participate in and protect. In an optimistic view, the more correct portions slowly become mainstream while the dubious parts fall away under inspection.

        To put it another way, expecting that all of your beliefs will be correct is a level of confidence which is harmful. Since the people who wield power aren’t the same people who doubt their beliefs, any network which remembers to doubt itself is pretty harmless. There’s no need to be a true believer in facts, they’ll be true regardless.

      • Desertopa says:

        I think much of the association here is driven by the “say publicly” element, because people who openly say things which make it effectively impossible for them to be respected in polite society are very often people who are not concerned with being respected in polite society, who are very often unpleasant people.

        • Evan Þ says:

          Unfortunately, people can easily correlate that in their own minds with “think privately.”

          • Yosarian2 says:

            I mean, the safest way to never say something socially unacceptable is to not believe it either. If you believe it and just censor yourself from saying it, it’s a constant struggle, and there’s always a risk you’ll slip. Safer to just convince yourself that thunder comes first and squash any doubts.

          • The Nybbler says:

            Safer to just convince yourself that thunder comes first and squash any doubts.

            It is not easy to become sane.

        • are very often people who are not concerned with being respected in polite society, who are very often unpleasant people.

          But not always.

        • albatross11 says:

          +1

          Suppose you live in a time and place where being a member of the local church is required to be a full participant in the community. And suppose you have some doubts about the rightness of the local church’s doctrines.

          If you just leave those doubts in place, even sometimes acknowledge them to close friends without making a big deal of them, etc., you can remain a member of the community. It won’t take a lot of mental effort to keep expressing acceptable opinions in public, because you’ll mostly have the ones you were raised with, just with a few inner doubts.

          On the other hand, suppose you spend a few years thinking deeply about your doubts, and come to the conclusion that there are neither gods nor devils, that all churches are scams including the local one, and that the whole subject is a bunch of nonsense. At this point, you will have a much harder time expressing acceptable opinions and fitting in. Instead of sharing most of the acceptable worldview but having a few qualms about what exactly happens to the bread and wine in the Eucharist, you’re constantly internally rolling your eyes when someone promises to pray for you, or when someone describes a tragic death as God’s will. Your immediate reaction when you hear about a plan to build a new church or to offer more Sunday school classes for your kids is not the one your community would expect.

          There’s a lot more work and a lot less safety in being the second kind of dissenter.

    • Leon says:

      This is similar to the Roman Catholic justification for going after heretics: adopt slightly unbalanced doctrine X, and soon you’ll be believing destructive and obviously false Y.

    • Doesntliketocomment says:

      I think you’re correct, and not just because of tribality, but because of selection against those holding opposing viewpoints simply because of safety concerns. The last person you would think of inviting to your whisper salon would be someone strongly holding the orthodox view, as they would be seen as the most likely to rat you out.

    • sconn says:

      Yes, I thorougly agree. And it doesn’t even have to be all that secret. I mean, I got into alternative medicine because I had noticed that ob/gyns do not always use evidence-based practices. And because there is no way to ensure you get a doctor who does use evidence-based practices (because they all claim to and then don’t), the only solution is to stop seeing doctors and googling for your information, and the next thing you know you think vaccines are poison and apricot pits cure cancer.

      I got out of that worldview (though I still agree establishment medicine has some really serious flaws) but I find it impossible to bring anyone with me. Trust piggybacks, like you say, and any study I can bring up which disproves the apricot-pit thing was always authored by a doctor or scientist who has some ties to Establishment Medicine, such as going to med school.

      Basically as long as Establishment View can’t debate on an equal floor with Edgy View, then people are going to have a very hard time evaluating their different claims for truth or falsehood, and they’ll just grab Establishment Package or Edgy Package wholesale, because you can’t unpack them and compare contents very easily.

      • 天可汗 says:

        Yes, institutions that want to be trustworthy have to work to retain that trust. A lot of institutions seem to have forgotten this.

        The apricot-pit thing can be interpreted game-theoretically. The institutions insist that thunder comes before lightning and that lightning is just the air reacting to the presence of the Thunder Devil after he makes thunder, and they’ve raised taxes on everyone over six feet tall under the assumption that they’re being stretched out by the Thunder Devil, and so on — so they’re catastrophically untrustworthy, they’re corrupt, and they aren’t acting in the best interest of the people. But they really don’t want people to believe that apricot pits cure cancer! So we’ll go all in on that and try to draw in anyone who pays attention to thunderstorms and anyone who’s over six feet tall, and hopefully they’ll realize why this is happening and try to prevent it by moderating their position.

        In most cases, it’s an unconscious process, but the apricot-pit people would probably moderate if the institutions dropped the Thunder Devil stuff. In some cases, however, it’s an entirely conscious strategy. But these days I don’t think it would work.

    • kokotajlod@gmail.com says:

      Well said Rechelon. I think that the Kolmogorov option is better than the whisper-network option unless the whisper-network option has extremely strong norms against overcorrecting.

  77. qwints says:

    I find the analogy between “being criticized for saying something” and the purges in Stalinist Russia to be ridiculous on its face. Log off now and then.

    • suntzuanime says:

      They weren’t just criticizing atheists, they were burning them at the stake.

      • drethelin says:

        And they aren’t just criticizing people now, they’re firing and blacklisting them.

        • qwints says:

          Sorry that was I unclear. I was referring to this paragraph :

          If you think it’s impossible to be that oblivious, you’re wrong. Every couple of weeks, I have friends ask me “Hey, do you know if I could get in trouble for saying [THING THAT THEY WILL DEFINITELY GET IN TROUBLE FOR SAYING]?” When I stare at them open-mouthed, they follow with “Well, what if I start by specifying that I’m not a bad person and I just honestly think it might be true?” I am half-tempted to hire babysitters for these people to make sure they’re not sending disapproving letters to Stalin in their spare time.

          You might get fired, not hired or yelled at, sure. You’re not going to be shot or put in a gulag. We are not talking about atheist bloggers in Bangladesh here.

          • vV_Vv says:

            You might get fired, not hired or yelled at, sure. You’re not going to be shot or put in a gulag.

            In various Western countries you can actually go to prison for saying the wrong thing. The US is the exception, for now, but even there you can have your head smashed with a bike lock by a masked college professor for taking part in a rally.

            Should we wait until the Red Guards are rounding up people and putting them against the wall before saying that something is going wrong?

          • qwints says:

            I like vvvvvv better :).

            How many people are in prison in “various Western countries” for saying the wrong thing? I mean, I’m with you in liking the US version of free speech and opposing hate speech and blasphemy laws, but I can’t find any examples of people actually in prison by googling “people in prison for hate speech laws” and “people convicted of hate speech.”

            And by all means criticize Clanton (who should be convicted) or anti-free speech groups more broadly. Feel free to say something is going wrong, but when when people like the Scotts compare it to Stalin’s USSR when doing so, I am much less likely to take their claims about it seriously.

          • Randy M says:

            I’ve seen face-palm retweets of British Hate Crime police, but I’d like some actual numbers on this too.

          • Protagoras says:

            @qwints, Irving spent 13 months in prison in Austria for holocaust denial. That is, admittedly, the only recent case of that sort of thing I know of.

          • qwints says:

            @Protagoras, thanks. I forgot about the bans on Nazi imagery and holocaust denial, and searching those leads to a few more cases.

          • Luke Edwards says:

            @qwints

            3,400 people arrested for offensive online comments last year in Britain alone. I’m not sure what sentences they received, though I know that at least one person was sentenced to 4 years in prison:

            https://twitter.com/TheSafestSpace/status/919199085411811328

          • qwints says:

            Luke Edwards, thanks. Your link led me to this report which was quite informative about the situation in the UK. There’s a lot of messiness here as threats, stalking (some of which would be illegal even in the US), ‘harassment’ and ‘offensive’ statements all seem to be covered in the same category. For example, I wouldn’t characterize a case where a person sent a picture of a knife and the statment that she would ‘get it like Jo Cox” as going to prison for saying the wrong thing. I would characterize the people punished for burning poppies that way, but those cases resulted in a 50 pound fine in one instance and having to talk to veterans in another. I’m against the arrests and punishment, but that’s not “going to prison”.

            Ultimately, as the report above shows, it looks like there is no comprehensive data on how many people are imprisoned for saying the wrong thing.

          • Paul Brinkley says:

            This reminds me of the California Climate Science Truth and Accountability Act proposed last year. It was reported in some places as “climate change denial is now a crime”. The text says it permitted people to sue organizations for spreading falsehoods about climate science. The obvious retort to this is to ask what is being considered to be a falsehood (the text is obviously pro-warmist), or to ask what would happen if, say, Heartland tried in good faith to sue Al Gore.

            The bill ultimately died on the inactive file (see the history tab).

          • B_Rat says:

            You might get fired, not hired or yelled at, sure. You’re not going to be shot or put in a gulag. We are not talking about atheist bloggers in Bangladesh here.

            But that’s because of which punishments were considered appropriate at those times, not because of what they chose to punish. The same could be said of insulting a man of power.

        • albatross11 says:

          Getting fired for your unacceptable opinions (or suspicion of having them) isn’t *as bad* as getting sent off to the gulag or burned at the stake, but it’s still pretty bad. And it still creates bad incentives.

          • rlms says:

            If getting fired for your opinions is bad, not getting hired because of your race must be unimaginably terrible (since the former is much easier to disguise). But nevertheless I think it would be silly to compare the pre-Civil Rights Act United States to Soviet Russia.

          • suntzuanime says:

            I think the correct analogy there would be Nazi Germany, right? As you mention, the dynamics of race are substantially different from the dynamics of ideology.

          • John Schilling says:

            If getting fired for your opinions is bad, not getting hired because of your race must be unimaginably terrible

            Not getting hired costs you the time invested in a cover letter and an interview, and a modest bit of aggravation tempered by the fact that you probably didn’t expect to get any one particular job and don’t know why you didn’t get that specific one. Getting fired adds in the opportunity cost of the job you quit/turned down in favor of the one you took, possibly the cost of moving to a new home, and likely some lost investment in non-transferable skills, knowledge, and social capital. Plus a much greater hit to your sense of self-worth, because the job you got fired from was almost by definition the best job you thought you could get, and you had thought you had earned recognition as being worthy of that job.

            Unless by “not being hired” you actually mean “blacklisted so cannot be hired anywhere“, getting fired is much, much worse. And blacklisting doesn’t fit your context or analogy.

          • The original Mr. X says:

            Even without a blacklist, a lot of jobs require reference from your previous employer(s). If you come back with “I can’t get a reference because I was fired,” that is quite probably lead to your application being rejected on the spot.

          • Evan Þ says:

            @The original Mr. X, that can be the case, but there’re sometimes ways around that. I know someone who was about to be fired on (unfounded, IMO) charges of harassment, but his employer let him resign instead, and he was able to find older managers who could give him good references.

    • Sniffnoy says:

      Attempting to permanently discredit someone as one of the bad people you should never listen to, is not the same as honestly arguing with them.

      Anyway, the important question isn’t, how badly are people hurt, but rather, to what extent is honest truth-seeking discouraged, and the resulting picture of the truth being distorted? Saying “oh but at least we didn’t kill anyone in the process” just isn’t much of a defense against the claim that you’ve given a lot of people a seriously bad epistemology.

    • hnau says:

      The original Scott Aaronson post was I-swear-totally-not-about the Google memo and its fallout. Getting fired for expressing opinions is not quite like being purged, but it’s more like that than it’s like being criticized.

    • Clarkey says:

      Another thing – I don’t think it took particularly much brains to figure out which matters to STFU about in Catholic Europe or Stalinist Russia. So the effect size of smartness on uncovering truths deemed dangerous seems small

      • Tracy W says:

        There was a sequence, at least in the Spanish Inquisition and Stalinist Russia.
        1. Declare a view (Judaism, pro-Tsarism) off limits and banish from Spain/to Siberia or kill anyone who expresses it
        2. Realise that people occasionally lie about their beliefs, particularly if threatened by banishment or death. Require public affirmations of Christianity or loyalty to the new regime on the threat of banishment or death.
        3. Realise that people can really lie about their beliefs. Start looking for subtle signs of a lack of belief in Christianity or Stalinism. On the threat, naturally, of banishment or death.
        4. Realise that people might be communicating their beliefs in code. Accuse people based on your interpretation of their code, on the threat of banishment or death.
        5. Say something that sounds vaguely like a code. Realise, possibly, that you probably should have stopped at step 1 as you are sent to the torturers or loaded on that truck heading East.

        • Jaskologist says:

          Is this actually true about the Inquisition, though? I’m not an expert myself, but I’ve seen the Deatheaters claim plenty of times that the Inquisition didn’t torture you until you confessed, but rather until youstopped confessing, which is very different. In fact, that would be very in line with allowing the sort of whisper networks Scott wants.

          • Evan Þ says:

            Based off half-remembered generalizations, I wouldn’t be surprised if that was a difference between the Papal and Spanish Inquisitions.

          • theofloinn says:

            cf.
            Edward Peters, Inquisition.
            Henry Kamen, The Spanish Inquisition.

            They did not routinely use torture. In fact, they used it less often than the secular courts, and only if they had solid evidence beforehand.

          • Tracy W says:

            @theofloinn:
            I stand corrected. Thanks!

        • dansimonicouldbewrong says:

          There seems to be a lot of misunderstanding here about how totalitarianism works. Totalitarian rulers really don’t care what people “really believe”–only that they demonstrate sufficient fear of saying what they really believe to convince the authorities that they’re no threat. In fact, one standard totalitarian technique is to force people to endorse absolutely ludicrous beliefs, thereby convincing everyone, not that the ludicrous belief is true, or even that everyone believes it, but rather that the authorities are so fearsome and all-powerful that everyone will say obviously ludicrous things to avoid challenging them.

          The result is that while there’s a lot of minor disinformation, nobody is so foolish as to trust the government just because nobody contradicts it. On the contrary, the public tends to propagate all sorts of anti-government beliefs–mostly wild rumors and gossip–by communicating privately with family members and close, trusted friends. Everyone learns from a very early age how to toe the party line and spout the requisite approved drivel in public, and how to recognize with high precision exactly what level of frankness is safe in any particular private circumstance. (Since this is just about the most important survival skill of all in such a society, there’s naturally a great deal of energy devoted to teaching it.)

          The vast majority of the population has no difficulty absorbing and mastering these skills, and the few who don’t are similar to those who can’t master basic social skills in open societies: compulsive rebels, hopeless misfits (perhaps of the SSC variety), and a few noble martyr types.

          As for scientists, I’ve got bad news–they’re pretty much like everybody else. The vast majority of them are perfectly capable of mastering the basic life skills required of a totalitarian state, and doing their science in that context. All this stuff about science requiring unstinting iconoclastic curiosity and so on is charmingly idealistic, but that’s not even how science works in the free world. Science is a method, and clever, creative, technically competent people who are also good at rigorously following a method can be very good at science–as well as at using their basic social/survival skills to direct their scientific research in harmless ways, whether the danger at hand is arrest by the secret police or just rejection by peer reviewers.

          • Tracy W says:

            But this describes a while after the Revolution. Not the true believers who drive the initial revolution (and generally didn’t survive Stalinism. Or the French Revolution’s similar terror.)

          • dansimonicouldbewrong says:

            Are you saying that the Bolshevik “true believers” were actually even worse than Stalin, since they’d have preferred to slaughter all non-believers (probably 80 percent of the country) rather than just murder a few tens of millions to terrorize the rest into submission? That’s certainly an interesting take on the purge trials…

          • Tracy W says:

            I don’t follow? I don’t recall ever reading anything of the kind.

            I got the impression that the true believers believed that once ordinary people had seen the wonders and virtues of the new system (be that French or Russian), they’d fall in line. Things only went downhill when such wonders didn’t quickly materialise and people started looking for whomever it was who was sabotaging the glorious new system. And then it worked out that those in government – the true believers – were particularly subject to the terror.

            Have I got my history wrong? Do you have any sources supporting this view?

          • multiheaded says:

            dansimonicouldbewrong is referring to the fact that many/most high-profile victims of Stalinist trials and purges were committed Old Bolsheviks who formed the party’s left/center-left in the 1920s.

          • Tracy W says:

            @mulitheaded: no, I’m the one who said that the true believers “generally didn’t survive Stalinism.”

            I don’t follow what
            dansimonicouldbewrong is saying but it seems to be that he thinks that I should believe that the true believers were setting out to kill some 80% of their fellow citizens (!). But he provides no evidence to support this assertion and I am very sceptical that large numbers of people were *that* cartoon evil.

          • multiheaded says:

            yeah I mixed up your usernames sorry

        • MostlyCredibleHulk says:

          5. usually happens regardless of what you say. Simply because if somebody gets the power to interpret codes, it’s the ultimate power of life and death. There can’t be too many people who wield that power. In fact, there can be only one – Stalin. All the rest have expiration date. That’s why Ezhov followed Yagoda, and Beria followed Ezhov, and if Stalin didn’t die, somebody would have followed Beria, who would have been found to be a spy and a traitor (which he promptly was anyway when Stalin died).

      • baconbacon says:

        Another thing – I don’t think it took particularly much brains to figure out which matters to STFU about in Catholic Europe or Stalinist Russia. So the effect size of smartness on uncovering truths deemed dangerous seems small

        When did you find out? In Stalinst Russia you found out after a purge…. Oh, everyone who was saying X for the last 3 years suddenly disappeared? Man am I lucky my girlfriend/pastor/professor didn’t turn me in…. yet.

      • Conrad Honcho says:

        Well, but the people in Scott’s examples weren’t persecuted for “uncovering truths.” Unless consorting with the Devil is “true,” or Bruno’s books about magic and his belief the planets had souls are “true.”

        I think there’s a difference between saying “I don’t think the orthodoxy is true” and “here’s something completely out of left field I think is true.”

        Naturally I don’t think anyone should be persecuted for their beliefs, and I’m proud of the Church for learning from it’s mistakes, and wish the secular world could do a little better at this.

    • Tracy W says:

      I find the analogy between “being criticized for saying something” and the purges in Stalinist Russia to be ridiculous on its face.

      It’s the nature of the criticism. Not the mere fact of it.

      • qwints says:

        Replying to you and Sniffnoy.

        But it’s not the same type of criticism. Stalin killed or exiled everyone who could possibly be a threat to Stalin without caring about type I error. Many other members of the revolution,famously including Trotsky, are murdered, executed or exiled. It essentially had nothing to do with orthodoxy or correct beliefs – the correct beliefs changed continuously and shamelessly as Orwell pointed out.

        I’m inferring that the Scotts are referring to beliefs about differences between individuals and groups which have attracted vociferous criticism, economic consequences and even physical assault in a few cases (and not beliefs that have a reasonable likelihood of you actually being murdered or imprisoned if you express them). It’s equally true for a number of other beliefs that attract extreme opposition, such as certain religious or political beliefs that are unacceptable in certain contexts.

        But there are plenty of contexts where one can express those kinds of views without any fear whatsoever. Moreover, there are contexts where voicing opposition to those views results in the same sort of consequences. I’m not talking about situations about Kerensky writing from exile, I mean that there are people with a large public profile enjoying economic and social success while they loudly proclaim these allegedly “unacceptable beliefs.”

        • 天可汗 says:

          I really don’t think that losing the ability to pay your bills and most of your social connections is that much of a walk in the park.

          • qwints says:

            I really do think there’s a lot of space between “a walk in the park” and [ shot by the NKVD].

          • 天可汗 says:

            Maybe it’s time to rehabilitate Mussolini then, seeing as how all he did was force-feed dissidents enough castor oil to give them a bad case of the shits.

          • rlms says:

            @天可汗
            “all he did was force-feed dissidents enough castor oil to give them a bad case of the shits.”
            Which falls into the category of “government infringes your rights” (same as gulags), not “other people exercise their god-given rights to freedom of association”. Also, see here.

          • 天可汗 says:

            Would you rather have a bad case of the shits or be unable to hold a job that pays more than twice minimum wage ever again?

          • rlms says:

            I don’t see how your question is relevant. Who’s in the latter situation? (SHA-256 of my response to your possible answer: 3028a166d0abc951d153b10dc1041dd91f7b478d9631c68ecff9816a4b368dfe). But even if e.g. Moldbug was starving on the streets, that question still wouldn’t cover the whole issue. We’re not considering whether mild torture is nicer than blacklisting or not, but rather what the costs/benefits of preventing each of them are.

          • Questioner says:

            rlms says:

            Which falls into the category of “government infringes your rights” (same as gulags), not “other people exercise their god-given rights to freedom of association”.

            So you support Masterpiece Cakeshop in their fight against the Colorado Civil Rights Commission?

            Or is the government destroying your business ok, when they’re doing it to pursue your preferred agenda?

          • rlms says:

            @Questioner
            “So you support Masterpiece Cakeshop in their fight against the Colorado Civil Rights Commission?”
            Yes. Consider not assuming my agenda in future.

          • Nornagest says:

            Is it me, or have accusations of hypocrisy gotten a lot more common around here in the last few weeks?

          • Questioner says:

            @rlms Glad to hear it

          • Brad says:

            Did we ever find out which of the martyrs, if any, is now destitute?

          • The Nybbler says:

            Did we ever find out which of the martyrs, if any, is now destitute?

            You wouldn’t hear of the ones who would end up destitute. They were just quietly fired for conduct reasons and are now finding it’s hard to get a job under such circumstances.

          • 天可汗 says:

            Imagine a Martian observer of Earth who takes the democracy stuff at face value. The state is the people, so the people are the state, so there’s really no difference between what the people do and what the state does. Everything that is done by the people is done by the state, which is the people; everything that is done by the state is done by the people, who are the state.

            In some places, people are tortured. In other places, people are blacklisted. Most cases of both don’t go viral. They just happen, and you don’t hear about them.

            Now, if your Martian is interested in political speculation, he might wonder if there are any patterns to this. What determines whether people are tortured or blacklisted or both?

            Surely “both” is the worst of the options, but how about torture vs. blacklisting? Torture is administrated by a central authority, and its target has the opportunity to recant. The Dark Lord Moldevort responded to his purge by asking if there was a creed he could assent to in order to be rehabilitated, but of course there isn’t and can’t be. Instead of torture followed by recantation and rehabilitation, we have blacklisting followed by… more blacklisting. This is known as “progress”, and it sure sounds like a reasonable response to “progress” is an upward revision of one’s opinion of Catholic theocracy, and I say that as someone whose glorious ancient culture revolves mostly around conspiracy theories about the Pope. What’s the worst that could happen — you have to get your condoms from the same place you already get your acid?

          • Brad says:

            600,000 people were killed in the Great Purge alone. Out of a much smaller population. The guy you keep going back to as some great victim — Yarvin — is not destitute. You can’t name a single person that’s destitute because of this purge you claim is analogous to Stalin’s.

            When you go way over the top with your analogies, it doesn’t become more convincing, it becomes less convincing. That’s the reason people try to avoid invoking Godwin’s law.

            And when you are called on going over the top, if you double down and say no really it *is* actually far worse, then you lose all credibility. Even if you think you don’t need to worry about credibility because you picked an unpronounceable handle.

          • Jack Lecter says:

            @rlms
            I’m glad to hear it, too- I kind of thought it was just me.
            I’m not a bigot, but I’m getting sick of having to preface support for civil liberties with “I’m not a bigot”.

            And this is exactly the kind of thing I was sure would never come up when people were arguing against gay marriage in the first place. It’s left a pretty bad taste in my mouth, to be honest.

            I’m still pro-gay-marriage, but things like this and the pizza place make it a lot easier to see where conservatives are coming from on this issue.

          • Questioner says:

            Brad says:
            Did we ever find out which of the martyrs, if any, is now destitute?

            Well, Masterpiece Cakeshop is no longer allowed to make wedding cakes, which cost them a large part of their business and forced them to lay off employees

          • Questioner says:

            Jack Lecter says:

            I’m still pro-gay-marriage, but things like this and the pizza place make it a lot easier to see where conservatives are coming from on this issue.

            I hope it also makes it easier for you to understand why there’s pretty much going to be no “give” on the Right on the transgenderism issue.

            We remember how quickly “if you don’t want to have a gay marriage, don’t have one” became “bake the cake, bigot!”

            So the response is “if you have a penis, you’re male. And if you try to go into a girls’ / women’s restroom / locker room, we want you arrested.”

            All sympathy is dead, because we saw where sympathy got us

          • Nick says:

            And this is exactly the kind of thing I was sure would never come up when people were arguing against gay marriage in the first place. It’s left a pretty bad taste in my mouth, to be honest.

            I’m still pro-gay-marriage, but things like this and the pizza place make it a lot easier to see where conservatives are coming from on this issue.

            Yeah. See Rod Dreher’s Law of Merited Impossibility for a succinct summary of the strategy here: “It will never happen, and when it does, you bigots will deserve it.”

          • Brad says:

            I’m not sure how bigoted bakers fit into the discussion at hand. Isn’t the discussion about whisper networks and being punished for not holding the correct beliefs?

            I don’t see how businesses being punished for violating public accommodation laws, which go back decades, have anything to do with the subject at hand, regardless of one’s position on those laws.

            What’s ‘thunder comes before lightening’ in the Masterpiece Cakeshop case?

            I understand some people have axes they are looking to grind, but you should probably look for at least a vaguely relevant context.

          • Nick says:

            I’m not sure whether you have me in mind, Brad, but I’d say my response to Jack’s post was relevant to it, and that Jack’s post, while at a tangent to rlms’, was entirely in line too. For that matter, Questioner’s post makes sense in response to rlms’: rlms was drawing a distinction between something he thinks is good (freedom of association) and something he thinks isn’t (government infringing on rights), and Questioner’s point, clearly, was to find a case of freedom of association that rlms would not accept, to show rlms either that freedom of association is not as uninfringeable as he thinks it is, or else that the cake shop should be fine too. No one mentioned “businesses … violating public accommodation laws” except you—but that’s fine, because that’s presumably either a reasonable limitation of freedom of association or it’s not, and that’s relevant to rlms’ having a consistent position on freedom of association.

            I acknowledge 天可汗’s argument is difficult to parse, not least because he’s speaking in parables now, but no one here has been grinding any axes. All the individual responses make sense in context. It seems to me it’s only by trying to impose one single topic on all this that they look irrelevant.

          • rlms says:

            @Nick
            Questioner’s line of argument was flawed, because it is perfectly possible to view freedom of association as generally good, and the categorical difference between negative effects from a government infringing on rights and negative effects from individuals freely associating as important, without viewing freedom of association as sacrosanct. But since they gave an example where I do favour the freedom of association side (and did so in an uncharitable way), I didn’t feel the need to explain this.

          • Nick says:

            rlms,

            Yeah, sorry: I should have clarified that the cake shop question was fair, but the “Or is the government destroying your business ok, when they’re doing it to pursue your preferred agenda?” was very much not. But I think my point was made that his question was on topic, if uncharitable and making an undue assumption.

          • Brad says:

            I don’t think Questioner’s original interjection in this thread was at all reasonable. S/he clearly has some kind of ax to grind about bigoted bakers. That initial impression was confirmed by the follow up post, which again was generic right wing table pounding, this time about trans people.

            The discussion between quints, Sniffnoy, and Tracy W was about whether or not it is reasonable to compare peer to peer criticism about what people think and say to Stalinist purges.

            Then Mr. Unpronounceable decided to characterize this peer to peer criticism as “losing the ability to pay your bills and most of your social connections” although he’s been unable or unwilling to give any examples of that. He then went on to further dig himself in the exaggeration hole by referencing Mussolini torturing people.

            That’s when rlms stepped in to point out that Mussolini was a dictator torturing people and not peers, in their private capacities, freely deciding whether or not to associate with someone based on their views. He did use the magic words “freedom of association” which is apparently what triggered Questioner to introduce his hobby horse, but Masterpiece Bakery has nothing at all to do with the distinction rlms was drawing between Mussolini as a state actor torturing people and peers dropping friends over perceived heresy (i.e. losing social connections).

            As I said, just random ax grinding. Unpronounceable’s position is absurd, but at least it is regarding the topic of discussion.

          • Nick says:

            Brad, rlms invoked freedom of association to defend his apparent (correct me if I’m getting this wrong, rlms!) contention that people losing their jobs for unacceptable views isn’t necessarily wrong. Questioner countered with a well-known case that invokes freedom of association, one in which folks who defend the firings typically argue it doesn’t apply for whatever reason, with his (uncharitably stated) implication that rlms either support the bakers too or consider freedom of association infringeable. I don’t think this means he has an ax to grind or to suppose that this is Questioner’s “hobby horse”, but even if it did, it’s not what makes his argument a poor one, and either way it hardly makes his post irrelevant.

          • Questioner says:

            rlms says:
            Questioner’s line of argument was flawed, because it is perfectly possible to view freedom of association as generally good, and the categorical difference between negative effects from a government infringing on rights and negative effects from individuals freely associating as important, without viewing freedom of association as sacrosanct.

            Well, both you and Brad are both wrong

            It’s not about “freedom of Association, and it’s not about public accommodation laws, sit’s about the right to disagree with orthodoxy.

            Phillips thinks that same sex marriage are not real marriages, and that therefore he doesn’t want anything to do with them.

            Either he’s allowed to have that belief without having his business be destroyed, or he isn’t. Those who hold he isn’t are modern versions of the Inquisition, different targets (so what?) but same totalitarian demand that all must bow before them.

            The “public accommodation” argument is so stupid that I wonder at the total lack of “sense of shame” among the people who make it.

            If a gay individual walks into Phillips store, and he refuses to sell them items available for purchase, that would be a violation of a public accommodations law.

            Refusing to make a custom product for use in a ceremony Phillips doesn’t approve of is not a violation of any sort of legitimate public accommodation law. “Common carriers” don’t provide “custom goods”.

          • Brad says:

            The answer is so completely obvious yet somehow Phillips never raised it before the Colorado Supreme Court. And instead conceded the public accommodation point.

            You never even bothered reading the key documents involved in the case you are demagoguing about, did you?

            Courts heard all these same claims made 50 years ago when your intellectual forebears were flipping out about miscegenation.

            And again this has zero to do with the subject at hand. There’s no “thunder comes before lightening” here. There’s just your unconvincing attempts to horn in on an argument whose entire logic was about leaving space for people to discover and share the truth. Bigoted bakers refusing to serve customers has zero to do with any kind of truth about the world.

          • Nick says:

            Oh for the love of God. I retract my claim that Questioner does not appear to have a hobby horse on new evidence.

          • Questioner says:

            @brad: There’s no “thunder comes before lightning here”

            Yes there is, there’s the claim that the union of two men 9or two women) is as valuable to society, and therefore as worthy of support, as a heterosexual marriage.

            We’ve got a ton of support for the idea that our society relies on heterosexual marriage for its well being. To take a simple case: insurance companies lowered rates for married males under the age of 25 because marriage made them more responsible, safer, individuals.

            There is no such evidence, that I know of, for same sex unions. Care to provide some?

            Lacking such evidence, there is no valid reason to grant such unions the benefits that society grants heterosexual marriages

            As for “read the documents”, Phillips pointed out that they were willing to sell the couple everything in the store, they just weren’t willing to make a custom cake for their “ceremony”.

            Before you knock others for not reading, you might try it yourself

        • But there are plenty of contexts where one can express those kinds of views without any fear whatsoever.

          This gets us back to the bubble issue. There are contexts where one can argue that AGW doesn’t exist or that men are smarter than women or that Obama was really born in Kenya. But there are groups where you can’t argue those things without serious penalties and where the members of those groups mostly limit their intellectual interaction to other members of them.

          If the result of saying X is that members of group Y no longer consider you someone worth taking seriously, then the logic of the situation within group Y is pretty much as Scott described.

        • Sniffnoy says:

          Again, I contend that you’re missing the point. The point isn’t if people are being hurt for speaking their minds; that’s just one means of the problem, not the problem itself. The problem is bad community epistemics (well, OK, a particular form thereof, but focusing on “are they hurting people” is still the wrong thing to focus on). Not merely “doesn’t converge to truth” bad but “goes into a well-known positive feedback loop and goes to hell while getting more and more unhinged” bad.

          So really as to the hurting people thing, whether you’re killing people or what… that’s really just a matter of degree. Obviously harsher punishments are wronger in and of themselves but even pretty small punishments can be enough to send you towards the positive feedback loop of doom. So for the purposes I care about I’m just going to ignore questions of degree of punishment.

          So. I want to outline the factors that I think you’ve ignored or the parts where I think you’re substantially wrong (well, those beyond the ones just mentioned above). I’m going to repeat myself a bit from earlier comments here if you don’t mind.

          1. You keep using the word “criticism” as if all the talking that’s being complained about is, you know, argument. Actual argument about the actual issues under contention. It’s not. We’re talking about “criticism” largely of the form “This person is a bad person, and so you should disregard everything they say.” Or things essentially similar.

          Let’s be clear — this is not argument. This is a failure mode of argument. The genetic fallacy… is a fallacy. Ad hominem… is not valid reasoning. Bulverism, psychologizing… to act like these things are actual criticisms of the ideas being discussed, is a mistake. Where you seem to see argument, I see things that shut down actual argument and destroy a community’s ability to reason. Unless we’re to suddenly believe that the genetic fallacy is in fact a good way to get at the truth.

          These sorts of demands for orthodoxy are often coupled with actual arguments, sure. Who cares? You throw in things that destroy the epistemic environment, the fact that also people are producing arguments is irrelevant; they’re not going to lead to truth under such conditions.

          2. So I’m hoping now you’re beginning to get an idea why I consider your claim “But there are plenty of contexts where one can express those kinds of views without any fear whatsoever” (which I will agree is true) to be mostly irrelevant. That’s not any sort of solution to bad community epistemics! Remember: Free speech is for the benefit of the listener, not the speaker! It’s not there so you can just feel the joy of saying things, it’s to make sure ideas actually get argued. If everyone worth taking seriously refuses to consider what you say because it’s you that’s saying it, it’s not much comfort that maybe a bunch of people you don’t care about might happen to find it interesting.

          In some cases, sure, splitting off and forming your own community may indeed be sufficient to get you what you want. But often it’s not. Imagine the following. You’re, I don’t know, a chemist somewhere. You have some ideas you think are really neat about… uh, I don’t know, I don’t know chemistry, about something chemical. Then suddenly all of chemistry blacklists you for some reason and decides you’re a crank. You can’t get articles published in chemistry journals, companies won’t hire you, nobody will even listen to you or pay any attention to your comments on chemistry blogs (they know what ideas you’re advancing and can recognize it must be you). It is then basically no comfort that, the community of chemists having kicked you out, the community of chemistry cranks will almost certainly gladly embrace you! Hell, it’s the opposite of a comfort — the last thing you want to do is further associate yourself with crankery! That’ll just demonstrate to everyone else that you really were a crank all along and it was all justified. (Not to mention that cranks are just awful company and even if it weren’t for the reputational effects you’d avoid them.)

          Thankfully I’m pretty sure chemistry doesn’t actually work like this. But map it onto whatever you want — “well you can go talk to cranks”, or the equivalent, is really no comfort.

          3. And so with the current situation. If you’re a Blue Tribe liberal, the idea that “oh well the Red Tribe will listen to you”… that’s not a comfort! That’s the threat, you know, if we’re talking about what are people actually being threatened with, they’re being threatened with being labeled as Red so that Blues will dissasociate from them, and their ideas will propagate only among people they fundamentally disagree with, not the people they think they’d be useful to as a supplement to their existing ideas.

          I mean there was a time when liberal online spaces (note: here as before that’s liberal in the American sense) seemed like a useful place to talk about things, you know? Maybe I’m wrong, because maybe the discussion was actually crap, but it certainly seemed like it at the time. Then… things shifted. In this case, at least, I’m fairly OK with how things have split off. But there’s enough overlap between the old and the new that it remains a problem; it’s not like people in the old groups have nothing of value to say, and it’s a problem that we often can’t seriously discuss it with them.

          But that doesn’t mean I can just disassociate everywhere! People say this stuff doesn’t come up in real life, bullshit it doesn’t, I lived in a housing co-op for over 5 years and you couldn’t escape it there. Let me tell you, living there you absolutely learn to judge what you say in public there and what you can’t. But I’m sure I can always just find another living situation and place to hang around, right?

          Well, maybe I can. But the scariest thing to me is this stuff taking over the academy. Academia’s entire specialty is supposed to be having good epistemology and getting things right. They’re supposed to be the leading edge of discovery. The place where human knowledge gets aggregated. They’re basically who acts as the arbiters of what’s known, in the literature, and what’s still unknown, not in the literature, in need of a solution. If the academy continues to fall to it… where then will there be to flee to? Who will replace it? Because for academia, for “the literature”, to fracture, with nobody really being able to say what’s known or not anymore, isn’t a good outcome. (I mean yes we are already having this problem not for political reasons but simply due to specialization and the overwhelming volume of literature but it is y’know a bad thing and this would be making it way worse.) And “well you can go talk to cranks” is definitely not going to cut it.

          • Aapje says:

            +1

            Very well argued!

          • qwints says:

            I’m not saying “well you can go talk to cranks” or “disassociate everywhere” or ““oh well the Red Tribe will listen to you.” I am saying that there are many ‘Blue Tribe’ contexts where there is a lot of tolerance for heterodoxy on the kinds of issues I believe the Scotts are referring to. Go to a Democratic Party event or a public book club or a liberal group happy hour, you will hear a ton of “heterodox” views.

            Of course, the problem when people are completely dependent on their employer to survive and that employer imposes its beliefs on its employees, then the employee is censored. This is an ubiquitous experience, but it’s not tied to a societal wide orthodoxy. The solution is to reduce the power that employers have over employees.

          • Sniffnoy says:

            qwints: I find your reply distinctly unsatisfactory, though I’m havin trouble pinning down just why. Let me try to state a few reasons, though I can’t guarantee this will be the most coherent, and it’ll probably involve quite a bit of overlap between the points (as well as repeating my comment above, sorry, hope you don’t mind).

            #1: I notice none of the venues you mention occur in writing. Writing, I think, is important. Writing lasts. Writing can be copied and referenced. Write an article, you can be cited. Write a comment, you can be linked to. That’s an important part of how an idea spreads. Writing comments on blogs may be a bit like shouting into the wind, but discussing things in real life seems way more so, unless you’re, like, directly talking to people who can influence things. (I already put too little of what I have to say in writing, I think…)

            #2: Another quality that seems to be lacking from what you suggest is any notion of being, you know, on the leading edge of truth. Like the bit above about a unified academia, a unified literature, where we can say what’s known and what’s not, is important. And various internet communities, such as LW or its diaspora, can function similarly. (I guess the whole unity thing goes together with the whole writing thing.) Why do I want to spend my time arguing with a book club, though? Is that where the best arguments on the subject are going to be found? Is that really where I’m going to encounter the best arguments against my position, so that we can really get to the heart of the matter? Maybe I’d convince some people of my position, but without some stake in the matter — if I’m not talking to people who are actually going to influence what happens in places I care about — then that’s not so interesting.

            #3. I don’t understand your advice regarding my living situation. I mean, I don’t live there anymore, though I still hang around there sometimes, but that’s not the point; we can imagine I am. You’re not saying to disassociate myself from there? OK, so what do I do then? Continue to carefully gauge what opinions can and can’t be expressed openly? Or what? What does one do about that problem?

            #4. More generally there’s this idea of… these are my friends you’re talking about, this is my home you’re talking about. Not necessarily literally my home (although also literally my home as mentioned) but that’s not the point. The point is, it’s, you know, a sad thing to see people and spaces I thought of as sane fall to this. OK, I can flee, sure. What’s going on is still bad and still has to be stopped somehow. Maybe some of these spaces can just be abandoned — maybe discussion there was never good, they were never anywhere near the leading edge of truth, you were never going to learn anything by arguing there, and they certainly didn’t have any real influence over anything you might care about. Like I said, I’m fairly OK with how I’ve abandoned a number of my old internet haunts. But sometimes, as I’ve already said, they can’t just be abandoned without cost. If there’s someone there who’d actually be worth arguing with if only they and the environment were amenable to actual argument, or someone who’s got a real sharp mind but turns into a zombie on this particular topic, it’s a real problem. If you literally live there or work there, it’s a real problem. (And this isn’t just a matter of the employer imposing their views on the employees; it’s also a matter of gangs of employees imposing their views on each other. The mob is just as scary as the tyrant.) When it’s an academic department, somewhere that really truly is supposed to be on the leading edge of truth, that can’t be easily substituted for, it’s a real problem.

            I was going to put some sort of conclusion here, but, eh, I’m tired. I hope I’ve made my point. Well, OK, here’s my one-sentence summary: It’s not so much about one’s personal situation, as it is rescuing the places where good, potentially leading-edge, potentially original discussion can both happen and be entered into whatever the local equivalent of “the literature” is. I’ll stop there.

          • qwints says:

            @Sniffnoy, perhaps it’s because we’re eliding the object level discussion that our conversation seems to be at cross purposes. I think that we vastly disagree about the meaningfulness of online discussion to society as as a whole.

          • Sniffnoy says:

            My suspicion is that the disagreement is over goals, not facts. My goal has little to do with society as a whole, and a lot to do with the garden. Perhaps I’ll elaborate on that later when I’m more up to it, but that’s the one-sentence version. (Well, one sentence that contains a link to explain an important concept. 😛 )

            Edit: BTW, I notice both your recent responses only respond to one part of my point. If I might ask, do you agree with the rest? (Or what?)

          • qwints says:

            I don’t really have anything organized to say about your other points. I’m enjoying our conversation, so here are some thoughts about the points you made I haven’t addressed:

            [Failure mode of argument]
            I agree that the fallacies you mention are common and fallacious. I have not observed that “things shifted.” I do not have a good sense of the extent to which these have become more prevalent, and I do not think anyone can make claims about this in the abstract – you need a baseline and a particular space to say anything meaningful about whether fallacies are more common.

            [writing]
            I disagree that writing in the broad sense is more important than speech. Recorded video and audio is at least as permanent and referenceable as a blog comment.

            [have to gauge what can be expressed openly]
            I think that human beings are social animals, and that it is literally impossible to communicate with someone without that impacting their view of you.

          • Sniffnoy says:

            I see! My own responses:

            I agree that the fallacies you mention are common and fallacious. I have not observed that “things shifted.” I do not have a good sense of the extent to which these have become more prevalent, and I do not think anyone can make claims about this in the abstract – you need a baseline and a particular space to say anything meaningful about whether fallacies are more common.

            OK, so, my claim isn’t just “these things are fallacies, and also they’ve become more common”. I mean, bad argument is everywhere, right? I think maybe I haven’t been clear enough as to just what my claim actually is. It’s this: This particular sort of bad argument — the declaring of large numbers of sources as somehow tainted and therefore useless (excpet of course when they’re not) — is part of the broader pattern of how groups, originally started to advance one cause, eventually become about advancing no cause but their own dominance. The key thing about this sort of argument isn’t just that it’s bad argument, but that, being fundamentally about people rather than arguments, it essentially does nothing but reflect ingroup/outgroup politics, or act as a medium for such. It’s not just bad argument, it’s politicking in the guise of argument.

            All this didn’t happen for no reason — it happened because of SJers pushing warped norms of discourse, presenting them as if they were just natural extensions of liberalism, liberalism done better, rather than being fundamentally illiberal. And I, and a whole lot of others, fell for it. But once you accept a little of it… well, it only goes in one direction, really.

            I disagree that writing in the broad sense is more important than speech. Recorded video and audio is at least as permanent and referenceable as a blog comment.

            Hm. I don’t think recorded video and audio changes much, really, because very little of what’s said is in fact recorded.

            That said I’m not sure the writing point was one of my better ones. If I think about, just, say, discussing things with other people in the math department, that hardly seems so bad for not being written down. But I guess because that’s a smaller world, and one I care about; things I say there might actually be remembered and spread around if they’re good. I dunno.

            That point should probably just be discarded. But it is worth noting how much better writing lasts, and I think it does really become important when your community become sufficiently large; an oral “literature” just doesn’t work at that point. Like, I’ve totally dug up old comments — of mine, of other people’s — when I wanted to make a point that I or someone else has already made or find someone’s exact wording or whatever… can’t really do that otherwise. (Imagine if I couldn’t link “garden” above…) Really I should just start writing things down more often, in accessible places, in general! Often I have to do some real digging to find a point I’ve made before…

            I think that human beings are social animals, and that it is literally impossible to communicate with someone without that impacting their view of you.

            I mean, sure, but there’s a real difference between “some people will decide they don’t like me” and “I’ll become marked to everyone as one of the bad people (whose ideas, let’s not forget, are tainted and should be disregarded).”

    • Desertopa says:

      Can you think of some point of reference that’s clearly better? Something where the members of the audience will all recognize it, understand that it’s a legitimate case of an inaccurate orthodoxy being enforced on people over correct knowledge, with consequences for the people who fail to at least pay lip service to the orthodoxy, which are not trivial, so people can understand that there’s real social force for the people involved, but the consequences aren’t so extreme as to offend anyone by comparison to consequences on modern issues?

      Examples which are too perfect for any criticism often do not exist.

    • Eponymous says:

      I aspire to do better than “Not literally as bad as Stalinist Russia”. In fact, I think we have to do *very much better* than that for our civilization to survive.

    • The logic of the situation may be the same, even though the costs that can be inflicted are much less.

      • rlms says:

        There is a thin line between illustrating a principle with examples that differ greatly in valence, and The Worst Argument In The World.

        • Paul Brinkley says:

          There might be concern that, while we’re not literally as bad as Stalin, that we are headed that way. People aren’t arguing about the current value; they’re arguing about the derivative.

          At least, that’s how I interpret arguments on the internet about how taxation is theft, ACA will make doctors into slaves, conservatives are trying to institute sharia law, a ban on guns is just around the corner, the GOP is waging war on women, etc…

  78. Clarkey says:

    Why is meta-level discussion of unspeakable truths such as this (as well as Aaronson and Graham) largely immune to the cudgel? If the effect is as strong as you suggest in the lightning example, wouldn’t meta-level discussion also be banned?

    Also, worth noting for the sake of discursive continuity that this falls into the category of cognitohazard discussions.

    • Sniffnoy says:

      I mean, under Stalin, it sure wouldn’t be. The fact that most Americans believe in the notion of free speech — by which here I mean the words “free speech”, though not necessarily much of the actual idea — may play a role; explicitly arguing against it, at least, may look bad. But I think the bigger explanations are: There’s no central authority doing this, just a popular movement, which relies on people’s enthusiasm; and the more levels of indirection, the less enthused people are going to be about going after it. And then on top of that, in many circles the core of this movement just isn’t large enough to accomplish what it’s doing alone, instead relying on popular support from people on its periphery, so it has to be able to convince them, too. (And then on top of that, in some circles it has to get not just people on its periphery, but people actually in the general population, limiting it even further.)

    • It isn’t immune. See t mes wielding his cudgel below.

      • hnau says:

        Exactly. “Dog whistle” means “everyone’s smart enough not to say [banned thing] openly, so when you say [accepted thing that relates to banned thing] it’s as if you’d said [banned thing]”.

        In some ways this whole post was about dog whistles on an overt level. Which I guess is ironic?

        • 天可汗 says:

          …at which point the formerly accepted thing is banned and the cycle starts all over again.

        • Edward Scizorhands says:

          Every time I see someone reference “What You Can’t Say” I remember a progressive colleague, having been seen a link to the essay, saying “oh, so I guess Paul Graham wants to be a racist but is upset that he can’t say he’s a racist.”

      • t mes says:

        I wasn’t being uncivil. Seems like a leap to call it cudgel wielding. I was just pointing out he might be wrong about which side is the thunder side and which is the lightning side.

        This discussion does point out a secondary problem to dog whistles and whisper networks. Not only is the outgroup position not allowed to enter discourse on equal footing, the in group has a hard time legitimately attacking the outgroup position because the outgroup can always retreat with “no i wasnt really talking about that”. So the true side has a hard time winning once and for all regardless of which side of the fence it is on.

        • Sniffnoy says:

          Like, OK, fine, you were just trying to talk about the object level things Scott might be implicitly discussing… but such things isn’t the topic here. And attempting to argue such things in this topic is… not really going to contribute to the cause of clear thinking, you know? Go discuss such things in another thread where it’s more relevant, or the open threads that aren’t culture-war-free. Talking about it here — and you’ve admitted, it’s not actually relevant to the point, so there’s no reason that here needs to be the place you talk about it — is just going to bring out tribalism in people and make the discussion worse.

          (Also, if you’re confused about the response you’re getting here… remember that people will generally read your comments in light of Grice’s maxims. If you say something clearly irrelevant, people won’t just assume you’re spouting irrelevancies for some reason; they’ll assume you believe it to be relevant. You’re only going to cause confusion if you go around saying things that aren’t relevant without explicitly marking them as such — especially if they’re things that other people honestly do believe to be relevant.)

          • t mes says:

            I did not concede it is not relevant to the point. Did Scott concede he had issues in mind himself? I don’t see my comments as inappropriate, off topic or impolite.

            If your critcism is that I could have been more explicit in stating what I’m talking about… I see your point but thats kind of ironic in this context isn’t it?

          • Sniffnoy says:

            You said your reply to my earlier comment here that the correctness of what particular examples Scott might have in mind does not bear on the epistemological point being made. Given that, what do you possibly expect to be gained from trying to discuss such particular examples here? Because there’s quite a bit to be lost — it’s going to make the whole discussion more mindkilling. Talking explicitly about examples is going to make people focus more on the examples than on the important point and tie the important point to the examples in their mind so that those who disagree with Scott will be more likely to dismiss the important epistemological point. There is a reason this post was written in the style that it was, and part of that is to keep people from dismissing the point out of hand based on (not actually relevant, but associated in their mind) examples. Don’t encourage such behavior!

            I’m not saying being more explicit. (Well, I am saying be more explicit about what you consider relevant or irrelevant.) I’m saying this is a bad thread to talk about such things on, given that you have plenty of other options for talking about such things here on SSC.

          • Nancy Lebovitz says:

            I suggest taking unspeakable object level objects to the current OT.

          • Jiro says:

            You said your reply to my earlier comment here that the correctness of what particular examples Scott might have in mind does not bear on the epistemological point being made. Given that, what do you possibly expect to be gained from trying to discuss such particular examples here?

            When someone removes the details of some real life situation and tries to get the audience to agree to some abstract principle, that means that the audience has to trust that he is properly mapping the situation to the principle.

            And real human beings are usually terrible at mapping real-life situations to principles like these and not leaving anything out, assuming things they have no right to assume, or otherwise breaking the mapping in some undetectable (because it’s unsaid) way.

            There’s a reason why crackpots constantly compare themselves to Galileo. This whole blogpost is basically “look at all these Galileos. Galileos get persecuted. Implicature means I think that I or someone I support is like Galileo, but I won’t tell you who it is. Trust me, they’re similar.”

            Scott could be referring to Damore or Horrible Banned Discourse, but the same thing that Scott is saying could equally well be said by a Holocaust denier, a homeopath, or someone designing a perpetual motion machine, and that person would sincerely think that it all applies to him.

            The details matter.

          • Sniffnoy says:

            Jiro: Good point! However let’s note that the details that you’re talking about are the details of, are the epistemological practices being complained about actually analogous? Rather than, are the object level claims all this is happening over actually correct?

          • Jiro says:

            There’s always epistemic learned helplessness. If someone creates a perfectly valid-sounding argument and successfully uses it to prove something that is sufficiently unbelievable such as homeopathy or flat-Earth, that’s Bayseian evidence that there’s something wrong with the argument and I just haven’t noticed it.

            In some cases, such as Holocaust denial, it’s even Bayseian evidence for lying. Not to mention that I may not want to argue the Holocaust with a Holocaust denier in the first place.

            Also, whether the object-level claims are correct often directly affects whether the situations are analogous–they can’t be as easily separated as you suggest. The homeopath is not being rejected by the medical profession because they’re pulling a Galileo on him; he’s rejected because homeopathy is nonsense. But I can’t figure that out if he doesn’t reveal that he’s talking about homeopathy.

        • Peffern says:

          @Sniffnoy

          The ability to write these things in such a way that people don’t immediately figure out what it is ‘really’ about and them immediately either dismiss it as heresy or preach it as the orthodoxy.

          I remember a while back there was a CGP Grey video along the lines of Scott’s Toxoplasma post, and at one point, he cuts the middle of the video and inserts something like “Stop. Whatever issue you think this video is about, it isn’t about that. Put that away” and I remember realizing that he had caught me mid-thought-forming-in-my-head and I felt ashamed for a moment, then realized he was right.

          There’s very obviously, as you say, a reason why the post is structure in the way that it was. Anything enlightening on how?

        • the in group has a hard time legitimately attacking the outgroup position because the outgroup can always retreat with “no i wasnt really talking about that”.

          A second reason is that someone who suspects the outgroup position might be correct will reasonably reject even correct arguments for the ingroup position on the grounds that he hasn’t had the opportunity to see if there are rebuttals. And defenders of the ingroup position will be reluctant to offer responses to the strongest outgroup arguments, for fear that some may be persuaded by them. The only people who can give convincing arguments for the ingroup position are people identified as heretics who happen to agree with it.

          I’m thinking of a real example–the issue of innate racial differences and Thomas Sowell’s discussion of it in Ethnic America. A reasonable person observing the orthodox approach to the issue, which amounts to “we know there are no such differences and anyone who thinks there are should be shunned as a racist,” will reasonably suspect that the reason blacks earn less than whites is that they are less intelligent.

          Sowell, being actually and obviously willing to offend orthodoxy, can present evidence–the success of West Indian immigrants–that is inconsistent with that view and be believed. It helps that he offers the same evidence in arguing against the other half of the orthodoxy–the claim that the real explanation is racism.

    • Jeremiah says:

      It’s not entirely immune. I was just looking through Paul Graham’s quote page, and I came across this:

      I had my own reactions to Paul’s essay– on the whole I liked it but when I connected some dots I found some suggestions of things I strongly disliked– not so much in the essay as suggested by it.

      – reaction to Paul Graham’s “What You Can’t Say”

  79. skybrian says:

    The whisper network: also known as gossip. Many true things are gossiped about before they become common knowledge. Also many false things.

    People who like to believe only verifiable things will avoid gossip, and then what happens?

  80. Razib says:

    fool! why speak now? the prophet shall come, and drive all before them. the orthodox shall burn and the heretics shall rejoice. the meek shall bend before power as they always have and always will. from violence we came, and to violence we shall go. the long peace is at an end….

    🙂

  81. suntzuanime says:

    The problem with the last paragraph is, how do you coordinate that? The common knowledge has spread, some of us are ready to start talking about it, and you’re still in “hush you fools” mode. How are you distinguishing “safety in numbers” boldness from “cringeworthy edgelord” boldness?

    • hnau says:

      The Deng and Zhao example is one possibility– you’ve got a group ready to come out of the woodwork when the lightning-and-thunder folks lose power.

      Another possibility is that everything happens by degrees. If you can get a lot of people to test the lightning-and-thunder rules just a little bit, you can make it more socially acceptable and move the needle a little bit at a time without attracting too much attention from the authorities (and if they do try to crack down they’ll appear unreasonable). When the authorities make an example of an edgelord it often scares people away and moves the needle back in the other direction, so doing everything by whispers may be a better way to make progress.

  82. hnau says:

    The thunder-and-lightning example seems like a bad comparison for this kind of situation, in that the false claim is (1) easily observable to be untrue, and (2) utterly useless to the society that propagates it. Neither of those statements applies to Stalinist communism or medieval Christian theology. (That doesn’t imply any particular judgment about whether those belief systems were true or good on balance, just that, Chesterton’s Fence-style, they existed for some reason better than serving as a strawman example.)

    I imagine you might respond that inane claims *do* serve a purpose. Forcing everyone in society to agree that “lightning comes before thunder” is a cheap and easy way to detect people who might also disagree with “the king rules by divine right”. This is true even if it’s not the overt motivation behind believing “lightning comes before thunder” (in fact, it almost certainly won’t be). This kind of honeytrap may be socially beneficial (from the divine-right fans’ point of view), even if accidental.

    But that situation would still beg the question of whether the edgelord types are missing something. Is divine right really beneficial? (For whom?) Often the edgelord perspective glosses over the fact that something besides “truth” is at stake. One might unwittingly take sides in a social or cultural power struggle, or destroy a useful institution, based on one’s perception of “truth”.

    (For the record, I don’t think that’s what’s going on with religion– or at least it isn’t the only thing going on. But the possibility is worth considering.)

    • hnau says:

      Oops… should be “thunder comes before lightning”. Apparently I out-Kolmogorov’d myself. 🙂

    • Steve Sailer says:

      You can substitute Lysenkoism as a real world example in the Soviet Union.

      • 6jfvkd8lu7cc says:

        Well, there things are less obvious: actually, some people kept believing some of Lysenko’s claims after explicitly trying to verify them.

        There was a paper published in Soviet Union with reproduction of Mendel’s experiment on relatively small populations, which alleged to contradict the Mendel’s laws. Actually, if analyzed with a good understanding of probability distributions, the experiment confirmed the Mendel’s laws.

        Kolmogorov immediately published a probability theory proof that this experiment had confirmed Mendel’s laws, which made Lysenko explicitly declare separate magisteria between mathematics and biology.

    • David Shaffer says:

      Don’t both of those things apply to both Stalinism and Dark Age Christianity? The harm of Stalinism (central bureaucrats don’t have the knowledge to effectively implement central planning, are highly vulnerable to corruption and both of these things being true, no one has an incentive to do good work) isn’t necessarily obvious until you try to implement such a system, but it doesn’t take long before the flaws become apparent once you do. As for the medieval church, these people were literally pushing faith as a virtue! It takes about five seconds before someone’s going to think, “wait a second, don’t I need evidence before I can have the trust needed for faith? And once I seek out evidence I’m no longer the guy who ‘believes though he has not seen’?”

      Neither of these things will necessarily lead to capitalism or atheism immediately. What’s going to seem more likely, especially to a scientifically-minded person who is probably more focused on questions of fact than on questions of politics-that the whole of society is wrong on something this obvious, or that the scientist has made a mistake? And so, anyone who cares about truth is going to be gaslit horribly as they wonder why nothing adds up. But if the whole world claimed that lighting and thunder were reversed, wouldn’t the exact thing happen to anyone who bothered to actually look at a thunderstorm? “It sure looks like the thunder is following after. Wow confirmation bias is strong, clearly I need to train my perceptions better!”

      As for being utterly useless, both Christianity and Stalinism would appear useless unless one bought in to the parts of the doctrines that explain why they’re actually super important (eternal salvation and/or a better economy). It wouldn’t take long for any society that decided to make lighting a political issue to come up with a reason why this was a big deal. Consider how parts of modern social justice have decided that video gaming is a mark of misogyny, rather than just a hobby like any other, or that futurism article Scott reviewed recently that decided that there were more white males in the optimistic and singularitarian futurist camps, and therefore believing in a good and/or singularitarian future must mark one as a heretic Evil White Male.

      Sure, lightning vs. thunder doesn’t obviously fit into an overarching narrative for us, because we haven’t seen it politicized. But couldn’t one just as easily imagine Stalin welcoming help on implementing the Five Year Plan better, because isn’t the important thing to build a worker’s paradise, and if Kantorovich can help out, he’s a Hero of the Soviet Union? Couldn’t one imagine a Christianity where bloody wars weren’t fought over whether the Communion turning into flesh and blood was literal or symbolic because Jesus didn’t specify, He just said to practice Communion? Anything can be politicized, and with disturbing ease.

      • hnau says:

        the false claim is (1) easily observable to be untrue, and (2) utterly useless to the society that propagates it.

        Don’t both of those things apply to both Stalinism and Dark Age Christianity?

        A) On the object level, no, they don’t.
        1) Just off the top of my head: “Dark Age” refers to a different period than the one discussed here (see: the entire last week of posts) and the medieval church mostly didn’t use “faith” to mean what you’re assuming it means. As to Stalinism, I didn’t claim that it was economically efficient– just that it wasn’t *manifestly* untrue as a belief the same way thunder-before-lightning is. You can find people even today who make cogent arguments in support of medieval Christian theology or central-planning-style Communism.
        2) In hindsight we might conclude we’re better off without them. Or we might not. There are plenty of people who think that religion and/or authoritarianism, even if unjustified, are good for a society (other things being equal). In any event it’s undeniable that there was *some* social impetus for them to exist (Chesterton’s Fence implies that much) and that they had some significant impact on society (which thunder-before-lightning doesn’t).

        B) On the meta-level, this is still missing the point. It’s a category error to reason from “the wrongness of Christianity and Stalinism seems obvious to me” (as it clearly does for you) to general conclusions about how to behave in the present.

        – First of all, we live in a very different kind of society and culture; *of course* Christian Europe and Stalinist Russia would seem manifestly unreasonable to us! So the apparent unreasonableness tells us much less than you might think about who’s actually right or wrong. (“Yeah, but can’t we see in hindsight that they were so much worse?” Nope, still missing the point. If they could see our modern-day society they’d likely be just as aghast.)

        – Second, we shouldn’t confuse outside view with inside view. We see these other worldviews from the outside, with the benefit of hindsight and massive selection bias toward history’s winners. We see our modern-day disputes from the inside. Our model for reasoning about current events shouldn’t be our own confidence about Stalinist Russia; it should be Stalinist Russians’ constant disagreement and near-complete cluelessness about Stalinist Russia.

        – Third, and most important, we should practice epistemic humility (see point A above). From the inside, all strongly held beliefs feel much the same, regardless of how they’re supported or arrived at. That is, it’s hard to internally distinguish between “I am highly confident in this belief based on examining all available evidence” and “I hold this belief for strong cultural, social, or personal reasons that have little to do with the evidence”. If you want proof of this, consider all the thinkers who were/are totally right in one area and horribly wrong in another, yet similarly forceful about both beliefs. The big risk I see in promoting edgelord-like beliefs is that it correlates very well with strength of belief but much less well with accuracy of belief.

        That last point could use a bit more illustration/elaboration. So: Anyone remember where Stalinism came from? That’s right– it was more or less a direct consequence of Marxism, the favorite belief system of “freethinkers” (read: edgelord types) for the previous half-century or more. By and large, the same people promoting the cultural/social/religious ideas that we now celebrate as “progressive” were also fervent believers in an economic system that we mostly consider discredited. It doesn’t look like their edginess did much to keep them from being wrong.

        • David Shaffer says:

          Thanks for the response! Thoughts on your post:

          “Dark Age” refers to a different period than the one discussed here

          You are correct; my apologies.

          the medieval church mostly didn’t use “faith” to mean what you’re assuming it means

          Do you have sources? I was under the impression that the church has pushed a blind faith line for the entirety of its history, plus or minus a few apologists who try to pretend otherwise using motte and bailey tactics e.g. C.S. Lewis.

          it wasn’t *manifestly* untrue as a belief the same way thunder-before-lightning is

          Maybe the failures aren’t quite as obvious, though for Holodomor victims one might argue they were more so. On the other hand, Aristotelian motion (objects move in a straight line until they run out of impetus, then drop) was widely and dogmatically believed for centuries, even though anyone who bothered to throw a rock could have noticed the manifest untruth of the model. This seems like a fairly close historical parallel to lighting-heresy.

          You can find people even today who make cogent arguments in support of medieval Christian theology or central-planning-style Communism.

          Cogent arguments? Certainly there are people who argue for such things, but I have not encountered coherent reasoning from Christian apologists, and I’ve read a fair amount of apologetics.

          On the meta-level, this is still missing the point. It’s a category error to reason from “the wrongness of Christianity and Stalinism seems obvious to me” (as it clearly does for you) to general conclusions about how to behave in the present.

          This is absolutely correct. Just because we can see the error of Stalinism or blind faith doesn’t mean that spotting an analogous error from the inside is easy. The outside view suggests that we could be making similar mistakes today, while being mostly or completely blind to them. The point I was making wasn’t “these are easy mistakes to spot, therefore we can be confident in our ideas today.” Rather, it was that these are mistakes that persisted in spite of having easily observable flaws, moreover their perceived utility to society is dependent on a narrative without which they would seem potentially irrelevant. Therefore, the fact that lightning/thunder is obvious if one observes carefully wouldn’t necessarily prevent it from taking a similar role in orthodoxy, especially since any society that turned this into a dogma would almost certainly construct a narrative in which believing that thunder came first was absolutely not utterly useless, but essential. And in that situation the fact that the mistake is obvious to someone who didn’t grow up accepting it doesn’t mean that it’s easy to spot if you’re used to it. Scott’s point isn’t about falsehoods that everyone can easily see; it’s about falsehoods that are easy enough to notice that the intelligent and curious tend to notice that they’re confused about them, but are hard enough to pick up on (at least from the inside) that getting everyone to realize the problem is difficult, and creating common knowledge of the problem is harder still.

          • hnau says:

            Thanks for the reply! Your request for sources on “faith” pushed me to do some actual research on the subject. (I find C.S. Lewis’s understanding of “faith” helpful in some ways, but I agree that it isn’t a typical one.)

            The premodern Christian document on the subject that I’d actually read most recently was Luther’s Preface to Romans, which has this helpful passage:

            Faith is not that human illusion and dream that some people think it is. When they hear and talk a lot about faith and yet see that no moral improvement and no good works result from it, they fall into error and say, “Faith is not enough. You must do works if you want to be virtuous and get to heaven.” The result is that, when they hear the Gospel, they stumble and make for themselves with their own powers a concept in their hearts which says, “I believe.” This concept they hold to be true faith. But since it is a human fabrication and thought and not an experience of the heart, it accomplishes nothing, and there follows no improvement.

            Faith is a work of God in us, which changes us and brings us to birth anew from God (cf. John 1). It kills the old Adam, makes us completely different people in heart, mind, senses, and all our powers, and brings the Holy Spirit with it. What a living, creative, active powerful thing is faith! It is impossible that faith ever stop doing good. Faith doesn’t ask whether good works are to be done, but, before it is asked, it has done them. It is always active. Whoever doesn’t do such works is without faith; he gropes and searches about him for faith and good works but doesn’t know what faith or good works are. Even so, he chatters on with a great many words about faith and good works.

            Faith is a living, unshakeable confidence in God’s grace; it is so certain, that someone would die a thousand times for it. This kind of trust in and knowledge of God’s grace makes a person joyful, confident, and happy with regard to God and all creatures. This is what the Holy Spirit does by faith. Through faith, a person will do good to everyone without coercion, willingly and happily; he will serve everyone, suffer everything for the love and praise of God, who has shown him such grace. It is as impossible to separate works from faith as burning and shining from fire. Therefore be on guard against your own false ideas and against the chatterers who think they are clever enough to make judgements about faith and good works but who are in reality the biggest fools. Ask God to work faith in you; otherwise you will remain eternally without faith, no matter what you try to do or fabricate.

            So Luther contrasts “faith” with “unbelief”, but he also seems to deny that “faith” is equivalent to saying “I believe”. My interpretation is that, for Luther, the relevant meaning of “faith” was not about believing the historical/philosophical/scientific propositions in the Bible– because everyone in his time did that! Instead, it was about personally taking to heart certain spiritual promises in the Bible and gaining confidence as a result. To summarize: “faith” for Luther meant “trusting in God”, not “believing God exists”.

            Needless to say, Luther is not a typical medieval theologian, so I also looked up Aquinas on the subject. He’s a little harder to parse, and being a philosopher he does seem to think of faith as something pertaining to propositions. But he characterizes faith as only applying to theological/spiritual knowledge (“First Truth” or “Divine Truth”), not propositions that are contingent or could be known through reasoning and observation. So I’d guess that he more or less belongs in the “non-overlapping magisteria” camp re: faith and science.

            Finally, Wikipedia seems to have a decent summary of what Christians generally mean by “faith”. You’ll notice that there’s a lot of discussion of the trust/loyalty aspects and the theological implications of faith, with practically no mention of “faith” in the sense of “blindly believing things the Bible says”; in fact the Roman Catholic section explicitly denies this sense.

            Hopefully this clears things up.

          • Nick says:

            Needless to say, Luther is not a typical medieval theologian, so I also looked up Aquinas on the subject. He’s a little harder to parse, and being a philosopher he does seem to think of faith as something pertaining to propositions. But he characterizes faith as only applying to theological/spiritual knowledge (“First Truth” or “Divine Truth”), not propositions that are contingent or could be known through reasoning and observation. So I’d guess that he more or less belongs in the “non-overlapping magisteria” camp re: faith and science.

            Aquinas’ take on it is sure to be a bit more propositional than most, but Christianity in general was a pretty propositional faith, at least until the 20th century got hold of it. I’d recommend not reading “non-overlapping magisteria” into Aquinas’ position, though, for a number of reasons; chiefly, it’s anachronistic, and it’s not how he would have divided fields of knowledge (and arguably not how we should either). The medievals didn’t think any sort of conflict could occur between things we know from revelation vs things we know from reason vs things we know from natural philosophy, etc. More importantly, though, they understood that it’s good to have reasons for one’s faith; it’s not for nothing that the Catholic Church teaches the existence of God can be known without the aid of revelation, and Aquinas and other medievals (and folks since) engaged in natural theology, reasoning about God in just that way.

            This was all a very long time ago, but Catholics have the benefit of continuity with a long tradition, so don’t discount its relevance. Thus my reply to David’s original question:

            I was under the impression that the church has pushed a blind faith line for the entirety of its history, plus or minus a few apologists who try to pretend otherwise …

            Also, David, can you use italics or blockquote or something when you quote someone? It’s very confusing reading your posts otherwise.

          • The original Mr. X says:

            Maybe the failures aren’t quite as obvious, though for Holodomor victims one might argue they were more so. On the other hand, Aristotelian motion (objects move in a straight line until they run out of impetus, then drop) was widely and dogmatically believed for centuries, even though anyone who bothered to throw a rock could have noticed the manifest untruth of the model. This seems like a fairly close historical parallel to lighting-heresy.

            Aristotle did actually have an explanation for why projectiles keep moving after they’ve been thrown, although I can’t remember what it is at the moment.

          • B_Rat says:

            Maybe the failures aren’t quite as obvious, though for Holodomor victims one might argue they were more so. On the other hand, Aristotelian motion (objects move in a straight line until they run out of impetus, then drop) was widely and dogmatically believed for centuries, even though anyone who bothered to throw a rock could have noticed the manifest untruth of the model.

            The blind reliance on Aristotelism of medieval scholars is a myth. Critiques and alternative theories of motions were developed already by John Philoponus in the VI Century, and later the quasi-inertial theory of impetus (a concept unknown to Aristotle) which responded to several objections to Earth’s possible motion was championed by figures like Buridanus and Nicholas d’Oresme.

    • Nate the Albatross says:

      It seems to me that the lightning and thunder example was chosen specifically because it isn’t a real world example. If he used an example that is more complicated like Judaism or a Guaranteed National Income then everyone would get sidetracked with debating the example and miss the point. I could mention here some obvious observable falsehoods regarding your examples, but that just falls into the trap. It is just easier in this case to stick to metaphors like lighting and the Emperor having no clothes in order to wrap our minds around the topic. A lot of things we hold as orthodox today will be proved wrong in the future, and debating the costs and benefits of a particular orthodoxy doesn’t really help people who find the flaws.